分からない問題はここに書いてね274

このエントリーをはてなブックマークに追加
1132人目の素数さん
さあ、今日も1日頑張ろう★☆

前スレ
分からない問題はここに書いてね273
http://science6.2ch.net/test/read.cgi/math/1172008460/
2132人目の素数さん:2007/03/19(月) 21:13:09
糞スレ立てんな>>1死ね
3すばらしいBlog:2007/03/19(月) 21:13:44
4132人目の素数さん:2007/03/19(月) 21:14:02
この先も統合案は完全無視でOKですか?
5132人目の素数さん:2007/03/19(月) 21:15:07
おk
6132人目の素数さん:2007/03/19(月) 21:17:56
7132人目の素数さん:2007/03/19(月) 21:40:49
2x^2-3xy-2y^2+x+3y-1

の因数分解教えてください
8132人目の素数さん:2007/03/19(月) 21:56:20
>>1
O2
9132人目の素数さん:2007/03/19(月) 21:58:12
>>7
xかyのどちらかで整理するのが定石

2x^2-3xy-2y^2+x+3y-1
= 2x^2 -(3y-1)x -(2y^2-3y+1)
= 2x^2 -(3y-1)x -(2y-1)(y-1)
= {2x +(y-1)} {x-(2y-1)}
= (2x+y-1)(x-2y+1)
10132人目の素数さん:2007/03/19(月) 22:14:37
フェルマーの定理の証明教えて
11132人目の素数さん:2007/03/19(月) 22:23:31
私は証明を知っているがそれを書くにはこの余白は狭すぎる
12132人目の素数さん:2007/03/19(月) 22:27:11
>>9
ありがとうございます
13132人目の素数さん:2007/03/19(月) 23:20:20
>>10
A={1,2,,,,,p-1} (p: prime)としたとき、任意のxに対し xA=A(mod p)を使う。
14132人目の素数さん:2007/03/19(月) 23:53:55
>>997
Bのグラフだけでも解けるが@Aのグラフで解いた方がイメージはしやすい

そのうち解の個数を調べる問題でこの考え方は非常に重要になってくる
間違っても、何がなんでもとにかく定数分離、
なんてことはしないように。
15前スレ905=988:2007/03/19(月) 23:58:19
>>前スレ995
(iii)のrとsはその通り ゴメン
>>993は誰か別の人
Nは平方和を3つかけた形で書ける、という意味だと思われ

>>988>>904を見てすぐ思いついたわけでない
以前この問題を解いたことがあっただけ
その時はピタゴラス数が大きなヒントになった
16132人目の素数さん:2007/03/20(火) 02:10:26
>>15
最初にこの問題を解いたのはいつ頃ですか?
俺今高一なんですが、これから先こんな問題出てもまるで解けそうにないですよ。。
17前スレ905=988:2007/03/20(火) 12:34:17
>>16
最初に解いたのは中3の時
整数の勉強がしたいなら数学オリンピックの過去問解説か数論の入門書を読むと良いかも
いくつかのパターンが見えるようにはなると思う
大学受験の観点から言えば、深入りしない方が身の為

そろそろスレ違い?
18132人目の素数さん:2007/03/20(火) 16:24:19
>>16
簡略な解答っぽいものを書いておく

4つの解をa,b,x,yをと置いたとき
N=a^2+b^2=x^2+y^2
M=(a+x)(a-x)=(y+b)(y-b)

よってM,Nは自然数s,t,uを用いて以下のように表せる
M=s*(t*u)=(s*t)*u
4N=[s-(t*u)]^2 + [(s*t)+u]^2
 =(s^2+u^2)*(t^2+1) (注:t^2+1≠4m)

3つ組の解があるときの十分条件は
(s^2+u^2)が異なる二つの解を持ち、かつ4の倍数になることである。
4(s^2+u^2)=(2s)^2+(2u)^2を用いれば
2つ組の解から直ちに3つ組の解が求まる。

必要条件については考察が行き届かなかったが
とりあえず325以下に解がないことを確認して許してもらうことにする
19132人目の素数さん:2007/03/20(火) 23:47:54
>>17
なるほど、参考になります。
この問題自体も自分の好きで挑戦してみたやつなんでちょっと深入りしてみますね。。
>>18
丁寧にありがとうございます。3つの時もその延長だったんですねぇ。
ただ一つだけ理解できないところが…。なんでt^2+1が4の倍数だといけないんですかね?

あと、A=a*b=c*d が成り立つときはいつも
A=s*(t*u)=(s*t)*u とか
A=(p*q)*(r*s)=(p*r)*(q*s)
みたいな感じに表すことがてきるんですか?
20132人目の素数さん:2007/03/20(火) 23:59:20
cos^2(3X)の積分を教えて下さい?
21132人目の素数さん:2007/03/21(水) 00:02:41
>>19
後半は何が言いたいのか分からないが、大きく脱線していないか?
22132人目の素数さん:2007/03/21(水) 00:02:44
整式の除法なんですが

A=3x^3-5x^2-7x+7

B=3x-2

の計算を教えてください
23132人目の素数さん:2007/03/21(水) 00:03:37
cos^2(3x)={1+cos(6x)}/2
24132人目の素数さん:2007/03/21(水) 00:04:22
>>22
普通に筆算しとけ
25132人目の素数さん:2007/03/21(水) 00:06:01
そのやり方が
26132人目の素数さん:2007/03/21(水) 00:08:19
>>22,25
なら教科書読め。
27132人目の素数さん:2007/03/21(水) 00:10:42
教科書ないんです><
28132人目の素数さん:2007/03/21(水) 00:12:57
>>27
じゃ、
3x^3-5x^2-7x+7=(3x-2)(x^2+ax+b)+c
とおいてa,b,cについて3つの式を作りa,b,cを求めろ。
29132人目の素数さん:2007/03/21(水) 00:23:58
>>27
教科書買えや
30132人目の素数さん:2007/03/21(水) 00:29:50
31132人目の素数さん:2007/03/21(水) 00:29:58
Ax+By+C=0
Dx+Fy+Z=0
を連立したときに係数を合わせてyを消去するとxが出ますよね?

Ax^2+Bx+C=0
Dx^2+Ex+G=0
を連立した時は
係数を合わせてx^2を消去してもxが出ないのは何故ですか?
32132人目の素数さん:2007/03/21(水) 00:32:23
>>31
変数が一つしかないから
二つの式が同値でなければ
解が求まるとは思えないが。
33132人目の素数さん:2007/03/21(水) 00:32:53
>>31
AE-BD=0 なんだろ
34132人目の素数さん:2007/03/21(水) 00:47:14
連立方程式を学ぶとき、なぜ連立させるのかというのは教えないのかね、学校の先生は
35132人目の素数さん:2007/03/21(水) 00:47:43
>>31
Ax^2+Bx+C=0 ・・・ (1)
Dx^2+Ex+G=0 ・・・ (2)

x^2を消去した方程式を(3)とすると
(1)かつ(2) <=> (1)かつ(3)  (または,(2)かつ(3))
(3)をみたす x で,(1)(または,(2))もみたすものが,(1)と(2)の共通解.
(3)単独では必要条件でしかない.

連立方程式だって,1文字消去した方程式と,もとの方程式のいずれか一方を連立して
解を得るでしょ.
36132人目の素数さん:2007/03/21(水) 00:49:33
>>19
>あと、A=a*b=c*d が成り立つときはいつも
>A=s*(t*u)=(s*t)*u とか
>みたいな感じに表すことがてきるんですか?

本質的な式は A=s*(t*u)=(s*t)*u だけ。
因数分解の途中式だと思って見れば自明に成り立つ式で、
要するにAを二つの数の積で書いたときに
2組の積のペアが存在するための条件を示している。
このようなs,t,uに対してs=a, b=t*u..と対応させれば一つ目の式になる。


>なんでt^2+1が4の倍数だといけないの

自分用のメモを消し忘れただけですw
ちなみにこれは条件ではなく定理です。


>>18の方針で続けるなら「(s^2+u^2)が4の倍数でないときに
4N=(s^2+u^2)*(t^2+1)を満たすs,t,uの組は少なくとも
N<325の範囲で一意に定まる」ことを示せれば完了です

まあ、折角だから一般解も欲しいと思って欲張って挫折したけど
上の証明だけなら数個数えるだけなので大した手間ではないです
37132人目の素数さん:2007/03/21(水) 00:49:54
>>32
変数が一つしかない場合は連立したときにxを消去しては
いけないんですか?理由を教えてほしいのですが。

>>33
AE-BDは0ではないです。
38132人目の素数さん:2007/03/21(水) 00:54:57
>>35
(3)を出しても(1)か(2)をまた計算して(3)の条件を満たさなければ
いけないのですか・・・・・。
何故必要十分条件にしかならないのを教えてほしいのですが。
39132人目の素数さん:2007/03/21(水) 00:56:16
訂正
何故必要十分条件にしかならない

何故必要十分条件にならないのか
です。
40132人目の素数さん:2007/03/21(水) 00:56:20
>>37
AE-BD≠0 なら x=(CD-AG)/(AE-BD) と求まるはず。
41132人目の素数さん:2007/03/21(水) 00:57:54
>>38
f=0 かつ g=0 ならば,af+bg=0 だが,一般に逆は成り立たないから.

>>40
だからそれは必(ry
42132人目の素数さん:2007/03/21(水) 01:01:43
>>36
6*6=4*9
43132人目の素数さん:2007/03/21(水) 01:36:49
>>42
元の問題をやると分かるがそのパターンが初めて出るのは
Nが1445のときなので考えなくても良いのだ

まあ他にもいろいろ暗黙の話があるけど面倒なので大目に見て
44132人目の素数さん:2007/03/21(水) 01:38:14
教えてください

次の不等式を解きなさい

{2х−1≦3х−1
┃−x+8〉2(х−2)


答えだけで構わないのでよろしくお願いします
ちなみに私は【х〉4】になったのですが…
45132人目の素数さん:2007/03/21(水) 01:40:50
0≦х<4
46132人目の素数さん:2007/03/21(水) 01:44:25
>>45わかりました
ありがとうございます!!
47132人目の素数さん:2007/03/21(水) 01:48:43
答えが分かったということなのか
解き方が分かったということなのか・・・
48132人目の素数さん:2007/03/21(水) 01:49:19
>>23 ありがとうございます
49132人目の素数さん:2007/03/21(水) 10:06:52
vはロケットの速度(ただし等速直線運動とする)
tは経過時間
yは宇宙の大きさが二倍になる時間
uはt年後のロケットの見掛けの速度
rはt年後のロケットの距離

v*2^(t/y)=u
(2^(t/y)*t*y)/log(2)

これでよろしいですか
5049:2007/03/21(水) 10:09:01
(2^(t/y)*t*y)/log(2)=r
でした。
51132人目の素数さん:2007/03/21(水) 10:26:29
>>49
見かけの速度の意味にもよる。
それとロケットの距離って何だよ?
52132人目の素数さん:2007/03/21(水) 10:29:55
>>48 , >>49はマルチ
53132人目の素数さん:2007/03/21(水) 16:38:20
円錐の求め方!誰か教えて下さい。<(_ _)>
54132人目の素数さん:2007/03/21(水) 16:40:17
>>53
円錐の何を求めるの?
55132人目の素数さん:2007/03/21(水) 16:44:06
>>54
円錐の全てです
56132人目の素数さん:2007/03/21(水) 16:44:58
>>54
体積に決まってます。ていうかそれくらい分かるでしょ・・。
行間も読めないんですか?
57132人目の素数さん:2007/03/21(水) 16:47:40
放物線は円錐をどのように切ると得られますか?誰か教えて下さい
58132人目の素数さん:2007/03/21(水) 16:49:05
>>54
表面積に決まってます。ていうかそれくらい分かるでしょ・・。
行間も読めないんですか?
59132人目の素数さん:2007/03/21(水) 16:51:02
>>54
中心角って何度でしたっけ?

で、側面積を求めたいのですが、子供に教えてくれと言われ分からなくて・・・(汗
60132人目の素数さん:2007/03/21(水) 16:51:24
>>57
底面上に任意の点P(ただし円周上は除く)
側面上に任意の点Q(ただし頂点、円周上は除く)をとる。
PとQを通る任意の平面による切り口は放物線になる。
61132人目の素数さん:2007/03/21(水) 16:57:30
>>59
中心角を求めたいのは分かった
既知のデータは何と何?
62132人目の素数さん:2007/03/21(水) 16:58:00
あ、中心軸に平行な平面は除いてね。
63132人目の素数さん:2007/03/21(水) 16:58:41
んで、円錐の名産地はどこですか?
64132人目の素数さん :2007/03/21(水) 17:11:08
πはオッパイと無関係なことを示せ。
65132人目の素数さん:2007/03/21(水) 17:22:27
>>64
偽。
66132人目の素数さん:2007/03/21(水) 17:24:20
無関係ではないかもしれん
67132人目の素数さん:2007/03/21(水) 17:28:00
今、携帯から見ているんですが、

名前の欄に何も表示されていない人と、
132人目の素数さんと表示されている人がいます。
この違いは何ですか?
68132人目の素数さん:2007/03/21(水) 17:30:57
>>60
双曲線になるかも
69132人目の素数さん:2007/03/21(水) 18:27:30
>>60
中心軸と交わるような平面じゃダメなんじゃないか?
70132人目の素数さん:2007/03/21(水) 19:15:52
頂点から底面を通る面できる以外は円か楕円だろ
71132人目の素数さん:2007/03/21(水) 19:17:09
母線上に頂点をもってきてもいい
72132人目の素数さん:2007/03/21(水) 19:18:55
>>71
「頂点」をどういう意味で使っているのか
73132人目の素数さん:2007/03/21(水) 20:35:10
「xy平面において、x座標、y座標ともに整数であるような点を格子点と呼ぶ。
格子点を頂点に持つ三角形ABCを考える。
(1)辺AB、ACそれぞれの上に両端を除いて奇数個の格子点があるとすると、
辺BC上にも両端を除いて奇数個の格子点があることを示せ。
(2)辺AB、AC上に両端を除いて丁度3点ずつ格子点が存在するとすると、
三角形ABCの面積は8で割り切れる整数であることを示せ。」

お願いします。
74132人目の素数さん:2007/03/21(水) 20:40:27
>>67
頭が良いか悪いかです。
75132人目の素数さん:2007/03/21(水) 21:27:59
>>73
マルチ
76132人目の素数さん:2007/03/21(水) 22:07:34
>>73は全質問スレへの糞マルチにつきβ並にスルーで
77132人目の素数さん:2007/03/21(水) 22:51:48
(4a+2b)/3+(a-3b)/6
おねがいします
78132人目の素数さん:2007/03/21(水) 22:52:44
>>77
小学生はまず国語の勉強をしましょう。
7977:2007/03/21(水) 22:59:24
80132人目の素数さん:2007/03/21(水) 23:00:26
79 名前:77[sage] 投稿日:2007/03/21(水) 22:59:24


81132人目の素数さん:2007/03/21(水) 23:02:57
ダレか助けて…!
『n3+5nは6の倍数である。証明せよ。』 タスケテケスタ! いゃぁ…未熟な身でしてw
82132人目の素数さん:2007/03/21(水) 23:03:02
>>79
「国語の勉強をしろ」という日本語も分からないのか?
83132人目の素数さん:2007/03/21(水) 23:03:52
>>81
もういいつまらん
84132人目の素数さん:2007/03/21(水) 23:04:15
>>81
マルチもしくはコピペ荒らし
85132人目の素数さん:2007/03/22(木) 00:50:07
>>79

とりあえず1/3+1/6からやりなおせと言っておく

86132人目の素数さん:2007/03/22(木) 02:32:02
「xy平面において、x座標、y座標ともに整数であるような点を格子点と呼ぶ。
格子点を頂点に持つ三角形ABCを考える。
(1)辺AB、ACそれぞれの上に両端を除いて奇数個の格子点があるとすると、
辺BC上にも両端を除いて奇数個の格子点があることを示せ。
(2)辺AB、AC上に両端を除いて丁度3点ずつ格子点が存在するとすると、
三角形ABCの面積は8で割り切れる整数であることを示せ。」

お願いします。
87132人目の素数さん:2007/03/22(木) 05:16:02
>>86
足の裏もんでくれ
88132人目の素数さん:2007/03/23(金) 03:39:20
おやすみking
89132人目の素数さん:2007/03/23(金) 12:27:12
f(x)をxの整式とする。
x^4+2x^3+{f(x)+4}x^2=∫[0,x]tf(t)dtが
xに関する恒等式になるという。
(1)f(x)の次数を求めよ。
(2)f(x)を求めよ。

f(x)が二次以上だと恒等式になって、次数が特定できないんですが…教えて下さい。
f(x)=a_nx^n+a_n-1x^n-1+…a_0とおいてやるのは馬鹿ですか?微分したりもありですか?一応数学UBのテキストで解答がないんです(>_<。)
90132人目の素数さん:2007/03/23(金) 13:04:04
>>89
そう置いても解けるから、良いと思う
当然、計算量は少し多くなる
次数だけ考えるんじゃなくて最高次係数まで考えれば次数は分かる
91132人目の素数さん:2007/03/23(金) 13:14:48
>>89
両辺を微分して
4x^3+6x^2+x^2*f'(x)+2x(f(x)+4)=x*f(x)

両辺の次数が合うにはf(x)が3次式である事が必要十分。

あとは何とかなるっしょ。
92132人目の素数さん:2007/03/23(金) 13:30:39
>>91
2次に見えるんだが
93132人目の素数さん:2007/03/23(金) 13:37:21
>>89
f(x) の最高次の項を ax^n (a≠0) とする。
右辺の最高次の項は {a/(n+2)}x^(n+2)
左辺のそれは
n>2 のとき a
係数を比較して a=a/(n+2)
a=0 となり不適
n=2 のとき (1+a)x^4
同様に 1+a=a/4
a=-4/3
n=1 のとき右辺は3次式なので恒等式にならず不適
94132人目の素数さん:2007/03/23(金) 14:00:23
なるほど(゚O゚)できました★
>>89です。
解答してくれた方々ありがとうございましたm(__)m
95132人目の素数さん:2007/03/23(金) 14:07:47
イケメンいますか?
96132人目の素数さん:2007/03/23(金) 14:27:39
>>95
お呼びですか?
97132人目の素数さん:2007/03/23(金) 14:38:54
^この記号ってどういう意味ですか?       水を差すようですみません
98132人目の素数さん:2007/03/23(金) 14:45:19
「x^2」=「xの2乗」
99132人目の素数さん:2007/03/23(金) 14:49:35
どうもありがとうございました
100132人目の素数さん:2007/03/23(金) 15:47:18
( ^ω^)
101132人目の素数さん:2007/03/25(日) 04:34:24
あなたはお菓子屋さんです。
お店に300円の商品を買いに来たお客がいました。
そのお客は1000円札をだしたけど、おつりの小銭がなかったので、
隣の薬屋さんにその1000円札を持っていって小銭と変えてもらいました。
店に戻ったあなたはその小銭のうち700円をお釣りとしてお客さんに渡しました。
しかしその後、両替してくれた薬屋さんがさっきの1000円札は偽札だったので返せと言ってきたので
あなたは仕方なしに薬屋さんに1000円札を渡しました。

問題:お菓子屋さんは何円損をしたでしょう?
102132人目の素数さん:2007/03/25(日) 04:41:11
300円

ケチクセー
103132人目の素数さん:2007/03/25(日) 04:44:42
>>102
なんで300円なんでしょうか?
104132人目の素数さん:2007/03/25(日) 04:48:20
700円の損にきまってんでそ。
105132人目の素数さん:2007/03/25(日) 04:52:22
>>103
300円のお菓子ってのがけち臭いと言っただけだ。
106132人目の素数さん:2007/03/25(日) 04:53:51
>>105
勘違いしましたw
お菓子やさんの損は700円なんでしょうか?
なんで700円になるんですか?
107104:2007/03/25(日) 04:56:52
           ・・・・・・
客から1000円受け取る→その金もって両替に行く→700円客に渡す→お菓子屋+300(商品は失う)
薬屋からクレーム1000円新たに返す→お菓子屋300円−1000円で−700円

∴お菓子屋−700円 あと商品の原価
108104:2007/03/25(日) 05:00:13
客の金を受け取って、その金の一部を客に返すから

偽札を薬屋から受けっていれば、また違ってくるがそういう風には読み取れない。
109132人目の素数さん:2007/03/25(日) 05:00:24
話を整理すると、薬屋のオヤジはぶっ殺して、偽札も消せばいいので、
犯し屋さんは、ただでお菓子と700円を詐欺師にあげたわけ。
お菓子の原価がわからんからどんだけ損したのかはさっぱりわからん。
110132人目の素数さん:2007/03/25(日) 05:04:02
それだと原価じゃなくて商品の値段は300円だから1000円の損なんじゃないでしょうか?
客は300円分の商品+700円の分だけ得しているし薬屋は損得なしなので
客の得した分だけ損してるのはお菓子やさんだと思ったんです。
111104:2007/03/25(日) 05:07:36
いや全然OK1000円でも
112104:2007/03/25(日) 05:11:24
薬屋は儲かってるけどね、偽札をお菓子屋に渡しているわけではないから、

現行法上は儲かる
113132人目の素数さん:2007/03/25(日) 05:12:56
>>112
なんでですか?
薬屋は偽札受け取って1000円渡して、偽札だと分かって1000円返してもらったから損も得もしてないんじゃ・・・
114104:2007/03/25(日) 05:16:45
だって薬屋は偽札の1000円もっているわけでしょ?

偽札を本物の札に変えてもらえばいい。だから現行法上はと書いたつもり
115132人目の素数さん:2007/03/25(日) 05:18:22
>>114
そうなんですか。よく分かりました。
116104:2007/03/25(日) 05:22:50
現代だと替えてくれるはず昔は認められてなかったけど
いつからか変わった。法律として規定されたかは知らないけど
規則かなにか変わったはず。偽札を流通させない為に
117132人目の素数さん:2007/03/25(日) 06:50:57
二乗根と二乗の違いを教えて下さい!
例えば9の二乗は81ですが二乗根はいくつですか?
118132人目の素数さん:2007/03/25(日) 07:01:54
>>117
3, -3.
119132人目の素数さん:2007/03/25(日) 09:00:07
自乗の根(=もと)なんだから自乗との違いなんて明白だと思うが。
120132人目の素数さん:2007/03/25(日) 09:04:45
>>110
得られるはずだった利潤がえられないことも損だと定義しているのか?
たとえば、販売店がお菓子を賞味期限切れで廃棄することを考えれば
同じように利潤が出ない状況ができるが、このとき販売店がどれだけの
損失を出したかといえば、原価分の損失が計上されるだろう。
121132人目の素数さん:2007/03/25(日) 10:58:12
>>117
○○の二乗=○○を二乗したもの。
○○の二乗根=二乗すると○○になるもの。
用語を聞いてどうすんだ?って気はする。調べりゃわかるだろ。
122132人目の素数さん:2007/03/25(日) 12:06:19
>>117
二乗根と二乗の違い

文字数が3と2
123132人目の素数さん:2007/03/25(日) 13:57:35
(x+8)(x+7)(x-3)(x-4)を、工夫して解くにはどうすればよいのでしょうか?
124132人目の素数さん:2007/03/25(日) 14:08:37
日本語で喋れカス
125132人目の素数さん:2007/03/25(日) 14:14:53
>(x+8)(x+7)(x-3)(x-4)を、工夫して解く

「解く」の意味は?
きみはまずこくごをべんきょうしてきなさい
126132人目の素数さん:2007/03/25(日) 14:15:46
昨日の夜、永遠と考えても全く理解できませんでした。
よろしければ解法を教えていただけますか?

高さhの円錐Fを、底面に平行な平面で切断してできる円錐をF'とする。
F'の表面積がFの表面積の半分になるようにするには、Fの底面からどの高さで切断すればよいか。
また、このときF'の体積はFの体積の何倍になるか。
127123:2007/03/25(日) 14:20:32
すみません、(x+8)(x+7)(x-3)(x-4)を、工夫して展開するにはどうすればよいのでしょうか?
、と訂正します。でもそんなに揚げ足とらなくてもw

お願いします。」
128132人目の素数さん:2007/03/25(日) 14:21:11
>>126
だから自分がどこまでやったのか書けよ低脳
129132人目の素数さん:2007/03/25(日) 14:24:01
>>127
揚げ足(ry
とか言ってるから数学の能力が伸びないんだよ

あ、あと国語の能力もか
130132人目の素数さん:2007/03/25(日) 14:27:02
>127
展開順を 真ん中の二つと両脇二つでする
そうすると 項 x~2+4x がでるから。それをAと置いて展開 
131132人目の素数さん:2007/03/25(日) 14:27:28
>>129
お前、汚れた人間だなw
132126:2007/03/25(日) 14:28:12
>>128
全くわからず、手が付けられない次第です。
自分でやれと言われたらそれまでだと思いますが、解法のコツを教えていただけませんか?
133132人目の素数さん:2007/03/25(日) 14:30:41
>>132
立式するとか図書くとか
努力してないだけだろ
134132人目の素数さん:2007/03/25(日) 14:32:08
>126
たとえば高さを半分にしたら表面積と体積はどうなる?
高さを1/4にしたらどうなる?
そのぐらい調べてみた?
135132人目の素数さん:2007/03/25(日) 14:32:51
>>130
受験が終わって浮かれてるのかもしれんが
安価の付け方や数式の書き方くらい調べてから書き込め
馬鹿丸出しだぞ
136127:2007/03/25(日) 14:34:35
すみません、項 x~2+4x とは何でしょうか?
137132人目の素数さん:2007/03/25(日) 14:35:44
>>136
マジで高1からやり直せ
138132人目の素数さん:2007/03/25(日) 14:36:15
>>131
131が汚い人間、つうか、馬鹿で自慰してるだけだと思う今日この頃。
139132人目の素数さん:2007/03/25(日) 14:36:33
>135
いちいちうるせ-よ。おまえ
x^2+4x だろ。

じゃ、S0、S1、S2、S3
さてこのなかでリー群でないのはどれ?答えてみな。
140136:2007/03/25(日) 14:37:25
>>137
まだ中2なんですよw
141132人目の素数さん:2007/03/25(日) 14:38:07
>>139
S2だろ
142132人目の素数さん:2007/03/25(日) 14:38:28
釣り堀化現象
143132人目の素数さん:2007/03/25(日) 14:38:32
念のため
リー群は 可微分多様体で群の構造をもつ
144132人目の素数さん:2007/03/25(日) 14:43:04
>>140
中1からやり直せ
145132人目の素数さん:2007/03/25(日) 14:48:11
お子様相手に 落ちこぼれが威張るスレはここですか?
146132人目の素数さん:2007/03/25(日) 14:49:37
少なくともお子様を甘やかすスレではないですね
147132人目の素数さん:2007/03/25(日) 14:58:17
射影空間上の複素直線束が巡回群になるということが分からないのですが、
どのように証明すればよいのでしょうか?(概略だけでも・・・)
148132人目の素数さん :2007/03/25(日) 15:23:33
いくぜ、昇竜裂破!!!
149132人目の素数さん:2007/03/25(日) 15:57:10
>>147
複素直線束の何が巡回群になるって?
150132人目の素数さん:2007/03/25(日) 23:56:10
(x+y+z)(x-y+z)(x+y-z)(x-y-z)
どなたかこれを展開していただけませんか?お願いします。
151132人目の素数さん:2007/03/25(日) 23:56:59
>>150
マルチ
既にレス付き
152132人目の素数さん:2007/03/26(月) 02:04:48
>>126
あなたの昨日の夜は永遠です。
あなたは今日には存在することができません。
あきらめなさい。
153132人目の素数さん:2007/03/26(月) 02:12:40
>>126
拡大縮小ってわかるかな

F' を拡大すると F になって
F を縮小すると F' になる。

ある図形を a倍すると
面積にあたる量は a^2 倍
体積にあたる量は a^3 倍
だから、F のa倍が F' になるなら表面積は a^2 倍
a^2 = 1/2 のとき a =1/√2

だから F の1/√2倍が F' だ。
h も h/√2になってる。

ただ、底面からだから h - (h/√2) = { 1-(1/√2)} h

体積は a^3 倍な
154132人目の素数さん:2007/03/26(月) 06:50:03
[問い]
xy平面状に(4,3)を重心とする凾`BCがある。相異なる3つの実数α、β、γを用いて
A(β+γ,βγ),B(γ+α,γα),C(α+β.αβ)と表すことができるとき、次の問いに答えよ.
(1)αβγのとり得る値の範囲を求めよ.
(2)凾`BCの外心が存在し得る範囲を求めよ.
(3)僊BCの外接円上の点が存在し得る範囲を求めよ.


(1) 0<αβγ<4
(2)1<x<3,かつy=5
((2)は余り自信ありません)となりましたが(3)が解けません。
(3)だけでもお願いしますm(__)m
155132人目の素数さん:2007/03/26(月) 07:08:14
サイコロを四回振って1が二回でる確率は?
156132人目の素数さん:2007/03/26(月) 07:16:57
>>155
1/36*25/36*4C2
157132人目の素数さん:2007/03/26(月) 08:15:21
>155
マルチ
158132人目の素数さん:2007/03/26(月) 12:59:06
>>154
これムズイな
159132人目の素数さん:2007/03/26(月) 13:48:44
確率は25/216
160132人目の素数さん:2007/03/26(月) 14:57:11
>>154
外心を(t,5)とすると外接円の半径が√(t^2 + 16)になったよ。
つまりこの外接円は2点(0,1),(0,9)を常に通る。
検算してないから自信ないけど。
161154:2007/03/26(月) 15:55:09
>>160
自分も計算したら、その通りになりました。
常に通る二点に着目するとは上手い方法ですねー
それで全て解決します。有難うございました!
162132人目の素数さん:2007/03/26(月) 16:48:11
n∈整数,n^2がkの倍数ならnもkの倍数であること証明したいのですが、どうすれば...
163132人目の素数さん:2007/03/26(月) 16:51:27
>>162
2^2は4の倍数だが、2は4の倍数じゃない。
164132人目の素数さん:2007/03/26(月) 17:02:42
>>163
ご指摘ありがとです。
n≧kなら成り立ちます?
165132人目の素数さん:2007/03/26(月) 17:05:26
>>164
900は20の倍数だが、30は20の倍数じゃない。
166132人目の素数さん:2007/03/26(月) 17:08:18
>>162
お前の予想なんか知るか
出典が確かな問題を持ってこい
167132人目の素数さん:2007/03/26(月) 19:00:40
反例出される前に
予想に名前つけとけ。
168132人目の素数さん:2007/03/26(月) 20:33:50
(1+(1/n))^s=1+(s/n)+Ο(1/n^2) (n→∞)(s>1,Οはランダウ記号)が成り立つのがどうしてかわからないので教えてください。
テイラー展開よりと書いてあるのですがどのように使うのかわからなくて・・・。
169132人目の素数さん:2007/03/26(月) 20:47:07
>>168
普通にテーラー展開すればあとはランダウの記号を知ってるかどうかだけじゃん。
170132人目の素数さん:2007/03/26(月) 20:51:42
>>169
普通にってどのようにすればよいのでしょうか。教えてください。
171132人目の素数さん:2007/03/26(月) 21:06:44
?
172132人目の素数さん:2007/03/26(月) 21:15:01
x軸上を移動する点pがある。pは原点から出発し、硬貨を1回投げるごとに、
表が出たら正の方向に2進み、裏が出たら負の方向に1進む。硬貨を6回投げるものとする。

(1)硬貨を6回投げたとき、点pが2度目に原点に戻る確率
(2)硬貨を6回投げたとき、点pが原点にはじめて戻る確立

宜しくお願いします
173132人目の素数さん:2007/03/26(月) 21:19:55
>>170
テーラー展開すればよい
174132人目の素数さん:2007/03/26(月) 21:23:18
>>173
s,nどちらについてテイラー展開しても右辺の式が出てこないのですが。一般二項定理を使えば出てくるのですが、テイラー展開だとわかりません。
175132人目の素数さん:2007/03/26(月) 21:38:02
(1)は3/64でいいのかな?だれかやってみてくれ。
176132人目の素数さん:2007/03/26(月) 21:42:21
(2)は3/16で合っているかな?両方とも自信はあまり無いかも。
177132人目の素数さん:2007/03/26(月) 21:57:12
>>172 (1)9/64(2)3/32
178132人目の素数さん:2007/03/26(月) 22:21:01
>>174
2項定理でできたんならそれでいいじゃん
179172:2007/03/26(月) 22:37:48
解き方教えてもらえませんか?
180132人目の素数さん:2007/03/26(月) 22:38:58
>>174
一般2項展開はテーラー展開の一種。
つか1/nに関するテーラー展開だろ。
181132人目の素数さん:2007/03/26(月) 22:40:30
>>179
誰に言っておるのじゃ
182132人目の素数さん:2007/03/26(月) 22:53:22
>>180
1/nに関するテイラー展開って具体的にどう計算すればいいのですか?
183132人目の素数さん:2007/03/26(月) 23:11:34
3人の死刑囚がいます
王様は3人の死刑囚に言いました
「ここに白い帽子3つと黒い帽子が2ある、白か黒どちらかをお前たちにかぶせ
白い帽子をかぶったものが逃げたらそのまま逃がしてやろう、ただし黒い帽子をかぶった
ものが逃げたらその場で射殺する」
死刑囚は自分がなに色の帽子をかぶっているかはわかりませんが
他の二人がかぶっている帽子の色はわかります
王様は3人とも白い帽子をかぶせました
3人はしばらく考えた後、自分が白だと確信していっせいに逃げました
さて、なぜ自分が白だと確信できたのでしょう?
※アイコンタクトとかそういう答えじゃない
184132人目の素数さん:2007/03/26(月) 23:18:51
>>182
x = 1/nで変数変換してごらん
185132人目の素数さん:2007/03/26(月) 23:23:34
>>184
x=1/nと変換すると(1+x)^sとなりますが、ここからどうすればいいのでしょうか。
186132人目の素数さん:2007/03/26(月) 23:23:49
>>183
王様万歳  。+゚(゚つω`゚)゚+。
187132人目の素数さん:2007/03/26(月) 23:24:13
>>185
xでテーラー酒井
188132人目の素数さん:2007/03/26(月) 23:26:04
>>187
xのどの点の周りでテイラー展開すればよいのですか?
189132人目の素数さん:2007/03/26(月) 23:33:41
>>188
0
190132人目の素数さん:2007/03/26(月) 23:36:15
>>189
なるほど。どうもありがとうございました。
191132人目の素数さん:2007/03/26(月) 23:48:31
高一の者ですが教えて下さい。

f(x)+x^2f'(x)=kx^3+k^2x+1を常に満たす。

f(x)をxのn次式とする時のnの値


微分の問題です。
宜しくお願いします。
192132人目の素数さん:2007/03/26(月) 23:51:23
>>183

3人ともしばらく考えたから。
193132人目の素数さん:2007/03/27(火) 00:07:37
>>191
k=0のときと、そうでないときで場合分け。
194132人目の素数さん:2007/03/27(火) 00:15:34
>>193
すみません。kは0でない定数です。

f(x)がn次式ということは
f'(x)は(n-1)次式ですよね?
そこから等式がどう変わるのかがわかりません。。
195132人目の素数さん:2007/03/27(火) 00:19:09
>>194
> f(x)がn次式ということは
> f'(x)は(n-1)次式ですよね?
x^2f'(x)は何次式?
196132人目の素数さん:2007/03/27(火) 00:23:56
>>195
n+1次式……ですか?
197132人目の素数さん:2007/03/27(火) 00:25:08
>>192
分かってない奴にそのヒントで分かれって言うのは酷だろ

>>183
「前略ならば他の死刑囚が逃げ出すはずだから後略」
198ラフィーナ ◆4uOfhyZmKc :2007/03/27(火) 00:25:41
>>183
黒い帽子が二つあるとすると、残りの一人はすぐに自分が白だとわかるはずである。
すぐに逃げ出した者はいなかったので黒は0か1※

∴黒が一つあるとすると、残りの2人は自分が白であることにすぐに気付くはずである。(∵※)

そこで逃げ出した者はいなかったので黒は0個。つまり全員白い帽子をかぶっているとわかる。
199132人目の素数さん:2007/03/27(火) 00:30:31
>>196
んじゃ、f(x)+x^2f'(x)は何次式?
200132人目の素数さん:2007/03/27(火) 00:45:55
n次式とn+1次式をたしたものはn+1次式?
わからなくなってきました。。
201132人目の素数さん:2007/03/27(火) 00:48:39
>>200
でそれが実際は3次式なのだからn=2
202β ◆aelgVCJ1hU :2007/03/27(火) 00:50:49
紙に書き直してやったほうがいいよ。ココで考えるとわかりにくい。
203132人目の素数さん:2007/03/27(火) 00:51:46
整数問題です。

@次を満たす整数a,bをすべて求めよ
 a^2+7ab+12b^2+a+3b-9=0
A次を満たす整数x,yをすべて求めよ
 4y+5x=3
B次を満たす正の整数の組(m,n)を1つ求めよ
 25m+17n=1623
C次を満たす正の整数の組(x,y,z)を全て求めよ
 xyz=x+y+z
D次を満たす整数x,y,zのうちでxが最大の組を求めよ
 0<x<y<z 1/2=1/x+1/y+1/z

よろしくお願いします。
204132人目の素数さん:2007/03/27(火) 00:52:10
>>201
ありがとうございます。
すっきりしました。
順番に考えていくのって大切ですね。

本当にありがとうございました。
205132人目の素数さん:2007/03/27(火) 00:52:53
>>203
マルチ
206132人目の素数さん:2007/03/27(火) 01:53:20
>>204
n=2のとき、本当に題意を満たすfとkが存在するかを、
一応確かめた方がよい。
207132人目の素数さん:2007/03/27(火) 02:15:36
>>167
kwsk
208132人目の素数さん:2007/03/27(火) 02:31:31
>>198
有名問題にいちいちマジレスする必要はない。
適当なサイトのリンクを貼って誘導すればよい。
209132人目の素数さん:2007/03/27(火) 02:33:17
>>208
まあ、>>198も不正確なんだがな、よく読むと
210132人目の素数さん:2007/03/27(火) 02:47:41
>>209
>>198は違うのか?
どこが不正確?
211132人目の素数さん:2007/03/27(火) 02:55:16
>>210
この問題のキモは、各人が自分の色を仮定した上で
他人の立場に立って、もう一度仮定してみるという点

背理法を意識した解答を作らないと自家撞着に陥る
212132人目の素数さん:2007/03/27(火) 03:01:13
>>211
わからん・・・
>>198も黒の数を仮定して矛盾を導いてると思うが。
論理も全員がわかる話だから自分だけにとどまってはいないのでは?
213132人目の素数さん:2007/03/27(火) 03:31:52
まあ
「黒が一つあるとすると、残りの2人は自分が白であることにすぐに気付くはずである」
この一行で逃げてちゃ単位はもらえないだろう。
214132人目の素数さん:2007/03/27(火) 04:04:36
>ここに白い帽子3つと黒い帽子が2ある

で、白い帽子が無罪なんだよな。
自分から見えている帽子は白2つ。
ということは自分は白黒分からない。

自分が黒の場合
他の奴等から見て白1黒1
ということはそいつ自身白黒分からない。

自分が白の場合も他の2人は気づかない。


だからなんかこの問題帽子の数がおかしくないか?
215132人目の素数さん:2007/03/27(火) 04:42:14
だからなんかこの問題帽子の数がおかしくないか?  ゴメンワロタ!
216132人目の素数さん:2007/03/27(火) 05:46:37
>>214
「他の奴等から見て」つか。
「他の奴」の立場も含めて推論してみな。

自分(Aとする)が黒だったと仮定して推論開始。
白をかぶっている他人をそれぞれB、Cとしておいて
Bの立場に立って推論をすると。

Bの目からは白1(C:確定)黒1(A:仮定)が見えている
→もしBが黒なら、黒二つ見えているCは逃げるはず→しかし逃げない
→俺(B)は白→すぐに逃げるはず→しかしBは逃げない。なぜか?
→Aが黒という仮定が誤り→各人について同様の推論成立
→「しばらく考えた後」自分が白だと確信

まあ、こういう奴が出てくるということは
>>209の指摘が正鵠を射ていた、ということか。
217132人目の素数さん:2007/03/27(火) 06:02:21
>>216 間違いです。
218216:2007/03/27(火) 06:06:32
自分は>>209ではないです。

何で仮定する必要があるの?

この問題は答えがでないよ。
219132人目の素数さん:2007/03/27(火) 07:06:59
>>218
騙り乙
220132人目の素数さん:2007/03/27(火) 07:11:45
有名問題だ、と何度同じことを言えば(ry

http://www.geocities.co.jp/Berkeley-Labo/6317/ronnri_06.htm
221ラフィーナ ◆4uOfhyZmKc :2007/03/27(火) 07:13:17
>>213
※から黒は二つないので。( ´,_ゝ`)
多少日本語がおかしくなることを覚悟で∴と∵両方使ってまで強調したつもりですが。
222132人目の素数さん:2007/03/27(火) 07:19:41
>>219


>>218の名前欄に216って書いてしまった。217の間違い

いや煽りじゃない。数学としては答えがでない。

223132人目の素数さん:2007/03/27(火) 07:25:03
>>221
とりあえず、お前はマイクロソフトには就職できないな。
224132人目の素数さん:2007/03/27(火) 07:25:58
>>222
ゲーム理論を知らない奴にはそう見えるかもな
225222:2007/03/27(火) 07:33:20
>>224説明してくれ

自分(Aとする)が黒だったと仮定して推論開始。

→もしBが黒なら、黒二つ見えているCは逃げるはず→しかし逃げない



なんで自分が黒だったと仮定した後に、もしBが黒ならってなるんだよ、
Bからすれば他の二人は白だろ

226216:2007/03/27(火) 07:34:42
>>220
あー、正直スマンかった。
あまりにバカが多すぎるんで少し意地になってたかも。

仕事もあるんでもう落ちるわ。
227132人目の素数さん:2007/03/27(火) 07:37:25
>>225
落ちると書いた直後でアレだが。

それは、Bの立場に立った推論だ。
「もしBが黒なら」を「もし自分(B)が黒なら」
と読み替えたら理解できるか?

それでもわからんなら俺はもう匙を投げる。
228132人目の素数さん:2007/03/27(火) 07:41:15
>>222はすげーバカだな。
229225:2007/03/27(火) 07:42:13
>>227

いや悪いけど理解できないね。

なぜ黒い帽子を二人ががぶっていた場合を仮定できる。

230132人目の素数さん:2007/03/27(火) 07:44:05
>>227
モチツケ

×:Bの立場に立った推論だ
○:Bの立場を考慮したAによる推論だ

このくらい書いとかなきゃ
きっと理解してもらえないぞ
231225:2007/03/27(火) 07:44:37
自分以外が白い帽子をかぶっているのは明らかだろ>>227
232ラフィーナ ◆4uOfhyZmKc :2007/03/27(火) 07:44:44
>>223
お医者さんになるからそんなとこ行く必要ないし(*゚艸゚)
`;:゙;`;・(゚ε゚ )ブッ!!

どう間違ってるのか説明して下さい。
模範解答と違うから間違いだなんてナシですヾ(ゝ∨б*)
233132人目の素数さん:2007/03/27(火) 07:47:21
>>231
Bにとって、Bが白をかぶっている事実は明らかではない、と
Aが考えなきゃいかんのだよ、コモンナレッジの世界では
234132人目の素数さん:2007/03/27(火) 07:48:45
>>229
Aが自分が黒だと仮定すると、Bからは白と黒が見えていることになる。
そのとき、Bが自分が黒を被ってたらと考えるとCは逃げるはずなので
Bは白だと確信しているはずである。
しかし実際そうならないので、Aが黒を被ってるってことは無くなる。
条件の対称性からみんな自分が黒でないと確信することになる。

まあ、こういう平衡状態を得るためには、十分多くの時間を費やしていることが条件だがな。
235132人目の素数さん:2007/03/27(火) 07:50:29
>>231
自分が知っている情報は、他人が知っているとは限らないというごく当たり前の話。
236225:2007/03/27(火) 07:50:35
囚人が勘違いして走り出したとかならまだわかるがさぁ。そうじゃないだろ>>227

Bの目からは白1(C:確定)黒1(A:仮定)が見えている 。   こうAが推論すること自体おかしい

なぜならBから見て他の二人は白い帽子なのは明らか
237132人目の素数さん:2007/03/27(火) 07:51:26
>>234
設問中に「しばらく考えた後」と但し書きがあることから
その平衡に達した、と解釈するよう示唆されてるんだろうな
238132人目の素数さん:2007/03/27(火) 07:53:32
>>236
どこまでバカなんだか。

>自分(Aとする)が黒だったと仮定して推論開始
>>216氏はこう書いてるだろ。

お前、高校生ならもっと適切なスレに行けよ。
239132人目の素数さん:2007/03/27(火) 07:54:30
>>236
他人の知っている情報は自分にはわからん。
240132人目の素数さん:2007/03/27(火) 07:58:03
>>236
他人が実際に知っている情報と
自分がその他人が知っているだろうと推論できる情報とは
まったく違うことに注意しなければならない。

そのうえで、各人は合理的かつ自分に有利になるように行動する。
241132人目の素数さん:2007/03/27(火) 08:00:00
>>236
んな外から3人見てるやつの思考したら、推論なしに全員無罪放免だろwww
242132人目の素数さん:2007/03/27(火) 08:01:24
>>232
既に誰かが指摘してたと思ったが。

この問題のポイントは
限定して提示された事実を出発点にして
意思の疎通が不可能なメンバーが
互いに合理的な判断をした結果
正解に到達したことを論理的に示す点。

そこを省略して「わかるはず」とか
「すぐに気付くはず」とか逃げてちゃダメだ、と。
243236:2007/03/27(火) 08:03:17
>>238
あのなぁ>>216の考えなんてはなから理解してるんだよ。その上で言っている。

>自分(Aとする)が黒だったと仮定して推論開始 これは分かる

でもBから見て他の二人は白だろ絶対に
244236:2007/03/27(火) 08:04:35
>>220 うんなこと分かっているの。その上で言っている。
245132人目の素数さん:2007/03/27(火) 08:05:10
>>236
Bは自分が白か黒かすぐには判断できる状態に無いという事実と
Aが自分が黒であると仮定して得られる、Bは白だと確信できるという結論とが
齟齬を来すというのがここでの肝。
246132人目の素数さん:2007/03/27(火) 08:05:58
>>243-244
ウンコ垂れ流して死ね。
247132人目の素数さん:2007/03/27(火) 08:06:18
>>243
Aが考えてるんだよ。

自分が黒だったらBからは白黒が見えているはず
…(中略)…それじゃ矛盾だから、Bも白白を見ている→逃げろー
248132人目の素数さん:2007/03/27(火) 08:06:56
>>236はKingの脳の中を覗き見ることができるエスパー
249132人目の素数さん:2007/03/27(火) 08:08:02
>>243
Bが実際に何を考えているかというのは、いまAにとっては関係ない。
250132人目の素数さん:2007/03/27(火) 08:08:20
>>248
こら。召喚するな。
251132人目の素数さん:2007/03/27(火) 08:09:32
>>248
ちがうぞ、B=Kingの脳を悪用するやつはAだ。
252236:2007/03/27(火) 08:11:33
>>243
>Aが考えてるんだよ。
>自分が黒だったらBからは白黒が見えているはず

言っている事は分かりますよ、仮にBがAの白黒が見えていたからといって逃げれる事には繋がらない
253132人目の素数さん:2007/03/27(火) 08:11:35
結局、>>243>>216の考えなんて理解してなかった、でいいのか?
254132人目の素数さん:2007/03/27(火) 08:12:09
>>247
>Aが考えてるんだよ。
>自分が黒だったらBからは白黒が見えているはず

言っている事は分かりますよ、仮にBがAの白黒が見えていたからといって逃げれる事には繋がらない
255132人目の素数さん:2007/03/27(火) 08:12:22
>>236は他人の気持ちがわからないいじめっ子
>>236はいじめを黙認する傍観者
>>236はバカすぎて自分がいじめられっこだと誤解する被害妄想狂
256ラフィーナ ◆4uOfhyZmKc :2007/03/27(火) 08:13:06
>>242
つまり合理的に判断しなかった場合を考えろってことッスか??(゚д゚ノ)ノ
そんなの無理でしょ!
何言ったって他の2人はバカだからここまで考えが及ばないかもしれないで答出ないじゃん!!!!!ジャンジャン♪
257132人目の素数さん:2007/03/27(火) 08:13:29
わかった、>>252はあまりにも日本語が不自由すぎてまともな推論ができないんだな。
258132人目の素数さん:2007/03/27(火) 08:13:57
なんかこの流れ。
初めてべーたが出現した当時を思い出させるな。
259132人目の素数さん:2007/03/27(火) 08:14:51
>>256
わかったから、お前は高校レベルの質問にだけ答えてりゃいい。
260132人目の素数さん:2007/03/27(火) 08:15:10
>>239>>240はスルーしているところを見ると、釣りのつもりなんだろうけど
ただ余りにも頭の悪い>>236が居るという状況しか産んでないよwww
261132人目の素数さん:2007/03/27(火) 08:18:34
>>252
Aは「BがAとCを見て何を考えるだろうか」ということを
自分の被っている色がどっちかわからないという前提の下で
類推しなければならん。
そんで、Bの行動もCの行動もAの推論の材料になる。
262ラフィーナ ◆4uOfhyZmKc :2007/03/27(火) 08:19:17
>>259
そーですねΣ(ノ∀`*)
263132人目の素数さん:2007/03/27(火) 08:19:24
専ブラで辿ると、この少年の初出は>>217-218のようだな。
引っ込みがつかなくなってるのか、ファビョってわけわかんなくなってるのか。
264236:2007/03/27(火) 08:23:34
>Bの目からは白1(C:確定)黒1(A:仮定)が見えている 。   こうAが推論すること自体おかしい

>なぜならBから見て他の二人は白い帽子なのは明らか

俺の書いた文これじゃ誤解されちゃいますね謝ります。 
265132人目の素数さん:2007/03/27(火) 08:28:01
>>264
誤解つか。
お前が理解してない、という点は明白だがな。

まあ、その他の書き込みも含めて判断するに
この問題は、お前には早すぎる、ということだ。
266236:2007/03/27(火) 08:39:23
>>265含め他多数の方ゴメンなさい、百回謝ります。理解できました。

Aが自分を黒だと仮定する

AはBがAを黒だと考えたと仮定する。

Bは自分の帽子は分からないが黒だと仮定する

BはCが逃げないので、白ということが分かる



267132人目の素数さん:2007/03/27(火) 08:45:31
> AはBがAを黒だと考えたと仮定する。

まだ分かってないなお前ww
268132人目の素数さん:2007/03/27(火) 08:46:20
>>266
どうも、なんかまだ理解できてないような気もするが。

納得したんなら、まあいいや。べーたよりはマシだな。
269132人目の素数さん:2007/03/27(火) 08:47:11
おいおい、まだまだ釣り続けるつもりかwwwwwwwwwwwwww
270236:2007/03/27(火) 08:51:35
あっまだ理解してないですか?

自分なりにもう一回じっくり考えてみます。

すいませんでした。親切に付き合っていただいてありがとうございました。
271132人目の素数さん:2007/03/27(火) 08:53:01
Aが自分を黒だと仮定する =AはBとCがAを黒だと考えると仮定する。
272132人目の素数さん:2007/03/27(火) 09:13:05
ラフィーナにも言えることだが。

*ゲーム理論は大人になってから
273132人目の素数さん:2007/03/27(火) 09:48:12
親切な出題者は
A 「わかりません」
B(少し考えて) 「わかりません」
C(だいぶ考えて) 「わかりました」
って感じに出題してくれるんだけどね

>>272
ハイパーゲームって割とマイナーな気がする
274132人目の素数さん:2007/03/27(火) 12:29:47
釣りをするなら一ヶ月くらい続けてくれ
275132人目の素数さん:2007/03/28(水) 03:13:04
何となく「釣り馬鹿日誌」
276132人目の素数さん:2007/03/28(水) 12:20:08
>多少日本語がおかしくなることを覚悟で∴と∵両方使ってまで強調したつもりですが。

何様ワロタ
277132人目の素数さん:2007/03/28(水) 12:20:38
あ、お医者様か。サーセンwww
278132人目の素数さん:2007/03/28(水) 15:52:43
( ∵)ノ
      ハーイ
279132人目の素数さん:2007/03/28(水) 16:16:17
2
1.29370A +32.9516631A + 2.6022751A*logA - 1221610000

よろしくお願いします
280279:2007/03/28(水) 16:17:21
正しい位置に出なかった、すいません

1.29370A*A +32.9516631A + 2.6022751A*logA - 1221610000

281132人目の素数さん:2007/03/28(水) 16:20:09
Aって何なのよ。その式をどうしろっての?
282279:2007/03/28(水) 16:27:00
度々すいません。
>>279の数式上にある「A」を求めたいのですが、どうでしょうか?
283132人目の素数さん:2007/03/28(水) 16:30:59
>>279
 Aは求まらない。
 「x^2-3x-4」からxを求めろっていわれても困るだろ?
284279:2007/03/28(水) 16:33:43
何度も本当にすいません。
最後=0です。
285132人目の素数さん:2007/03/28(水) 16:50:43
>>284
多項式と対数が混じった一般の方程式は、
近似的に数値で求める以外に現実的な解法はない。

それの場合、30706よりちょっと小さいところに解がある、
くらいのことしかわからん。
286132人目の素数さん:2007/03/28(水) 16:57:51
>>284
対数は常用対数? 最後の定数あってる?
287279:2007/03/28(水) 17:51:57
対数は常用対数です。最後の定数は「12211610000」です。
本当にすいません。
288132人目の素数さん:2007/03/28(水) 18:55:47
>279
それで計算機にグラフ書かすと90000〜100000ぐらいに解がある
みたいだな。>285がいうように、ぴったりと求まるわけじゃないか
ら、計算機使うしかないんじゃないかな。
289132人目の素数さん:2007/03/28(水) 21:56:20
mapleで求めてみると
1.29370*A^2 +32.9516631*A + 2.6022751*A*log10(A) - 1221610000=0

A = 30711.83223
という解が出た
290132人目の素数さん:2007/03/29(木) 02:43:04
291132人目の素数さん:2007/03/29(木) 03:07:23
奥が深すぎるなこの問題
292132人目の素数さん:2007/03/29(木) 03:08:08
mapleで求めてみると
1.29370*A^2 +32.9516631*A + 2.6022751*A*log10(A) - 12211610000=0

A = 97138.26078
という解が出た


293132人目の素数さん:2007/03/29(木) 05:53:00
16人を4人ずつ、4チームに分けて、チーム対抗戦をするとします。
その上で、他の3チームのうちの一人ずつとあつまり4人で競技をし、
[同じチームの人と戦わない]・[一度当たった事のある人とは戦わない]
という条件で試合を組むとすると、4回戦まで試合をする事は可能でしょうか?
294132人目の素数さん:2007/03/29(木) 08:19:47
>>293
クソマルチ野郎が。氏ね
295279:2007/03/29(木) 08:55:26
おはようございます。亀レスすいません。
>>289 ありがとうございます。この方法で正解とのことでした。
296132人目の素数さん:2007/03/29(木) 13:11:41
この方法って maple…
297132人目の素数さん:2007/03/31(土) 20:30:48
お願いします
(1)
自然数nを面積とする3辺有理数の直角三角形が存在するときnを合同数という
どのような自然数nが合同数となるか?

(2)nをn=8k-3、8k-2、8k-1(kは自然数)を満たす平方因子を持たない数とする。
nを面積とする3辺が有理数の直角三角形が必ず存在する?
298132人目の素数さん:2007/03/31(土) 20:37:13
未解決問題ですがな。
299132人目の素数さん:2007/04/01(日) 17:27:28
質問です、よろしくお願い致します。来期から高3になる者です。

xy平面上の2円
P: (x+a)^2+(y-b)^2=k^2
Q: (x-a)^2+(y-b)^2=k^2 (つまり円Qはy軸について円Pと線対称)
について、円Pの円周上の点をs、円Qの円周上の点をtとおく。
線分stが常に定数nをとるように、s,tがそれぞれ円周上を動くとする。
このとき線分stの中点Mの描く軌跡の方程式を求めよ。

以下、私なりに試行したことです。無視して頂いても構いません。
まず、中点Mの軌跡の方程式を求めようとしたものの、
「定点と円周上の点」といったものならば解けるのですが、
「円周上の点と円周上の点」ではどのように解けばいいのか分かりませんでした。
そこで「Function View Ver 5.55a」を用いて軌跡を描いてみました。
すると中点Mの描く軌跡はレムニスケートのようなものになったのですが、
「線分stを1:2に内分する点」や「k,nの値」など、条件をかえてみると、
曲線がよく分からない形になってしまいました。
「線分stが常に定数nをとるように」という条件がなければ、
つまりどちらも独立して自由に動く場合は、
ドーナツ状の軌跡を描くことは調べて分かったのですが、
この条件を加えたときについては結局分かりませんでした。
長くなりましたが、どうかよろしくお願い致します。
300132人目の素数さん:2007/04/01(日) 17:30:14
>>299
未解決問題ですがな。
301299:2007/04/01(日) 17:48:40
>>300
そうなのですか。
もし名称があるなら、よろしければ教えて頂けませんか。
302132人目の素数さん:2007/04/01(日) 18:02:49
・次の条件を満たす2つの多項式を求める問題なのですが…
1) 最大公約数が x−1,最小公倍数がx3−2x2−5x+6
2) 積が(x−2)(2x−1)二乗(2x+3),最小公倍数が(x−2)(2x−1)(2x+3)

お願いします。 答えまでの過程もお願いします。
(1)も(2)も次数が等しいとは限らないそうなのです。
くだらない問題でごめんなさい。

303132人目の素数さん:2007/04/01(日) 19:16:11
>>302 (1)
書き方くらい調べてから来いよ
x^3-2x^2-5x+6=(x-1)(x+2)(x-3)
ここまできたらわかるだろ
304132人目の素数さん:2007/04/01(日) 19:18:17
>>302
(1)とりあえず、因数分解。
(2)って、答えが複数ないか?
305132人目の素数さん:2007/04/01(日) 19:58:08
>>303-304
わけわかんねえよwwww
306132人目の素数さん:2007/04/01(日) 19:59:34
a,b,k,n
全て未知数? 未知数が多すぎて解けん。

例えば 「ax^2+bxy+cy^2=0 はどんなグラフですか?」
と聞かれて、完璧に解答できるヤツは少ない(特に2chではw)
307132人目の素数さん:2007/04/01(日) 20:01:36
Aがコイン2枚、Bがコイン3枚を同時に投げたとき、Aの表の枚数がBの表の枚数より多い確率を求めよ
308132人目の素数さん:2007/04/01(日) 20:16:35
数学苦手で助けてほしいです
ウマシカ過ぎですいません
塩水算で分母にxがついてる場合はどうやって計算すればいいんでしたっけ?

309132人目の素数さん:2007/04/01(日) 20:27:29
>>308
塩水を飲みながら考えましょう。
310132人目の素数さん:2007/04/01(日) 20:28:25
>>307
Aの方が多いって事は
表の出方はA2枚 B1枚しかねぇじゃん!!!
311132人目の素数さん:2007/04/01(日) 20:29:05
312132人目の素数さん:2007/04/01(日) 20:29:29
>>307
Aが2枚のとき Bは1枚か0枚
Aが1枚のとき Bは0枚

この確率を全部足す。
313132人目の素数さん:2007/04/01(日) 20:56:32
わけわかりません。高一の問題。教えてください。

(a+[b-1]/2)^2-(b-1)^2/4+b^2-b+1/3=0

([]は、2分のb-1という事で)

上の式をbについて平方完成すると

(a+[b-1]/2)^2+3/4(b-1/3)^2=0

になるらしいんだが、どうしてこうなるのか理解出来ない。
314132人目の素数さん:2007/04/01(日) 21:00:25
asso
315299:2007/04/01(日) 21:01:28
>>306
レスが遅くなりました、すみません。
a,b,k,nは全実数をとるとして一般化させたかったので、複雑になってしまいました。
a=3,b=3,k=2,n=7とおいて、

xy平面上の2円
P: (x+3)^2+(y-3)^2=4
Q: (x-3)^2+(y-3)^2=4
について、円Pの円周上の点をs、円Qの円周上の点をtとおく。
線分stの長さが常に7となるように、s,tがそれぞれ円周上を動くとする。
このとき線分stの中点Mの描く軌跡の方程式を求めよ。

としてお願いします。恥ずかしながら方針すらたてられないので…。
316132人目の素数さん:2007/04/01(日) 21:12:57
>>313
まず、(a+[b-1]/2)^2 を普通に展開してみましょう。
317316:2007/04/01(日) 21:16:17
>>313
間違えた。([b-1]^2/4)を展開してみましょう。んで、b^2-b+1/3と通分してみましょう
318132人目の素数さん:2007/04/01(日) 21:46:20
>>317ありがとうございます!

通分してみたのですが、どうもそこから平方完成する事ができません。通分した結果を教えてくれませんか?
319132人目の素数さん:2007/04/01(日) 21:52:23
>>318
一般形の二次式の平方完成の式書いてみろ
320316:2007/04/01(日) 21:56:41
>>318
(a+[b-1]/2)^2はそのままね。んで、残りの式は、通分してやると

(9b^2-6b+1)/12 となった。
321132人目の素数さん:2007/04/01(日) 22:01:27
次の問題が分かりません。
n(n+1)(n+2)が6の倍数であることを示せ。

n(n+1)(n+2)=6kとおくんですかね?

そして解の分離でしょうか?本当に分かりません。
322132人目の素数さん:2007/04/01(日) 22:03:10
>>321
場合わけ
323132人目の素数さん:2007/04/01(日) 22:07:13
>>321
どう場合分けするのでしょうか?
324132人目の素数さん:2007/04/01(日) 22:07:26
>>321
2の倍数かつ3の倍数
325132人目の素数さん:2007/04/01(日) 22:09:36
>>323
nを
326132人目の素数さん:2007/04/01(日) 22:13:11
>>321
連続する3整数だろ。何が場合わけだ。
327質問者:2007/04/01(日) 22:14:12
結局、解の分離でしょうか?
328132人目の素数さん:2007/04/01(日) 22:15:14
>>327
は?
何言ってんのこいつ
329132人目の素数さん:2007/04/01(日) 22:15:37
解の分解なんでしょ!
330132人目の素数さん:2007/04/01(日) 22:16:05
>>329
じゃあいいよそれで
331132人目の素数さん:2007/04/01(日) 22:18:14
>>319>>320
わかりました!解けました!

本当はできるのに、数字を見てこれは無理だろと諦めていたようです。ありがとうございます!!
332質問者:2007/04/01(日) 22:21:09
僕も解けないけど、結局、このスレの質問者はこの問題解けないのか。
回答者はみんな尾道大学いかかよ。。。
333132人目の素数さん:2007/04/01(日) 22:22:45
>>332
じゃあいいよそれで
334132人目の素数さん:2007/04/01(日) 22:26:41
では、解の分離に決定されました。
335質問者:2007/04/01(日) 22:28:14
>>334
ありがとうございました。
336132人目の素数さん:2007/04/01(日) 22:29:31
で、何が解けないの?
337132人目の素数さん:2007/04/01(日) 22:34:31
>>336
321 :132人目の素数さん :2007/04/01(日) 22:01:27
次の問題が分かりません。
n(n+1)(n+2)が6の倍数であることを示せ。

n(n+1)(n+2)=6kとおくんですかね?

そして解の分離でしょうか?本当に分かりません。
338132人目の素数さん:2007/04/01(日) 22:58:58
>>332
かわいそうだからマジレスしとく。

このスレに書き込んでる連中の内
9割以上の人間は解ける。

ただ、あまりに基礎的な問題なので
釣りの可能性等を考慮して
真剣に対応する気になれないのだ。

まあ、中学生スレでも行ってなさいってことだな。
339132人目の素数さん:2007/04/01(日) 23:18:53
>>338
釣りがこわいなら回答者やめろ。
340132人目の素数さん:2007/04/01(日) 23:23:14
>>339
釣り前提で質問スレが成立すると思ってる春厨は度し難い
341132人目の素数さん:2007/04/01(日) 23:31:12
ちょっと質問です。
とあるネットゲームにておけるネトゲ板で起きた問題なのですが、
数学的見地から以下のような表現をするのでしょうか?

「○○のとき、△△%の確率で、□□〜■■%の範囲に収まっている」

もし、上記のような説明が成り立つのであれば、それはどのような状況下であるのかお教えいただきたく願います。

その当該ネットゲームにて、ある技能を一定のレベルまで上昇させ、
その技能の特徴といえる行動に対しての統計らしいのですが、
いまいち理解できない状態ですので、詳しい方がおられましたら説明いただけたら幸いです。

参考スレッド
http://game11.2ch.net/test/read.cgi/netgame/1072316139/793-
342132人目の素数さん:2007/04/01(日) 23:37:38
>>341
>「○○のとき、△△%の確率で、□□〜■■%の範囲に収まっている」
統計学でやりそうなことだ。厳密にはちょっと違うけど。

http://ja.wikipedia.org/wiki/%E4%BF%A1%E9%A0%BC%E5%8C%BA%E9%96%93
343132人目の素数さん:2007/04/01(日) 23:40:15
数さん :2007/04/01(日) 23:23:14
>>339
釣り前提で質問スレが成立すると思ってる春厨は度し難い


341 :132人目の素数さん :2007/04/01(日) 23:31:12
ちょっと質問です。
とあるネットゲームにておけるネトゲ板で起きた問題なのですが、
数学的見地から以下のような表現をするのでしょうか?

「○○のとき、△△%の確率で、□□〜■■%の範囲に収まっている」

もし、上記のような説明が成り立つのであれば、それはどのような状況下であるのかお教えいただきたく願います。

その当該ネットゲームにて、ある技能を一定のレベルまで上昇させ、
その技能の特徴といえる行動に対しての統計らしいのですが、
いまいち理解できない状態ですので、詳しい方がおられましたら説明いただけたら幸いです。

参考スレッド
http://game11.2ch.net/test/read.cgi/netgame/1072316139/793-


342 :132人目の素数さん :2007/04/01(日) 23:37:38
>>341
>「○○のとき、△△%の確率で、□□〜■■%の範囲に収まっている」
統計学でやりそうなことだ。厳密にはちょっと違うけど。
344132人目の素数さん:2007/04/01(日) 23:42:00
2ちゃんねる ■掲示板に戻る■ 全部 1- 最新50 [PR]体験→ライブチャットBBchatTV[PR]

--------------------------------------------------------------------------------
っっっっっっっっっっっっっh
分からない問題はここに書いてね274
342 :132人目の素数さん :2007/04/01(日) 23:37:38
>>341
>「○○のとき、△△%の確率で、□□〜■■%の範囲に収まっている」
統計学でやりそうなことだ。厳密にはちょっと違うけど。

http://ja.wikipedia.org/wiki/%E4%BF%A1%E9%A0%BC%E5%8C%BA%E9%96%93


343 :132人目の素数さん :2007/04/01(日) 23:40:15
数さん :2007/04/01(日) 23:23:14
>>339
釣り前提で質問スレが成立すると思ってる春厨は度し難い

っっっっっっっっっっっっっっっf.cgi/aastory/1171701108/
【投票所】         http://www.finito-jp.net/vote1-1/comvote.cgi?id=aaa
               http://www.finito-jp.net/vote1-1/comvote.cgi?id=lily
               http://www.finito-jp.net/vote1-1/comvote.cgi?id=mary
【アトリエ絵チャット】   http://www.takamin.com/oekakichat/user/oekakichat3.php?userid=230033
【アトリエ絵チャット2号】.http://takenoko01.nm.land.to/atorie2/atorie2.html
【掲示板(避難所兼)】  http://jbbs.livedoor.jp/study/7287/

お約束・依頼テンプレ・レシピテンプレなどは>>2-10あたり
345132人目の素数さん:2007/04/02(月) 11:49:00
>>321
日付が変わったのでマジレス
>>324を読んで考えるか、組み合わせと思って納得するか、好きな方で
346132人目の素数さん:2007/04/02(月) 15:23:21
組み合わせの解答は本当にいいのだろうか?
組み合わせの式は割り算をしているのになぜ必ず整数になるのだろうかと不思議に思い、考えた結果、
連続する整数の積だから必ず分母の倍数になるんだという結論に達した高一の頃の俺がいる

要するにこの結論を得て初めて組み合わせの式は一般に整数になるんだということが示されると思うのである
組み合わせの数は必ず整数だから・・・?
う〜〜〜〜〜ん
347132人目の素数さん:2007/04/02(月) 15:25:07
日本語でおk
348132人目の素数さん:2007/04/02(月) 16:38:11
>>346
> 連続する整数の積だから必ず分母の倍数になる
怪しいな。証明になってないんじゃない?
nCrを
nCr=n!/(r!(n-r)!)
で定義したとき、これらが整数になることは
例えば、パスカルの三角形の関係を用いて帰納的に行う
などの方法で容易に証明できる
349132人目の素数さん:2007/04/02(月) 17:02:22
>>348
いや連続する整数の積ってのはまぁ>>324の考え方さ
みんながヒント出して質問者に考えさせてるのに答みたいなこと言っちゃうと悪いじゃん?

だがなるほど。
コンビネーションが整数であることは他に証明方法があるからよいわけだ
自分の考えの浅はかさを痛感した
ありがとう

そしてさようなら
350348:2007/04/02(月) 17:29:38
んー・・・日本語でおk
「連続するn個の整数の積はn!で割り切れる」ことを
ちゃんと証明できていさえすれば、いいんだけど。
351132人目の素数さん:2007/04/02(月) 18:38:03
>>349
とりあえず>>346>>349は何が言いたいのか分からん。
すくなくとも、>>324宛てであると推察するのはエスパーでも
不可能だと思うぞ。まともな日本語で書き直してくれ。

# >>324は和分法・差分法でしめせるんじゃねーか?
352132人目の素数さん:2007/04/02(月) 19:14:06
>>346
比較的直感的な証明。
1〜kの自然数の中には2の倍数がk/2個(端数切り捨て)あり、
n-k+1〜nの自然数の中にも2の倍数がk/2個以上(端数切り捨て)ある。

更に1〜kの自然数の中には4の倍数がk/4個(端数切り捨て)あり、
n-k+1〜nの自然数の中にも4の倍数がk/4個以上(端数切り捨て)ある。
2^pの倍数について以下同様のことが言える。
と言うことはn-k+1〜nの積を素因数分解した時の2の指数は
1〜kの積を素因数分解した時の2の指数以上。

3以降の他の素数についても同様。
と言うことでn-k+1〜nの積は1〜kの積の倍数になる。
353132人目の素数さん:2007/04/02(月) 19:18:46
kore ha hidoi !!!
354132人目の素数さん:2007/04/02(月) 23:43:21
>>352
直感的っちうか、>>324の方針ではその証明以外見た事無い
一番標準的じゃないか?
>>351の言う通り、差分で出来る気はする)
355132人目の素数さん:2007/04/04(水) 20:51:32
>348,350
 任意の素数p、自然数mをとる。
 1〜k の中に p^m の倍数は [k/(p^m)] 個だが,
 n-k+1〜n の中には それ以上ある。(終)
356132人目の素数さん:2007/04/04(水) 22:32:26
最近になってワイルズによるフェルマー(最終)予想の証明に誤りが発見されたというのは本当なのでしょうか?
また、ワイルズによる証明を理解できるのは、日本・世界にはどのくらい居ると言われているのでしょうか?
ご存じの方はぜひにお教え下さい。
357132人目の素数さん:2007/04/04(水) 23:19:58
突然書き込みすみません。
教えてください。
二次関数のことについてなのですが、定義域の変化する(例 a<x<a+2)のような問題は
どう考えればよいのでしょうか?
358132人目の素数さん:2007/04/04(水) 23:24:10
頂点の位置などで場合わけ
359132人目の素数さん:2007/04/04(水) 23:25:21
>>357
どう考えるって、何を?
値域?
360132人目の素数さん:2007/04/04(水) 23:26:25
レスありがとうございます。
どのように場合わけすればいいのかわからないのです。
すいません…
361132人目の素数さん:2007/04/04(水) 23:28:08
a_{n+2} = 2 * a_{n+1} * a_n + a_{n+1}^2
a_0 = 1
a_1 = 3

a_nを綺麗な式で表せますか?
362132人目の素数さん:2007/04/04(水) 23:29:00
3行1列の行列を3行3列の行列にする線形写像なんてあるんですか?
363菅_理人@kmath1107BBS ◆.ffFfff1uU :2007/04/04(水) 23:30:00
>>362
右から3行3列の行列をかければいいとおもう
364132人目の素数さん:2007/04/04(水) 23:34:29
>>361

ちなみに、これはあるデータ構造に関係する漸化式です。
365132人目の素数さん:2007/04/04(水) 23:50:01
y=x^2-2ax+a^2+4a-99
aは6より大きい実数

(1)@頂点をaで表せ
(2)20-a≦x≦12aにおける@の最小値が237であるとき
aの値とそのときのxの値


(1)はわかったのですが、(2)がわかりません。
よかったらどうかよろしくお願いします。
366132人目の素数さん:2007/04/04(水) 23:50:31
>>363
それはそうですが・・・
fを線形写像、Bを表現行列とすると
f(A)=BAになるのでは?
右から掛ける線形写像もあるんでしょうか
367132人目の素数さん:2007/04/05(木) 00:38:59
問題文『ある地域では、病気Xに感染している人の割合は0.01%であるという。この病気Xについては定期検査があり、Xに感染している人は99%の確率で「陽性」と判定される。ただし、この検査では感染していない人も1%の確率で「陽性」判定されてしまう』

設問
(1)ある人がこの検査を受けたところ、「陽性」と判定された。この人が本当に病気Xに感染している確率を求めよ。
(2)この人が再検査をうけ、再び「陽性」判定された。この人が本当に病気Xに感染している確率を求めよ。


僕の考えでは、
(1)「陽性」と判定されたんだから99%は感染してるってことでしょ?
と考えたのですが、簡単すぎますよね?
留意点にこれみよがしに条件付き確率に注意せよなんて書いてあるし・・・。

(2)2回検査受けて「陽性」と判定されたにも関わらす、実は感染していなかったという確率は
1/100 × 1/100 = 1/10000ですよね?
ということは9999/10000の確率で感染してると思うのですが、絶対こんなに簡単なわけがないですよね?
おそらく最初の文の「ある地域では、病気Xに感染している人の割合は0.01%であるという」が関連すると思うのですが、うまく結びつけられません・・・。

感染の割合が0.01%(100人に1人)だろうが、「陽性」と判断されれば、99%で感染してるんだから関係ない気もします。


どなたか、教えていただけると助かります
368132人目の素数さん:2007/04/05(木) 00:43:35
>>365
y=f(x)=(x-a)^2+4a-99 より軸はx=aだから、
20-a≦a≦12a→10≦aのときに軸が範囲に含むまれるので、最小値=f(a)=237
また、6<a<10のときは最小値=f(20-a)=237になる。
369132人目の素数さん:2007/04/05(木) 00:44:37
>>367
(1)からすでに間違ってる
370132人目の素数さん:2007/04/05(木) 00:49:21
>>367
「感染して陽性」「感染して陰性」「感染しなくて陽性」「感染しなくて陰性」はそれぞれどれだけいるのか?
371132人目の素数さん:2007/04/05(木) 01:02:27
>>356
その「最近」が四月一日だったら指さして笑ってやる。
372sage:2007/04/05(木) 01:08:38
>>367
感度、特異度、尤度比などをググれ。
(1)約1%
(2)約49.5%
373132人目の素数さん:2007/04/05(木) 01:10:23
>>367
本当に感染している確立なら
(1)99/100000
(2)9911/10000000
になったけど、これは感染していないのに「陽」と判定されたのを「本当に感染している」に含めてない。
374132人目の素数さん:2007/04/05(木) 01:12:29
じゃあ>>373は間違いだね
by>>373
375132人目の素数さん:2007/04/05(木) 01:34:02
0.0001*0.99/(0.0001*0*99+0.99999*0.01)≒0.0099
376132人目の素数さん:2007/04/05(木) 01:35:16
>>375 ×
0.0001*0.99/(0.0001*0.99+0.99999*0.01)≒0.0099
377132人目の素数さん:2007/04/05(木) 01:35:32
>>375
0.0001*0.99/0.010098=99/10098(およそ0.98%)
378132人目の素数さん:2007/04/05(木) 17:53:11
>>368さん
365です。
ありがとうございました。
379368:2007/04/05(木) 18:06:03
>>365
ところで、問題正しいか?
380132人目の素数さん:2007/04/05(木) 18:27:07
>>379さん
すみません。問題間違ってました。
私は
> y=x^2-2ax+a^2+4a-99
と書いてましたが、
y=x^2-2ax+a^2+40a-99
でした。
その他はあってました。

381368:2007/04/05(木) 20:17:10
>>380
では改めて、
y=f(x)=(x-a)^2+40a-99 より軸はx=aだから、
20-a≦a≦12a→10≦aのときに軸が範囲に含まれるので、最小値=f(a)=40a-99=237→a=42/5<10 より不適。
また、6<a<10のときは範囲が軸より「右」になるから、
最小値=f(20-a)=237→(a-2)(a-8)=0 より、条件を満たすのは、a=8,x=20-a=12
382132人目の素数さん:2007/04/05(木) 20:33:40
>>380さん
私も今、ちょっと違う解き方だけど
問題が解けたところです。
ヒントを下さってありがとうございました!
何度も本当にありがとうございました。
383132人目の素数さん:2007/04/05(木) 21:43:50
10万円持ってる人が、1万円札を10枚持ってる確率ってあるけど、
十万円札は無いので「十万円札を一枚持ってる確率」を外す事を何と言いますか。
384132人目の素数さん:2007/04/05(木) 21:48:30
捨象?
ごめん確率はやってないから知らん
385KingOfUniverse ◆667la1PjK2 :2007/04/05(木) 22:02:33
talk:>>383 標本を外すか、事象を外すか?
386132人目の素数さん:2007/04/05(木) 23:01:41
>>381さん
何度もすみません。

> また、6<a<10のときは範囲が軸より「右」になるから、

範囲が軸より「右」になる、っていう文の意味が
ちょっと分からなくて…。
すみません。説明していただいてもいいですか?
387132人目の素数さん:2007/04/05(木) 23:08:58
>>386
つまりまずグラフを書かないと話が始まらないということです
388132人目の素数さん:2007/04/05(木) 23:18:00
>>387さん
グラフを書いて、
20-a≦a≦12a のaに6や10を当てはめてみて
やっと分かりました。
何度もすみません。ありがとうございました。
389132人目の素数さん:2007/04/05(木) 23:43:16
>>362
右から1行3列の行列をかける写像
390132人目の素数さん:2007/04/06(金) 02:34:55
1と2と3と4と5と6と7と8と9と10の」最小公倍数
391132人目の素数さん:2007/04/06(金) 03:12:22
確率の問題です

箱の中に玉が赤5個、青4個、白3個入っています。ここから4個同時に取り出します。
このとき、取り出した色の種類が2色のみとある確率は?
392132人目の素数さん:2007/04/06(金) 03:13:18
× 2色のみとある確率は?
○ 2色のみとなる確立は?
393132人目の素数さん:2007/04/06(金) 03:19:29
>>391
赤を取り出さないのは何通り?
青を取り出さないのは?
白を取り出さないのは?
これらを足して、
取り出したのが一色だけになる場合を引く。
それから全ての取り出し方の数で割る。
394132人目の素数さん:2007/04/06(金) 03:39:45
赤青、赤白、青白で4個の組み合わせ/全数から4個とる組み合わせ?
xxxy,xxyy,xyyy * 3c2
395132人目の素数さん:2007/04/06(金) 03:43:24
y"=6x^2+x
396132人目の素数さん:2007/04/06(金) 03:48:44
>>391
青白で四個になる組み合わせの数と
赤白で四個になる組み合わせの数は違う。
赤青で四個になる組み合わせも、またこれらとは違う。
397132人目の素数さん:2007/04/06(金) 05:41:06
問1:次の微分方程式を演算子法で解け
 y"+6y'+13y=5sin(x)

(D^2+6D+13D)y=5sin(x) (D=d/dx)
y=5sin(x)/(D^2+6D+13D)まではできたのですが
その後の解き方が分かりません。
398132人目の素数さん:2007/04/06(金) 05:52:06
>>397
Im 5e^(ix)/(D^2+6D+13)
=Im 5e^(ix)/(-1+6i+13)
= (1/6)(-cosx+2sinx)
399132人目の素数さん:2007/04/06(金) 10:07:27
f(x)を(x-1)^2で割ると、2x+2あまり、(x+1)^2で割ると3x-1あまる。
このときf(x)=x^3-x^2-x+1を割ったあまりを求めよ

という問題で、解答の方に
f(x)=(x+1)(x-1)^2*D(x)+p*(x-1)^2+2x+2
と書いてあるのですが、
何故「p*(x-1)^2+2x+2」となるのかがわかりません。

説明していただけないでしょうか
よろしくお願いします
400132人目の素数さん:2007/04/06(金) 10:16:09
>>399
> f(x)を(x-1)^2で割ると、2x+2あまり

この条件から、直接そのように表せる。
401132人目の素数さん:2007/04/06(金) 10:25:17
すみません、よくわからないです。。。

それだけならわかるのですが、
その前にD(x)の部分があるので混乱してしまいます。
何故このような形にできるのでしょうか?
402132人目の素数さん:2007/04/06(金) 10:32:24
わからなかったら
f(x)=(x+1)(x-1)^2*D(x)+ax^2+bx+c
とでもおいて実際に割り算すればいい。

ax^2+bx+c=a(x-1)^2+(b+2a)x+c-a
(x-1)^2 で割ったあまり (b+2a)x+c-a が 2x+2 と一致するから
b+2a=c-a=2

ax^2+bx+c
=ax^2+2(1-a)x+a+2
=a(x-1)^2+2x+2
403132人目の素数さん:2007/04/06(金) 10:46:05
>>402
なるほど、式の意味はわかりました。
しつこいですが、
どういう考え方をすれば
このような式を導き出せますか?
説明されないとわからないままだと思うので…
404132人目の素数さん:2007/04/06(金) 10:58:24
>>403
覚える。
405132人目の素数さん:2007/04/06(金) 10:59:57
>>403
君は先人が見つけ出した方法をすべて自分で見つけようとでも思ってるのか?
406403:2007/04/06(金) 11:03:22
なんというか、
公式というかその分野をまだ理解できない感じですよね…すみません
407132人目の素数さん:2007/04/06(金) 11:13:57
2つの不等式|x+1|<2,|x−2|>Kをともに満たす整数xが1個だけ存在するように、正の定数Kの範囲を求めよ。またそのときのxを求めよ。

この問題で−3<x<1 ・・・・@
     x<2−K,2+K<x ・・・・A
までは理解できるのですが、次の「@、Aをともに満たす範囲に整数が1個だけ存在するためには−2<2−K≦−1である必要がある」
なぜその式になるかわかりません・・・ 高校の初歩中の初歩のような問題なんですが教えていただけないでしょうか?
408132人目の素数さん:2007/04/06(金) 11:16:32
すいません高校の質問スレがあったみたいですね。
スレ違い失礼しました;;
409132人目の素数さん:2007/04/06(金) 11:20:41
>>407
K > 0なのだから
2+K > 2 > 1

したがって
-3 < x < 1

2+K < x
の重なりは無い。

-3 < x < 1 に含まれる整数は-2,-1,0の3つある。

-3 < x < 1

x < 2-K
の重なりに整数が1つしかないとしたら
-3 < x < 2-K に含まれる整数が1つということ。

-3 < x < -2だと1つも含まない
右辺が少しでも大きくて
-3 < x < -1.9 だったりすると-2を含む

-3 < x < -1でも整数は1つだ。
しかしさらに右辺が大きくなると
-3 < x < -0.9だと x = -2,-1の2つ含む。
410132人目の素数さん:2007/04/06(金) 11:22:09
ありがとうございます。
ホントにすっきりしました。
モヤモヤしたまま先に進むの嫌だったんで・・・
本当に助かりました。
411あ〜む:2007/04/06(金) 11:56:46
次のinfで定まる c(e,n) の評価を知りたいんだけど、
特に 0<e<1 を固定して n→∞ としたときの 0 に
収束する速さを知りたい。

c(e,n)= inf{ c :
高々n次の多項式 p で p([0,e])⊆[0,c], p([1-e,1])⊆[1-c,1]
を満たすものが存在する. }
412TIBA:2007/04/06(金) 14:27:58
次の連立方程式を解きなさい。
2χ+У+Ζ=6
χ+2У−Ζ=0
χ+У+2Ζ=4

これ教えてください(-_-メ
413132人目の素数さん:2007/04/06(金) 14:33:14
>>412
2つ選んで文字を一つ減らせ。
別の組み合わせを選んで同じ文字を減らせ。
414132人目の素数さん:2007/04/06(金) 14:34:15
またすごい記号を使うのが出てきた。
χ=3, У=-1, Ζ=1
415TIBA:2007/04/06(金) 14:37:10
親切にありがとうございます!
416132人目の素数さん:2007/04/06(金) 15:53:48
>>398
ありがとうございます。
ようやくわかりました。
417132人目の素数さん:2007/04/06(金) 16:18:01
>>412
ギリシャ文字、キリル文字etcがごちゃ混ぜってのは
おまえは一体なに人なのかと。
418132人目の素数さん:2007/04/06(金) 16:19:31
カイ+ウー+2ゼータ=4
419132人目の素数さん:2007/04/06(金) 17:32:09
小文字のアルファベットを使うだけのことに何か抵抗でもあるのかねえ・・・
420132人目の素数さん:2007/04/06(金) 17:32:30
卍+Ч+乙≒3
421132人目の素数さん:2007/04/06(金) 20:20:26
突然ですが、ルーレットやバカラの必勝法として有名なウィナーズ投資法が駄目駄目だってことを数学的に証明できないですか?
422132人目の素数さん:2007/04/06(金) 20:21:48
423菅_理人@kmath1107BBS ◆.ffFfff1uU :2007/04/06(金) 20:22:21
>>421
一回一回の賭けの期待値が全て負であることを言えば十分だよ。
424132人目の素数さん:2007/04/06(金) 20:25:59
そもそもベットをどの様にいじったところで期待値は動かないってことで終わりなのかな?
425132人目の素数さん:2007/04/06(金) 20:27:42
423>>
噛み締める様に3回読んだところで、納得しました。ありがとうございました。^^
426132人目の素数さん:2007/04/06(金) 20:35:20
1:n人ゲームでn人が結託すれば、つねにn人側がかつ
427132人目の素数さん:2007/04/06(金) 21:08:02
事前に1〜6の中からの好きな数字を5つ(重複可能)選び
サイコロを振り、選んだ数字をすべて揃えた人がアガリというゲームをします
これをx人でやった場合、同時にアガル確立はどれくらいあるのでしょうか?

*1 選んだ数字に同じ数字がある場合、同じ回数だけ選らんだ数字が出ないとアガレません
*2 選んだ数字の順番は関係ありません。選らんだ数字が出ればアガリです

宜しくお願いします
428132人目の素数さん:2007/04/06(金) 21:15:02
数学って何だろうね?
429132人目の素数さん:2007/04/06(金) 21:17:55
差分法って何ですか?
430132人目の素数さん:2007/04/06(金) 21:33:10
>>429
ググレカス
431132人目の素数さん:2007/04/07(土) 18:40:37
ロース20枚、バラ20枚、たん10枚、豚バラ20枚
4人で焼肉、ロースを5枚以上食べられる確率は?
432132人目の素数さん:2007/04/07(土) 18:42:10
>>431
全員平等なのか?
433132人目の素数さん:2007/04/07(土) 20:09:07
箸は同時にいれられない。焼けあがるのは入れた順、誰の肉をとってもいい。
434132人目の素数さん:2007/04/07(土) 20:11:20
右回りに食べてゆくとしたら?
435132人目の素数さん:2007/04/07(土) 20:11:51
.
436132人目の素数さん:2007/04/07(土) 20:12:18
肉は2枚まで同時における
437132人目の素数さん:2007/04/07(土) 20:34:41
>>431
まず確率というものを勉強することからはじめてくらさい
438132人目の素数さん:2007/04/09(月) 09:13:53
y=x^2-2ax+2a+1の0≦x≦3における最少値の中の最大値を求めよ

ぼんやりとは解けるのですが解答に自信がありません
よろしくお願いします 
439132人目の素数さん:2007/04/09(月) 09:17:30
>>438
日本語でおk

あえてエスパーすると
*最小値をaで表せ
*その最大値を求めよ
あたりの典型問題だとは思うが
質問者が恣意的に改変しているとすれば
回答は不可能である。
440132人目の素数さん:2007/04/09(月) 10:11:41
>>438
じゃあ「ぼんやりと」した解答を書け
441132人目の素数さん:2007/04/09(月) 14:35:41
自然対数eの値の中の最初の、連続する10桁の素数ってどうやって求めるんですか?
分かる方お願いします。
442132人目の素数さん:2007/04/09(月) 15:43:51
数的推理の問題でも質問していいですか?
443132人目の素数さん:2007/04/09(月) 15:45:06
>>442
よく分からないけどパズル板に行ったらどうですか?
444132人目の素数さん:2007/04/09(月) 15:47:44
パズル版!ありがとうございます。
445132人目の素数さん:2007/04/09(月) 16:47:27
>>441
連続する10桁の素数のリストを作ってから
比べるだけ。
446132人目の素数さん:2007/04/09(月) 17:19:05
>>445
素数のリストを先に作るのと
eに含まれる10桁を順番に素数判定するのとどちらが速いだろう?
447132人目の素数さん:2007/04/09(月) 17:45:32
>>446
既存の素数表使え。
448132人目の素数さん:2007/04/09(月) 19:21:38
質問なんですけど、
1+2+3+・・・・・って、ずっと足していくとどうなるんですか?
449132人目の素数さん:2007/04/09(月) 19:22:03
>>448
疲れる
450132人目の素数さん:2007/04/09(月) 19:27:47
>>449
確かに疲れちゃいそうです。
今日の朝日新聞の科学の記事でオイラーが紹介してあってずっと足していくとその和が-1/12になるらしくてどうしてそうなるのか誰か教えてください。
451132人目の素数さん:2007/04/09(月) 19:30:49
なんで新聞ってのは、そういう素人を惑わすような事ばかり書くかねえ‥‥。
わかって書いてるんだろうか?
452132人目の素数さん:2007/04/09(月) 19:31:53
はあ・・・
453132人目の素数さん:2007/04/09(月) 19:32:37
>>450
疲れる
454132人目の素数さん:2007/04/09(月) 19:33:30
>>450
朝日新聞の記者が疲れて計算ミスをしたんだろうな
455132人目の素数さん:2007/04/09(月) 19:34:30
素人が書いてるからだよ。伝染病のように広まっていくのさ。
ほんとこれだからマスゴミは。
456448:2007/04/09(月) 19:36:50
>>455
ってことはマスコミの間違いですか?
もう訳わかんないですよね。
457132人目の素数さん:2007/04/09(月) 19:38:03
ですよね、とか聞くなカス
消えろ
458132人目の素数さん:2007/04/09(月) 19:39:04
Σ[k=1,n]kが収束するなんて聞いたこと無いんだが
459132人目の素数さん:2007/04/09(月) 19:39:28
>>456
大学で勉強すればわかるからそれまで我慢してね
460132人目の素数さん:2007/04/09(月) 19:39:53
無知乙
だが、聞いたことなくていい
461448:2007/04/09(月) 19:41:41
すいませんでした。
462132人目の素数さん:2007/04/09(月) 19:46:59
>>459
駅弁だと院に進まないと出てこないってことも起こりそうな気がす
463132人目の素数さん:2007/04/09(月) 19:47:48
出てこないとか受身じゃなくて、勉強しにいくんだろ
464132人目の素数さん:2007/04/09(月) 19:49:03
それなら高校生でも勉強できるんだから>>459の回答は不適切
465132人目の素数さん:2007/04/09(月) 19:58:25
不適切(笑)
466132人目の素数さん:2007/04/09(月) 20:00:42
回答の意図を汲めてないんだから>>464の批判は的外れ
467132人目の素数さん:2007/04/09(月) 20:01:18
-1/12 じゃなかったっけ?
468132人目の素数さん:2007/04/09(月) 20:17:10
「ゼータ関数」とか「カシミール効果」とかで調べると良いんじゃないでしょうか
俺は専門じゃないのでさっぱりだけど……
469468:2007/04/09(月) 20:23:20
http://science6.2ch.net/test/read.cgi/math/1038540474/650
まだ読んでないんだけど、レスをみつけたので置いときます
470132人目の素数さん:2007/04/09(月) 20:32:39
>>468
大学一、二年の教養レベルで専門もクソもないきがするんだが。
471468:2007/04/09(月) 20:40:25
>>470
少なくとも俺はそんなのやってないぞ
ζ(2)くらいは学部の頃の演習問題だったけど
472132人目の素数さん:2007/04/09(月) 20:42:24
整数nを下のように素因数分解したとき
n=p^a*q^b*r^c......

6/pi^2 =0.6079...< (1-1/p^(a+1))*(1-1/q^(b+1))*(1-1/r^(c+1)).... < 1
473132人目の素数さん:2007/04/09(月) 20:44:53
>>471
解析関数の解析接続で関数等式が保たれるとか
関数論で普通にやるだろ。
474132人目の素数さん:2007/04/09(月) 20:45:43
数学科基準で言われても
475132人目の素数さん:2007/04/09(月) 20:47:12
>>472
失敬。勘違いでした。取り消し
476132人目の素数さん:2007/04/09(月) 20:51:01
>>474
これ数学科基準てwwww
どんだけレベルの低い数学科だよwwww
477468:2007/04/09(月) 20:52:34
>>473
俺の大学とカリキュラムが違う、ってだけの事じゃないか?
俺の所じゃ関数論は学部3年にならないと無いから、教養って気がしない
3年になった頃には、俺は代数と幾何しかやらなくなってた
478132人目の素数さん:2007/04/09(月) 21:07:32
東大ですが解析接続って言葉自体知りません><
479132人目の素数さん:2007/04/09(月) 21:11:02
ゼータ関数のあの表示はs>1でのみ有効だから、それ以外の値を代入
した式をわざとかくから勘違いする人が増える。
480132人目の素数さん:2007/04/09(月) 21:56:28
>>479
標語的には面白いし、あれが収束する総和法が与えられてると
見ることもできるし、もっというとあれは通常の位相とは別の位相で
収束する級数で、そっちの位相の方が自然とおもえるような舞台が
あるのかもしれない、とか想定するのもおもしろいのだよ。
数論的には有限アデールのほうが重要ジャン?
481132人目の素数さん:2007/04/09(月) 22:09:01
ん?実数の範囲内だと収束しないのか?
482132人目の素数さん:2007/04/10(火) 01:42:06
>>441
とりあえず、2chなんかで聞いてる時点で
google入社は諦めることだ。
483132人目の素数さん:2007/04/10(火) 01:51:32
方形分布関数の2次元フーリエ変換を考えているのですが,よく分かりません.

方形分布関数とは,
f_rect(x, y) = 1 ( |x|<=a, または |y|<=bのとき ) その他は = 0
という関数です.

∫[y=-∞, ∞] ∫[x=-∞, ∞] f_rect(x, y) exp(-j(ω_x x + ω_y y)) dx dy
この f_rect(x, y) をどのように記述したらよいのかが分かりません.

f_rect(x, y) の形は「十字」になりますが,
|x|<=a かつ |y|<=b の長方形部分と,その他の4つの長方形
合計5つの項に分解すればよいのでしょうか?
つまり,
∫[y=-b, b] ∫[x=-a, a] exp(*) dxdy
+ ∫[y=-b, b] ∫[x=a, ∞] exp(*) dxdy + ∫[y=-b, b] ∫[x=-∞, -a] exp(*) dxdy
+ ∫[y=-∞, -b] ∫[x=-a, a] exp(*) dxdy + ∫[y=b, ∞] ∫[x=-a, a] exp(*) dxdy
を計算すれば良いのでしょうか?

方針は合っているでしょうか?
間違っているならば,ご教授お願いします.
484483:2007/04/10(火) 01:52:33
exp(-j(ω_x x + ω_y y))

exp(*)
と省略しています.

お願いします.
485132人目の素数さん:2007/04/10(火) 02:10:24
>>483
問題ないと思う
486132人目の素数さん:2007/04/10(火) 02:11:30
f=δ(x-a)+δ(y-b)
487132人目の素数さん:2007/04/10(火) 02:15:43
f_rect=Sδ(x-a)da+Sδ(y-b)db
488132人目の素数さん:2007/04/10(火) 02:18:27
やっぱココの人は微積高校時代すんなり解けたの?
VCになってさっぱりなんですが…
489132人目の素数さん:2007/04/10(火) 02:18:45
f_rect=SSδ(x-a)δ(y-b)dadb
490132人目の素数さん:2007/04/10(火) 02:27:50
>>481
1+2+3+・・・が実数の通常の距離位相で収束するのか。
491132人目の素数さん:2007/04/10(火) 02:33:45
f_rect=SSδ(x-a)δ(y-b)dadb
SSf_recte^-2πi(kx+sy)dxdy
=SSSSδ(x-a)δ(y-b)dadb e^-2πi(kx+sy)dxdy
=SS e^-2πi(ka+sb)dadb
=δ(k)δ(s)

492132人目の素数さん:2007/04/10(火) 02:39:27
初歩的な話で申し訳ないのですが
たとえば正弦関数のような周期関数が引数を無限にとばしても
収束「しない」ことは、どうやって証明すればいいでしょうか。
それとも、正弦関数の定義より・・・くらいでも問題ないでしょうか。
493132人目の素数さん:2007/04/10(火) 02:44:13
y=(x-1)^2はy=x^2をx軸方向に1移動させたグラフであるって定理だよね
494132人目の素数さん:2007/04/10(火) 02:48:29
>>492
周期関数でも定数関数なら収束する。
逆にx→∞で収束する周期関数があるならそれは定数関数。
495132人目の素数さん:2007/04/10(火) 02:50:45
>>492
「収束」の定義を使って背理法で証明する。


とくにsin xのときにはε=1/2とすると、
sin x>1/2をみたすxが無限に存在する。
496132人目の素数さん:2007/04/10(火) 02:53:53
d(f,a)->0
497132人目の素数さん:2007/04/10(火) 02:54:24
>>492
収束する部分列を二つ、異なる値に収束するようにとる
というのでもいけるんじゃないか。
498132人目の素数さん:2007/04/10(火) 02:57:22
>>497
その通りです。
499132人目の素数さん:2007/04/10(火) 02:59:26
y=(x-1)^2はy=x^2をx軸方向に1移動させたグラフであるって定理だよね
500132人目の素数さん:2007/04/10(火) 03:08:43
>494
>495
>497-8

参考になりました。どれも納得のいくものだったので活用させていただきます。

趣旨からそれてはいるのですが>494さんの指摘はごもっともでお恥ずかしい限りです。
多少の興味でかじってはいるのですが、結局は野暮で怠惰な学生なもので、精密な議論をするには
まだまだ修行が足りませんね。
501483:2007/04/10(火) 08:22:00
>>485
ありがとうございます.
502483:2007/04/10(火) 08:40:23
>>489,491
見落としていました.
レスありがとうございます.
一見では理解できませんが,考えて見ます.
503483:2007/04/10(火) 08:54:01
連稿失礼します

>>489,491
ノートに書き出したところ,何をしているのかが分かりました.
このように考えることも出来るのですね.
私の考え方と同じ結果となるのかも試してみます.

ありがとうございました.
504132人目の素数さん:2007/04/10(火) 10:21:19
>>483
なにをしたいのか、さっぱりわからん。
> この f_rect(x, y) をどのように記述したらよいのかが分かりません.
f_rect なんてどうせゼロか1しかとらんのだから、特殊関数まで
引っ張り出して「記述」する必要などさらさらないだろう。
exp(-i ω_x x - i ω_y y)の積分範囲が [-a,a]×[-b,b]の四角形
領域になるというだけのことだろう。
505504:2007/04/10(火) 10:28:04
そのうえ、exp(-i ω_x x - i ω_y y) = exp(-i ω_x x)exp(-i ω_y y)
と分解できるのだから、この面積分は1次元の積分の積に表現できて、
∫[y=-∞, ∞]∫[x=-∞, ∞] f_rect(x, y) exp(-i(ω_x x + ω_y y)) dxdy
=∫[y=-a,a]∫[x=-b,b] exp(-i(ω_x x + ω_y y)) dxdy
=∫[y=-a,a]exp(-i ω_x x) dx ・∫[x=-b,b] exp(-i ω_y y) dy.
結果までは表示しないが、sin(x)/xという関数(標本化関数)の
積だろう。
506132人目の素数さん:2007/04/10(火) 21:35:05
層のpush-forwardというのが良く分かりません。
引き戻しなら大体分かるのですが。
どのようなものか、教えていただけないでしょうか?
507132人目の素数さん:2007/04/11(水) 04:14:07
もし、人間の指が6本あったなら人類は12進数を使っていたのでしょうか?
508132人目の素数さん:2007/04/11(水) 04:14:09
重積分の計算問題なのですが、
∬sin(x+y)dxdy , (0≦y≦x, π/4≦x≦π/2)

普通にxを固定してyで積分した後、xで積分したのですが、
解答では答えが1になっていたのですが、1になりません。
どういった過程で1になるのでしょうか?
509132人目の素数さん:2007/04/11(水) 04:43:36
>>508
どうやっても1にならない
510132人目の素数さん:2007/04/11(水) 05:19:26
>>509
そうですよね。解答が間違ってると思ったので…。他にも誤植や間違いの多い本だったので。ありがとうございました
511132人目の素数さん:2007/04/11(水) 05:43:43
2の133332乗って何でしょうか?
512132人目の素数さん:2007/04/11(水) 06:11:56
>>511
そんなでかい数知ってどうすんの?
およそ8.53850e40136
513132人目の素数さん:2007/04/11(水) 06:36:35
0<x=<1で、y=1-x
-1<x=<0で、y=x+1
f(x+2)=f(x)という関数を考えてみます(三角形が連続で並んでるような関数です)
この関数をフーリエ級数に展開するためにはどうすればいいですか?
f(x+2π)=f(x)ならわかるけど、f(x+2)=f(x)じゃわからない・・・
514132人目の素数さん:2007/04/11(水) 07:07:13
>>512
自分に約何人分の遺伝子が入ってるか知りたくて、
400万年前の人類の始祖から俺までのおよその交配人数を知ってみたかったんだ。
アニメーション製作の題材にもしようかと思ってさ、トンクス!!

515132人目の素数さん:2007/04/11(水) 07:15:06
>>512
あ、8.53850e40136のeってどういう意味?
516132人目の素数さん:2007/04/11(水) 08:21:47
>>513
>f(x+2π)=f(x)ならわかるけど、f(x+2)=f(x)じゃわからない・・・
その関数の周期は2だから、
たとえばt=πxとでも置き換えて周期を2πにしてやればいいだろ。
517132人目の素数さん:2007/04/11(水) 10:12:43
8.53850*10^40136
518132人目の素数さん:2007/04/11(水) 10:18:01
>>514
血縁同士で結婚する事を考慮に入れるべき
519132人目の素数さん:2007/04/11(水) 11:03:11
8.53850564621533047152304985035624653354…*10^40136
520132人目の素数さん:2007/04/11(水) 11:15:46
>>513
 f(x) = 1 - |x|  (|x|≦1)
    = (1/2) + (4/π^2){cos(πx)/(1^2) + cos(3πx)/(3^2) + cos(5πx)/(5^2) + …… }
521132人目の素数さん:2007/04/12(木) 18:41:27
>>508
(3-√2)/2.
522132人目の素数さん:2007/04/19(木) 21:11:28
三十一日。
523132人目の素数さん:2007/04/23(月) 01:57:27
こんばんわking
524132人目の素数さん:2007/04/25(水) 14:03:42
こんにちはking
あなたーの えがーお
525132人目の素数さん:2007/04/25(水) 14:21:21
1/cos(シータ)の積分お願いします
526132人目の素数さん:2007/04/25(水) 14:27:08
$ \int \sec x \, \D x
= \log \left| \frac{1 + \tan \frac{x}{2}}{1 - \tan \frac{x}{2}} \right|
= \frac{1}{2} \log \frac{1 + \sin x}{1 - \sin x}$
527132人目の素数さん:2007/04/25(水) 14:32:35
>>526 の \D x は dx と読んでね。

方法1:tan θ/2 = t とおく.
方法2:1/cos θ = cos θ / {(1 - sin θ)(1 + sin θ)} と変形し,sin θ = t とおく.
528132人目の素数さん:2007/04/25(水) 15:36:06
>>525
2/cosθ = 2(cosθ)/(cosθ)^2 = 2(cosθ)/(1-(sinθ)^2)
= {(cosθ)/(1-sinθ)} + {(cosθ)/(1+sinθ)}
を積分すると

∫{2/cosθ} dθ = -log(1-sinθ) + log(1+sinθ) +c = log{(1+sinθ)/(1-sinθ)} +c

ここまででもいいですが

cos(θ+(π/2)) = sinθなので
(1+sinθ)/(1-sinθ) = {1+cos(θ+(π/2))} / {1-cos(θ+(π/2))}
と、tanの半角公式
tan(x/2) = {1+cos(x)}/{1-cos(x)} から
(1+sinθ)/(1-sinθ) = tan((θ/2) + (π/4))
となります。

∫{1/cosθ} dθ = (1/2) log{tan((θ/2) + (π/4)} +(c/2)

確かメルカトル積分という名前だったかと思います。
名前から想像できるものに使われる積分です。
529132人目の素数さん:2007/04/25(水) 21:02:56
>>271をどなたかよろしくお願いします。
530132人目の素数さん:2007/04/25(水) 22:43:11
ちょっと教えてください
質量Mの惑星が質量mの物にかける重量は
(G*mM)/(距離^2)
とりあえず簡単にするために
宇宙を二次元と考えて、質量Mの惑星がある場所を(0,0)
質量mの物がある場所を(x,y)とすると
物にかかる力F(x,y)の大きさは

G*mM/√(x^2+y^2)
力の方向は(x,y)
だから結局重力によって作られる力場は
F(x,y)={G*mM/(x^2+y^2)}*(x,y)

これでいいの?
531132人目の素数さん:2007/04/25(水) 23:22:58
>>530
あれ、重力って赤緑なんだっけ課…??
532132人目の素数さん:2007/04/25(水) 23:25:05
>>529
どういう問題なの?
533132人目の素数さん:2007/04/26(木) 01:53:41
90度を三等分する問題(コンパスで)

従姉妹に聞かれたけどわかりません(><)
まじ お願いします!
534132人目の素数さん:2007/04/26(木) 02:01:51
>>533
直角の頂点から円弧を描いてから線分との交点から同じ半径の弧を描いて
前の弧と2つの交点をそれぞれ頂点と結ぶ
535132人目の素数さん:2007/04/26(木) 02:19:15
>534
ありがとうございました

ようするに正三角形の60度を使えばいいのですね
536132人目の素数さん:2007/04/26(木) 02:23:52
>>534
すまんが日本語で説明してくれ。
537132人目の素数さん:2007/04/26(木) 03:28:30
まあ、マルチだったわけだが
538132人目の素数さん:2007/04/26(木) 08:02:00
1/1 + 1/4 + 1/9 + 1/16 + ・・・ + 1/n^2 = (1/k)^2
これをnの式で表すと一般項どうなりますか?結果だけ教えくださいな
539132人目の素数さん:2007/04/26(木) 08:47:42
すみません,>>272でした。よろしくお願いします。
540132人目の素数さん:2007/04/26(木) 08:49:01
またまたすいません,どうやらスレ間違えましたのでスルーして下さい。
541132人目の素数さん:2007/04/26(木) 08:58:15
なんだかよくわからんが
>>529=539=540
で、結局、誤爆だった、ということなのかな。

実は、俺が>>272であり、かつ
>>216その他であったりするわけだが
あの頃のバカがまた蒸し返し始めたか、とオモタ。
542132人目の素数さん:2007/04/26(木) 09:42:57
全ての自然数nに対し、b(n)>0、Σ[n=1,∞]b(n)=∞、
a(n)/b(n)→α(実数)(n→∞)のとき、Σ[k=1,n]{a(k)/b(k)}→α
を示すにはどうすればよいのでしょうか?
543132人目の素数さん:2007/04/26(木) 09:43:45
正五角形の作図の問題に関して、x^4+x^3+x^2+x+1=0は2次方程式の解の公式を2回使えば
解けると書いてあるんですがわかめません(∋_∈)
544132人目の素数さん:2007/04/26(木) 09:50:33
>>543
ググレカス

相反方程式
545132人目の素数さん:2007/04/26(木) 09:58:25
546132人目の素数さん:2007/04/26(木) 10:42:16
>>543
両辺(x^2≠0)で割って、x+(1/x)=tと置くとtの二次方程式に帰着。
547132人目の素数さん:2007/04/26(木) 13:00:37
結局、今井先生が出演した鑑定団の動画は無いのですか?
548132人目の素数さん:2007/04/26(木) 16:14:51
75%と30%のショ糖水溶液から50%のショ糖水溶液を作るための比率を求めなさい
75%と30%のショ糖水溶液は何対何になりますか

549132人目の素数さん:2007/04/26(木) 17:57:43
>>548
75%のショ糖水溶液xグラム内にショ糖は0.75xグラム
30%のショ糖水溶液yグラム内にショ糖は0.3yグラム

まぜるとx+yグラムで
ショ糖が0.75x+0.3yグラムの水溶液になる。

濃度が50%だから(0.75x+0.3y)/(x+y) = 0.5

75x+30y=50(x+y)
25x=20y
5x=4y
x:y=4:5
550132人目の素数さん:2007/04/27(金) 00:12:54
殆ど関係ないけど、常温常圧で75%のショ糖水溶液って存在できるのかな
551132人目の素数さん:2007/04/27(金) 00:13:40
2つの箱に、異なる6個の球を空箱ができないように入れる。
以下の条件の時、球の入れ方は何通りあるか。
(i)箱に区別がある場合
(ii)箱に区別がない場合


解答と答え合わないけど、どこで間違ってるのかわかりません。
可能なら途中の説明や式とかも含めて教えてください。
552132人目の素数さん:2007/04/27(金) 00:22:06
じゃあ自分の解答を書いてみろ
553132人目の素数さん:2007/04/27(金) 03:18:12
>>550
気合いがあればできる
と思う
554132人目の素数さん:2007/04/27(金) 03:34:18
水じゃなくて俺の唾液には溶けそうな気がする。
555132人目の素数さん:2007/04/27(金) 05:38:21
1/{1+e^(s/t)}を(a/1)+{b/e^(s/t)}って形にしたいんですけどaとbには何が入りますか?
556132人目の素数さん:2007/04/27(金) 06:50:25
は?
557132人目の素数さん:2007/04/27(金) 07:23:22
ベクトルの外積について質問
一次独立な三次元ベクトルが二つ(A,B)あったら、A×Bが外戚なんでしょ?
たとえば五次元のベクトルが四つあったら同じような計算をしていいんですか?

A×B×C×Dが外積?
558132人目の素数さん:2007/04/27(金) 07:39:08
それが何を意味するかわからんが
三次元で任意のベクトルC に対して
(A×B)・C = |A B C|
が成り立つように、任意のベクトルE に対して
(A×B×C×D)・E = |A B C D E|
とでも定めればいいんじゃ?
559132人目の素数さん:2007/04/27(金) 11:21:29
>>555
1/(1+x) = a + (b/x)にするってこと?
無理
560清書屋:2007/04/27(金) 11:30:51
>>551
1. 一つ目の箱にk個玉を入れるのはC[6,k]通り。但し、0<k<6。
残りは全てもう一つの箱に入れる。これは一通りに定まる。
C[6,1]+C[6,2]+C[6,3]+C[6,4]+C[6,5]

または、空箱ありでも良い場合を数え上げてから
空箱が出来る場合を引く。
2^6-2

2. 箱を区別しなければ1で求めたものの半分。
(1で箱の中身が逆になっているものを同じと考える。)
561132人目の素数さん:2007/04/27(金) 11:45:15
>>560
ありがとうございます。

謎が解けた。
問題を「2つの箱に異なる6個の球から2つ選んで入れる」に
勝手に脳内変換してた。
テラバカスorz
562132人目の素数さん:2007/04/27(金) 12:05:34
>>550
常圧の場合、60℃で無理、80℃では可能
563132人目の素数さん:2007/04/27(金) 14:37:13
>>557
外積代数、共変p-ベクトル、外微分形式あたりを自分で調べろ。
564132人目の素数さん:2007/04/27(金) 16:18:45
教えてください、お願いします。

数列{An}がつぎの条件をみたすとする。定数0<c<1が存在して、

c|An - A(n-1)|≧|An - A(n-1)| (n≧2)

このとき、{An}はコーシー列であることを示せ。

よろしくおねがいします。
565132人目の素数さん:2007/04/27(金) 16:27:25
>>564
マルチ
566132人目の素数さん:2007/04/27(金) 16:41:06
>>564
マルチ死ね
567132人目の素数さん:2007/04/27(金) 17:29:47
教えてください、お願いします。

数列{An}がつぎの条件をみたすとする。定数0<c<1が存在して、

c|An - A(n-1)|≧|An - A(n-1)| (n≧2)

このとき、{An}はコーシー列であることを示せ。

よろしくおねがいします
568132人目の素数さん:2007/04/27(金) 17:31:11
教えてください、お願いします。

数列{An}がつぎの条件をみたすとする。定数0<c<1が存在して、

c|An - A(n-1)|≧|An - A(n-1)| (n≧2)

このとき、{An}はコーシー列であることを示せ。

よろしくおねがいします
569132人目の素数さん:2007/04/27(金) 17:33:59
{An}がコーシー列でないとする。
このとき0<c<1だから、
c|An - A(n-1)|<|An - A(n-1)|
この対偶を取ると、
定数0<c<1が存在して、
c|An - A(n-1)|≧|An - A(n-1)
のとき{An}はコーシー列である
570132人目の素数さん:2007/04/27(金) 17:34:21
教えてください、お願いします。

数列{An}がつぎの条件をみたすとする。定数0<c<1が存在して、

c|An - A(n-1)|≧|An - A(n-1)| (n≧2)

このとき、{An}はコーシー列であることを示せ。

よろしくおねがいします
571132人目の素数さん:2007/04/27(金) 17:35:12
{An}がコーシー列でないとする。
このとき0<c<1だから、
c|An - A(n-1)|<|An - A(n-1)|
この対偶を取ると、
定数0<c<1が存在して、
c|An - A(n-1)|≧|An - A(n-1)
のとき{An}はコーシー列である
572132人目の素数さん:2007/04/27(金) 17:35:33
荒らすなよ
573132人目の素数さん:2007/04/27(金) 17:48:12
>557
>たとえば五次元のベクトルが四つあったら同じような計算をしていいんですか?
>A×B×C×Dが外積?

なにが質問したいの? 別に三次元でも A×B×C×Dは計算可能
外積がわかってないな
574132人目の素数さん:2007/04/27(金) 18:06:30
>>573
いや、三次元『では』A×B×C×Dは計算可能だけど、
五次元ではそもそもA×Bが三次元みたいには定義できない。
だから「三次元『でも』」という言い方はおかしい。

>>557が聞きたいのはおそらく
「四次元以上でも三次元の外積みたいなのがあるのか?」という
素朴で曖昧な疑問だと思う。
575132人目の素数さん:2007/04/27(金) 20:05:43
>574
いや  こいつにとって「×」が
グラスマン代数の積を示す。 
と解釈すれば問題ない
576132人目の素数さん:2007/04/27(金) 21:47:17
お尋ねします。
□の中に+、−、×、÷のいずれかを
書き入れて式を完成させる問題です。
問題
(1□8□1)□8=9
誰か解いてください〜お願い。
577132人目の素数さん:2007/04/27(金) 21:48:15
÷+×
578132人目の素数さん:2007/04/27(金) 21:57:19
>577
ありがd
579132人目の素数さん:2007/04/27(金) 21:58:36
>577
ありがd
580132人目の素数さん:2007/04/27(金) 23:20:04
テンソルって要するに
2次のテンソル=オセロのますのような数
3次のテンソル=ルービックキューブのような数
4次のテンソル=四次元のルービックキューブのような数

こんな感じでおkなの?
581132人目の素数さん:2007/04/27(金) 23:21:06
xの2次方程式x^−(2a+1)x+a^=0が実数解をもつようなaの範囲を求めよ
判別式を使うんでしょうか?
教えていただけないでしょうか
582132人目の素数さん:2007/04/27(金) 23:21:44
>>580
全然オッケーじゃないぉ
583132人目の素数さん:2007/04/27(金) 23:27:00
>>581
>>1
584132人目の素数さん:2007/04/27(金) 23:30:01
>>581
死ね^-^
585132人目の素数さん:2007/04/27(金) 23:44:04
え・・スレ違いでしょうか・・・?
586132人目の素数さん:2007/04/27(金) 23:46:18
y=3-x/x-4

この逆関数の求め方を教えてください
587132人目の素数さん:2007/04/27(金) 23:46:32
不等式3x+1>2aを満たすxの最小の整数値が4であるとき、整数a
の値を全て求めよ。
という問題がわかりません><
どなたか教えていただけますか??
588132人目の素数さん:2007/04/27(金) 23:47:48
不等式3x+1>2aを満たすxの最小の整数値が4であるとき、整数a
の値を全て求めよ。
という問題がわかりません><
どなたか教えていただけますか??
589132人目の素数さん:2007/04/27(金) 23:48:29
表記を間違えていると思われる
書き直し
590132人目の素数さん:2007/04/27(金) 23:48:36
このスレには数式の書き方のテンプレは無いんだな
591132人目の素数さん:2007/04/27(金) 23:49:26
>>588
x=4の時3x+1>2aを満たすようなaを求めろ
592589:2007/04/27(金) 23:50:04
>>589>>581
593132人目の素数さん:2007/04/27(金) 23:53:15
>>586
y=3-1-4=-2に逆関数は無いよ…
594132人目の素数さん:2007/04/27(金) 23:53:22
二つの二次方程式2x2乗+kx+4=0、x2乗+x+k=0が共通の実数解を
持つように定数kの値を定め、その実数解を求めよ。
がわかりません。
教えてください。
595132人目の素数さん:2007/04/27(金) 23:56:03
592さんありがとうございます!!
591さん
ということは6,5>a
ということですか??
596132人目の素数さん:2007/04/27(金) 23:57:10
592さんありがとうございます!!
591さん
ということは6,5>a
ということですか??
597132人目の素数さん:2007/04/28(土) 00:00:26
君がそう思うならいいんじゃないのか
598132人目の素数さん:2007/04/28(土) 00:00:26
点Oを原点とする座標平面上に点A(-2,2)と直線l:y=二分の一xがある。x軸に関して点Aと対称な点をBとし、点Bを通り、直線lに平行な直線をmとする。







・直線mと直線OAとの交点をCとする。ΔOBCの面積を求めよ。         
この問題教えてください!
599132人目の素数さん:2007/04/28(土) 00:00:26
>>594
実数解をαとすると、
2α^2+kα+4=0、α^2+α+k=0
2式から、(k-2)(α-2)=0
600132人目の素数さん:2007/04/28(土) 00:22:28
皆さんはご存知ですか?
兵庫・尼崎で起きた小4男児による同級生女児への性的暴行事件。

加害男児は複数児童が居る自宅マンションへと被害者の女児を誘い、
そこで性的な暴行を加えたそうです。
その件がある保護者から担任教師に伝えられ
学校側は市教委と協議の上対応していたはずだったにも関わらず、
何と2度目の性的暴行事件が起きています。

男児は行為を認め、保護者と共に一旦は被害女児と母親に対して謝罪したものの、
その後話はもつれ、最終的には加害男児の父親が警察に対して「恐喝された」と訴える始末。
今でも加害男児、被害女児ともに同じ学校へ通っているとのこと。

*まとめサイト
http://faq.s12.dxbeat.com/index.php?TOP
*ニュース映像
http://www.youtube.com/watch?v=lp6GUXIfi0U
*事件概要動画
http://www.youtube.com/watch?v=vOrs75kR0co

何故かマスメディアでも大きく取り上げられず、市教委や県教委の回答も曖昧なまま。
異様なまでに隠蔽された雰囲気にこの事件をもっと沢山の人に知ってもらいたいと思っています。

何かしらの行動を起こしたくなったら
2ちゃんねる掲示板内を「おまえらに協力して欲しいことがある」で検索して下さい。
601132人目の素数さん:2007/04/28(土) 00:26:49
小4で同級生とセクースかぁ…うらやましいなあ……
602132人目の素数さん:2007/04/28(土) 00:44:21
来週解いてきて説明しなくちゃいけない問題。


a,b を定数とし、2次関数 y=3x2乗-ax-a-b のグラフをCとする。グラフCは直線 x=a/ア を軸とする放物線であり、
グラフCとx軸とが異なる2つの共有点をもつのは、 b>-a2乗+イウa/エオ のときである。

以下、グラフCとx軸とが異なる2つの共有点をもち、その1つのx座標が1であるとする。このとき、 aを用いてbを表
すと b=カキa+クである。

また、もう一方の共有点のx座標はa-ケ/コであり、これが区間-1≦x≦0に含まれるaの値の範囲は, サ≦a≦シ である



aがこの範囲にあるとき、グラフCの頂点のy座標の最大値はスセ/ソである。(05 センター試験追試)


/は分数です、ア〜ソの片仮名は数字や記号が入る答えです。
当方あほな文型なんで全くちんぷんかんなんです。オネガイシマス。
603132人目の素数さん:2007/04/28(土) 00:45:31
>>602
文科系もできない "文型" のようだな
604132人目の素数さん:2007/04/28(土) 00:56:49
実数x,y,zが次の2式
x+y+z=1
x≧y≧z≧0

を満たしながら変化する時、ax+by+czのとり得る範囲を求めよ。
ただしa.b,cはa<b<cを満たす定数とする。


どっかの入試の過去問らしいんですが解けないんです…
よろしくおねがいします…
605132人目の素数さん:2007/04/28(土) 00:59:34
599さん
ありがとうございます!
606132人目の素数さん:2007/04/28(土) 01:16:03
問題と言うか質問なんですけど、異なる相関性を比べることって出来るんですか?
なんでもいいんですけど、例えばある地域においてAとBの関係を知りたいとしますよね。
で、10年前にそれを図ったら相関性がaと出たとします。
で、今年また同じ地域で相関性を図ったらbだったとします。
この時aとbは比べることが出来るんでしょうか。
(説明がわかりにくいかもしれないですけど)
607132人目の素数さん:2007/04/28(土) 01:33:26
互いに可換な巡回置換の集合ってどうやって見つけるのか?



{(123), (132)}
{ (1234), (13)(24), (1432).}
{ (12)(34), (13)(24), (14)(23).}
{(12)(35)(46) , (13)(26)(45), (14)(25)(36), (156)(243), (165)(234)}
608132人目の素数さん:2007/04/28(土) 01:41:53
>604
z消去。あとは普通の問題。
609132人目の素数さん:2007/04/28(土) 02:01:11
>>607
定義に忠実に
610132人目の素数さん:2007/04/28(土) 03:54:30
>>609
置換σと可換なのはσ^a (a=1,2,,,,n)のみでしょうか?
611132人目の素数さん:2007/04/28(土) 04:47:57
>>610
なんで定義に忠実に確認せずにそうやって手抜きを考えて肥溜めに嵌るの?
ウンコに塗れるのが趣味なの?
612132人目の素数さん:2007/04/28(土) 10:08:44
行列の乗算は
A*(B*C)=(A*B)*C
が成り立ちますか?
613132人目の素数さん:2007/04/28(土) 10:10:42
>>606
差があるといえるかどうかの検定はできるんじゃないの?
614KingOfUniverse ◆667la1PjK2 :2007/04/28(土) 10:18:38
talk:>>612 (a^{i}_{j}b^{j}_{k})c^{k}_{l}=a^{i}_{j}(b^{j}_{k}c^{k}_{l}) (アインシュタインの記法)。
615KingOfUniverse ◆667la1PjK2 :2007/04/28(土) 10:20:20
その和の公式を証明しないといけないのか。
(x_{1}x_{5}+x_{2}x_
616KingOfUniverse ◆667la1PjK2 :2007/04/28(土) 10:23:23
(x_{1}x_{5}+x_{2}x_{7})x_{9}+(x_{1}x_{6]+x_{2}x_{8})x_{11}=x_{1}(x_{5}x_{9}+x_{6}x_{11})+x_{2}(x_{7}x_{9}+x_{8}x_{11}).

人の脳を読む能力を悪用する奴を潰せ。
617132人目の素数さん:2007/04/28(土) 11:12:56
>>610
置換群の可換な部分群を探す
部分群として一元生成でないものも存在する
例えばクラインの四元数群(>>607の3番目)
618132人目の素数さん:2007/04/28(土) 11:20:42
>>611
須加吐露か・・・しばらくしてないお
619132人目の素数さん:2007/04/28(土) 11:53:59
>>607
中心化群=(群の数)/(共役類の数)
620132人目の素数さん:2007/04/28(土) 13:06:52
>>602
y=3x^2-ax-a-b = 3{x-(a/6)}^2 -(1/12)a^2-a-bだから
x=a/6が軸で、
3{x-(a/6)}^2≧0だから共有点を持つのは
-(1/12)a^2-a-b < 0
b > -(1/12)a^2-a = (-a^2 -12a)/12
のとき

共有点の一つが(1,0)だから
x=1,y=0を代入して
0=3-a-a-b
b=-2a+3

このとき
y=3x^2-ax-a-b =3x^2 -ax+a-3=(x-1)(3x-a+3)
でもう一つの共有点のx座標は x=(a-3)/3
-1≦(a-3)/3≦0
であれば
-3≦(a-3)≦0
0≦a≦3
621132人目の素数さん:2007/04/28(土) 16:28:29
xの2次方程式x^-(2a+1)x+a^=0が実数解をもつようなaの範囲を求めよ

>>581です
書き直しました。教えてください・・
622132人目の素数さん:2007/04/28(土) 16:30:32
判別式を使えばいい
623132人目の素数さん:2007/04/28(土) 16:30:59
>>621
全然直せてなくてワラタ
ほれ
http://science6.2ch.net/test/read.cgi/math/1177282598/3
624132人目の素数さん:2007/04/28(土) 16:31:41
書き直せてねー
625132人目の素数さん:2007/04/28(土) 16:34:01
>>621
判別式を使ってください
626132人目の素数さん:2007/04/28(土) 16:39:49
>>621
x^- はえっくすの上付きバー
a^=はえーの上付き二重バー
だよな、バーはどういう意味で使ってるの?
627132人目の素数さん:2007/04/28(土) 16:42:06
0°<θ<90°でsinθ-cosθ=√3/2のときのsinθ+cosθの求め方を教えてください。
628132人目の素数さん:2007/04/28(土) 16:43:15
>>627
符号に気をつけつつ、(x+y)^2=(x-y)^2=4xyを使う。
629132人目の素数さん:2007/04/28(土) 17:00:29
xの2次方程式x^2-(2a+1)*x+a^2=0が実数解をもつようなaの範囲を求めよ
こんな感じですか?
>>623さんありがとうございます
630132人目の素数さん:2007/04/28(土) 17:08:24
しつこいな、教科書嫁
631132人目の素数さん:2007/04/28(土) 17:09:54
>>629
もう回答でてるんだから手を動かした結果を見せろ。
632629:2007/04/28(土) 18:08:41
>>629です
判別式を用いて解いた結果、答えは
a≧-1/4になりました
あっているかな
633132人目の素数さん:2007/04/28(土) 18:09:34
>>629
死ね^-^
634132人目の素数さん:2007/04/28(土) 19:31:59
誹謗中傷は誹謗中傷版で
635132人目の素数さん:2007/04/28(土) 21:18:01
>>632
それでいいよ
636132人目の素数さん:2007/04/29(日) 06:34:36
おはようking
637132人目の素数さん:2007/05/01(火) 00:51:25
>>632
あっとる
638132人目の素数さん:2007/05/01(火) 01:02:27
ΔPQ=(ΔP)Q
を証明してください。
本当に分からなくて困ってます…
639132人目の素数さん:2007/05/01(火) 01:02:58
ΔPQ=(ΔP)Q
を証明してください。
本当に分からなくて困ってます…
640132人目の素数さん:2007/05/01(火) 01:03:50
1つ多く出してしまった…
641132人目の素数さん:2007/05/01(火) 01:05:29
実に分からない問題だなあ
642132人目の素数さん:2007/05/01(火) 01:06:02
うん、本当に分からないね
643132人目の素数さん:2007/05/01(火) 01:13:28
大学の線形代数の授業で…
644132人目の素数さん:2007/05/01(火) 01:15:03
君は、典型的な数学ができない日本人だな
645132人目の素数さん:2007/05/01(火) 01:15:28
問題くらい一字一句間違いなく写せバカ
646132人目の素数さん:2007/05/01(火) 01:19:30
>>639
Δ
P
Q

が何を表しているのか定義くらい書くように
647132人目の素数さん:2007/05/01(火) 01:20:18
ひょっとして


おでんマン?
648132人目の素数さん:2007/05/01(火) 06:13:08
オデンくん
649132人目の素数さん:2007/05/01(火) 07:48:08
a_n=1/2^n
S_n=1/2+1/4+1/8+1/16+・・・
で、∞までやったら、1・・・
ところで積分だと

f(x)=1/2^xを積分したらどうなる?
650132人目の素数さん:2007/05/01(火) 07:55:14
-2^(-x)/log2
651132人目の素数さん:2007/05/01(火) 10:08:16
x,yは実数、fは実数→実数への関数
fについて
すべての実数aについて、f(x+a,y+a)=f(x,y)、0以外の実数aについてf(a*x,y/a)=f(x,y)
こんな関数f(x,y)を考えたときf(x,y)=定数以外にf(x,y)はありますか?
あったらそんな関数の例を一つ出してください
652132人目の素数さん:2007/05/01(火) 10:19:50
ない
653132人目の素数さん:2007/05/01(火) 11:06:44
>>651
f(x+a,x+a)=f(x,y)より
u=(x-y)としてfはuで決まる関数f(u)

f(ax,y/a)=f(x,y)より
v=xyとしてfはvで決まる関数f(v)

uを決めるととu=(x-y)はy=xに平行な直線でfの値は一定
どんなuでもxy=1と交わりxy=1上でfの値は一定なので、
fは定数でなければならない。
654132人目の素数さん:2007/05/01(火) 13:34:05
コレスキー法を使うと連立方程式が解けるらしいのですが、全然意味がわかりません。
A=LL*に分解していったいどうしろというのですか?

ガウスの消去法で連立方程式を解くのは理解したのですが、コレスキー法とガウスの消去法
のつながりがまったくわからんのです。

http://encyclopedie-ja.snyke.com/articles/%E3%82%B3%E3%83%AC%E3%82%B9%E3%82%AD%E3%83%BC%E5%88%86%E8%A7%A3.html

どういうことかご教授頂たくおもいます
655132人目の素数さん:2007/05/01(火) 13:52:30
フィボナッチ数列の第n+1項と第n項の比(分母が第n+1項)が収束することの証明を教えて下さいm(__)m

第0項と第1項は正数です。
656132人目の素数さん:2007/05/01(火) 14:02:01
失礼しました。第n項が分母です。
657132人目の素数さん:2007/05/01(火) 14:04:21
3^(1/2)、7^(1/3),12^(1/4)の大小関係を比べよ、という問題なんですが。
それぞれを12乗して3^6,7^4,12^3としました。
これ以上簡単にできますか?
それともここで計算すべきですか?
658132人目の素数さん:2007/05/01(火) 14:19:37
>>654
ガウスの消去法というのは
Ax=bという方程式の係数を抜き出して

(A b)という行列を
(E c)という形に変換し
Ex=c
x=c
と求める方法

コレフスキー分解は
Ax=bから
(L^(-1)){(L^(-1))^*}x=b
{(L^(-1))^*}x = Lb
x = (L^*)Lb
659132人目の素数さん:2007/05/01(火) 14:23:57
>>654
リンク先の一行目に書いてあるとおり。
Cholesky分解は正定対称行列に対してのみ使えるが
ガウスの消去法は一般の行列に対して使える。
660132人目の素数さん:2007/05/01(火) 14:51:34
>>658,659
なるほど、
コレフスキー分解は
Ax=bから
(L^(-1)){(L^(-1))^*}x=b
{(L^(-1))^*}x = Lb
x = (L^*)Lb
この部分が欲しかったのですよ。なるほどなるほど
ありがとうございます
661132人目の素数さん:2007/05/01(火) 15:08:34
>>655
a(n+2)=a(n+1)+a(n)
a(n+2)-a(n+1) = a(n) >0 だもんで
狭義単調増加数列であり
a(n+2) = a(n+1)+a(n) < 2a(n+1)
0<a(n+2)/a(n+1) < 2

b(n) = a(n+1)/a(n)とおくと
b(n-1)<b(n) ならば b(n-1) < b(n+1) < b(n)
b(n)<b(n-1) ならば b(n)<b(n+2)<b(n-1)
が成り立つ。

b(1)<b(2)だとすると
b(1)< b(2)
b(1)<b(3)<b(2)
b(1)<b(3)<b(5)< … <b(6)<b(4)<b(2)
となり、それぞれ極限値を持つ。
b(1),b(3),b(5),…の極限をpとし
b(2),b(4),b(6),…の極限をqとすれば

a(n+2)=a(n+1)+a(n)から
a(n+2)/a(n+1) = 1+{a(n)/a(n+1)}
nの値によって、
p = 1+(1/q)
q = 1+(1/p)
の2つが得られる。
したがってp=qとなり同じ値に収束する。
662132人目の素数さん:2007/05/01(火) 15:11:27
>>655
フィボナッチ 黄金比でググるといろいろ出て来ると思うよ
663132人目の素数さん:2007/05/01(火) 20:31:00
>>655
 フィボナッチでググりますた。
 φ = (1+√5)/2 = 1.618033988749895…
とおくと、φ+(1/φ) = √5,
 F_n = (F_0/√5){φ^(n-1) -(-1/φ)^(n-1)} + (F_1/√5){φ^n -(-1/φ)^n},

http://mathworld.wolfram.com/FibonacciNumber.html
664132人目の素数さん:2007/05/01(火) 21:55:28

10種類のクーポン券があります。それぞれを取る確率は同じです。また、クーポン券の束は何回とっても無くなりません。
確率変数XはN枚クーポン券を取ったときにその中に含まれるクーポン券の種類の数です。
Xの期待値を求めよ。
665132人目の素数さん:2007/05/01(火) 22:00:52
次の不等式を証明せよ。ただし、nは2以上の自然数とする。
1+(-1/n)+1/n^2<1/1^2+1/2^2+1/3^2+・・・・・+1/n^2

白チャートの問題なんですがわかりません。
答えはhttp://imepita.jp/20070501/783430です

ゆえに・・って何で赤文字に変形がなりたつんですか
666132人目の素数さん:2007/05/01(火) 22:11:38
1/k^2は幅1高さ1/k^2の長方形の面積
667132人目の素数さん:2007/05/01(火) 22:14:31
>>665
写真不鮮明だけど、まず 1/x^2 < 1/k^2 と書いてあるんか?
そんなら両辺を領域 k < x < k+1 で定積分してごらん。
左辺 = ∫[k,k+1]dx/x^2
右辺 = ∫[k,k+1]dx/k^2 = (1/k^2)∫[k,k+1]dx = 1/k^2.
668667:2007/05/01(火) 22:17:59
>>665
ほんとうは 1/1^2 + 1/2^2 + … +1/n^2 はn→∞で 1.64くらいに
なる。1-(1/n)+1/n^2 は n→∞で 1にしかならない。あまりよい
不等式じゃないなあ。
669132人目の素数さん:2007/05/01(火) 22:20:49
>>666
>>667でわかりました。ありがとうございました。
二時間も無駄に考えたorz
670132人目の素数さん:2007/05/01(火) 23:12:23
なんか証明の必要性なんて欠片もないような自明な不等式に見えるんだが。
671667:2007/05/01(火) 23:37:05
たぶん次の方法でもこの不等式は証明できる。
S(n) = 1/1^2 + 1/2^2 + … + 1/n^2 とする。 S(n) = s(n-1)+1/n^2
である。ところで、S(n-1) > 1/(1・2) + 1/(2・3) + … + 1/((n-1)n)。
1/(k(k+1)) = 1/(k+1)-1/k より、上の右辺は1-1/n。
よって S(n) > 1-(1/n)+1/n^2。
672132人目の素数さん:2007/05/01(火) 23:43:14
左辺は1未満で右辺は1より大だと思うんだが、
本当にこんな問題が出てるのか?
673667:2007/05/01(火) 23:56:54
>>672
左辺が 1より小さくなる理由は、1/1^2 + … の初項 1 のかわりに、
>>671に示したように 1/(1・2) = 1/2 を使用したところにある。
よって、それを補正して、
1.5 - (1/n) + (1/n^2) < 1/1^2 + 1/2^2 + … + 1/n^2 も証明
できるね。これだと少しは意味のある不等式になるかな?
(オイラーが ζ(2) = (π^2)/6 を証明したときは、準備として
1/1^2 + 1/2^2 + … を事前に何10桁だかの精度で計算して、
アタリをつけておいたと聞いた。)
674132人目の素数さん:2007/05/02(水) 00:00:57
少なくとも、おいらはそうやった
675132人目の素数さん:2007/05/02(水) 00:12:13
>>664
n回目の試行時にm種類のクーポンを持っている確率を
Pm(n)とおいて、10個の漸化式を解けばよい
676132人目の素数さん:2007/05/02(水) 00:43:42
この問題が分かりません。どうアプローチすればいいのでしょうか。

R^3のベクトル a1,a2,a3,a4 について、a1,a2,a3は一次独立でいずれもa4と異なる。
さらに、a1〜a4は原点を通るある平面に関して、その片側にだけ位置すると仮定する。
FはR^3の一次変換(線形写像)で、
F(ak)=a(k+1) (K=1,2,3), F(a4)=a1 (※k+1はaの添字です)
を満たすとする。このとき、F(a3)をa1,a2,a3の一次結合の形に表せ。
677132人目の素数さん:2007/05/02(水) 00:48:12
タクシードライバーになりすまして、レス618の問題を軽く問いていただけませんか?よろしくお願いします!
http://c-others.2ch.net/test/-/traf/1172962646/617-
678132人目の素数さん:2007/05/02(水) 01:00:24
1/5
679132人目の素数さん:2007/05/02(水) 01:08:46
>>676
Fは4回適用すると元に戻る
680132人目の素数さん:2007/05/02(水) 01:14:54
F(a[3])=a[4]=sa[1]+ta[2]+ua[3]とおいて
両辺にFを何度かほどこしてs,t,uの関係式を導く。Fの線形性に注意
681132人目の素数さん:2007/05/02(水) 01:25:30
>>675
ありがとうございます。おかげでできましたm(__)m
682132人目の素数さん:2007/05/02(水) 02:13:25
{n^(m+1)}/(m+1)=176となる
整数(m,n)を一組求めよ。お願いします。
683132人目の素数さん:2007/05/02(水) 02:28:27
(0,176)
684132人目の素数さん:2007/05/02(水) 02:29:27
>>682
m=0のとき、どうなる?
685132人目の素数さん:2007/05/02(水) 02:35:32
                      ,. ―‐- 、
                   /´`       ヽ、
                      ,'        、} ト.
.    rr、                ,'    '   ,.イ゙V }
  ,イ川、             ,' .!  ,' .i  〈_,イ「l/        フ ヽヽ         /
  |  !」_          / ,'::! .!,': ::! .:.:| ハ〉|     / J ヽ     ̄ ̄ ̄   Λ_丿
  l  //```ヽ、       l:i::!::! :::l:::l :::l ::::,`:!:i::|
  `ーl_{     ``丶、__ _从{::|:::::l::,':::::| ::::,':::'l::l::!                 ,.┐
     \_          `/ Λ!::/:ノ!::/!::ノ::/ノノノ                 /フ′
       `¨` ー- 、_   / /  ´ ´ ''´ '´7´/´ ̄`¨¨`¬……――''"´`>ーァ'´,イ__
             \! ..___    / /               / /  '´,.-┘
                 ...___   ̄´ /     ___ ___      {___j--‐'´
              |      ̄ ̄´ ´厂 ̄´        ̄`¨¨´
                /         ノ'´
                }___、      |
            /     ``¬―v‐'
            /        ,.   |
          //        /     !
         //      ,'      '、
        / /l         !        ヽ
        \,'|         !          >
686682:2007/05/02(水) 02:38:05
mは5以上でおねがいします。
687132人目の素数さん:2007/05/02(水) 02:40:49
>>686
後出し野郎氏ね
688132人目の素数さん:2007/05/02(水) 02:43:39
>>687
176=2^4*11
以下略。
689682:2007/05/02(水) 03:04:47
>>688
解なしということ?
690132人目の素数さん:2007/05/02(水) 03:06:16
その根拠を書け屑
691132人目の素数さん:2007/05/02(水) 06:58:06
申し訳ありません,二問あるのですが
質問させてください.

1.
DをCの部分集合の空でない集まりと仮定する(C:標本空間).
事象の全体
B=∩{ε:D⊂ε かつ εはσ集合体}
を考える.Φ∈Bであることに注意せよ.なぜなら,Φはそれぞれのσ集合体
に含まれ,したがって,とくにこれは各々のσ集合体ε⊃Dに含まれるからである.
同様に続けてBがσ集合体であることを示せ.

2.
C=Rとする.ここにRはすべての実数の集合である.また,IをRのすべての開区間とする.
いま,実数直線状のボレルσ集合体を考える.つまり
B0=∩{ε:I⊂ε かつ εはσ集合体}
によって与えられるσ集合体B0である.定義より,B0は開区間を含む.
[a,∞)=(-∞,a)^c であり,B0は補集合について閉じているので,a∈Rについて
[a,∞)の形式のすべての区間を含むことになる.このようにして,B0が実数上の
すべての閉区間と半開区間を含むことを示せ.

多くて申し訳ありませんが,よろしくお願いいたします.
692132人目の素数さん:2007/05/02(水) 07:41:33
あと何を示せばいいかな、仮定から何がわかるかな
693132人目の素数さん:2007/05/02(水) 10:53:50
B=∩{ε:D⊂ε かつ εはσ集合体}の意味と、σ集合体(完全加法族)の定義わかってますか?
 A∈B⇔D⊂εなるすべてのσ集合体εに対して,A∈ε
これをhintに(ii)A∈B⇒A^c∈B(iii)A[1],…A[i],…∈B⇒∪A[i]∈Bを示せばよい。
694132人目の素数さん:2007/05/02(水) 11:04:03
http://imepita.jp/20070502/375590

結論を教えて下さい!
695132人目の素数さん:2007/05/02(水) 11:12:38
上の図、よく見ると斜辺が一本の直線じゃないんだよな
696132人目の素数さん:2007/05/02(水) 11:16:12
そうなんですか!?
697132人目の素数さん:2007/05/02(水) 11:34:07
698132人目の素数さん:2007/05/02(水) 11:53:37
>>697
ありがとうございます。おかげで良く分かりました!
699132人目の素人さん:2007/05/02(水) 12:07:55
こんにちは、どうしてもわからないので質問があります。
三軸相貫円柱(先生が見た目から命名しただけで、通称が3つの円柱を
真ん中辺で垂直に交わらせた相貫体というらしいです。)の積分による
体積の求め方がわかりません。
説明の仕方が下手で申し訳ないのですが、わかる方いたらレスお願いしますorz
700132人目の素数さん:2007/05/02(水) 12:08:21
教科書よめばいいじゃん
701132人目の素数さん:2007/05/02(水) 12:27:04
X-1、X+1、X+3が鈍角三角形になるための必要十分条件は[ア]<X<[イ] が成り立つことである


お願いします
702132人目の素人さん:2007/05/02(水) 12:28:22
教科書とかにはなくて、先生の気まぐれで思いついた図形らしくネットとかにも
ありませんでしたorz 自分で図形を作るも、重なっている部分の求め方が
イメージできずにもとめることができません・・。
ちなみに、図にするとしたのような感じです、下手でわかりにくいですがorz

  ○_
 _|| )_
(_/ /_()
○|
703132人目の素人さん:2007/05/02(水) 12:29:38
  ○_
 _|| )_
(_/ /_() ずれました…こっちです
○|
704132人目の素人さん:2007/05/02(水) 12:30:56
  ○_
 _|| )_
(_/ /_() 完全にてんぱりましたorz どなたかお願いします(;´д`)
  ○|
 U
705132人目の素数さん:2007/05/02(水) 12:36:54
CADも使えないのか
706132人目の素数さん:2007/05/02(水) 12:43:16
>>699東大の過去問だよそれ
707132人目の素数さん:2007/05/02(水) 12:47:43
3X^2 + 4xy + 3y^2 =1 のとき、x^2 + y^2 の最大値・最小値は?

この問題の答え教えてください。お願いします
708132人目の素数さん:2007/05/02(水) 12:49:32
>>707
> 3X^2 + 4xy + 3y^2 =1
が表す図形は?
709132人目の素人さん:2007/05/02(水) 12:57:55
>>705高校なんでorz
>>706mjd!?
710706:2007/05/02(水) 13:11:18
>>699
とりあえず解等の概略だけ。
xyz空間内で領域X:x^2+y^2≦1,y^2+z^2≦1,z^2+x^2≦1の体積を考えるが
これは第1象限〜第8象限についてそれぞれ対称なので、第1象限に制限した領域X1の体積を8倍すればよい
z=c(0≦c≦1)を通る平面でX1を輪切りにした断面A(c)を考えると
円x^2+y^2=1と直線x=√(1-c^2),y=√(1-c^2)で囲まれた領域になる
特にc≧1/√2のときA(c)は半径2√(1-c^2)の正方形になる。
したがって∫[1/√2,1]A[c]dcはX1のz≧1/√2における領域Azの体積になる
同様にz=a,y=bでの断面を考え、領域Azと合同な図形Ax,Ayを得る
特にXからAx,Ay,Azを取り除いた図形は直径1/√2の立方体になる。
つまりXの体積=(1/√2)^3+3(Azの体積)
711132人目の素数さん:2007/05/02(水) 13:12:03
712711:2007/05/02(水) 13:13:37
出された問題とは若干違う(東大のは2本の円柱の内部かつ他の円柱の「外」部)が
ほぼ同じことなので大分参考になるだろう
713132人目の素数さん:2007/05/02(水) 13:23:14
∫√(a-bsin(2φ))dφ  全部√の中です。
これなんですが調べても解けませぬorz
よろしくお願いします
714132人目の素数さん:2007/05/02(水) 13:25:45
教えてください。
527を割ると23余り、436を割ると16余る自然数のうちで、
最小の数の1の位はいくつか。

自然数をNとすると、
527-23=504
436-16=420

は、ともにNで割りきれます。
従ってNはこれらの公約数、つまり
2×2×3×7の約数で
しかも23より大きくて最小のもの
ということになり、
それは2×2×7=28で、
1の位は8です。


と解答にあるのですが、どうして2×2×7になるのですか?

715132人目の素人さん:2007/05/02(水) 13:27:48
>>710-712ありがとうございます!
まさか東大の問題とは夢にも思いませんでした・・・。
これから連休なので頑張ってみます。
716132人目の素数さん:2007/05/02(水) 13:28:45
なんらかの関数f(x)があって、f(x)はxが有理数だけに定義されているとします。
たとえばf(1)=1,f(a+b)=f(a)*f(b),f(x)>0
この定義の仕方だと、f(2)とかf(100)とかf(1/7)とかわかるけど、f(√2)とかf(π)とか絶対わからないよね?
xが有理数でない場合、↑の与えられた条件だけでは絶対にf(x)はもとまらないってことを証明してみてください。
717132人目の素数さん:2007/05/02(水) 13:29:23
演習 大学院入試問題 [数学]T <第 2 版> の、P.20 例題 1.4 の (1) で、
  (m, n) 型行列 A と (n, l) 型行列 B について、rank(AB) <= rank(B) を示せ
という問題の解答について質問があります。

解答は、
  AB の各行ベクトルは、B の各行ベクトルの線形結合であるから、
  rank(AB) = rank { AB の各行ベクトル } <= rank { B の各行ベクトル } = rank(B)
となっています。

ここで、rank { AB の各行ベクトル } や、rank { B の各行ベクトル } というのは、
線形独立な行ベクトルの個数、を意味していると考えてよいのでしょうか?
718132人目の素数さん:2007/05/02(水) 13:49:23
>>716
定義域の外でどうなろうが知ったことか。Q.E.D.
719132人目の素数さん:2007/05/02(水) 13:51:39
>>717
なんか気持ち悪い表現だが、たぶんそういう意味だろう。
720132人目の素数さん:2007/05/02(水) 14:04:16
>>718
証明になってませんよw
勝手に定義域を決めないでください。定義域xは実数です
f(1)=1,f(a+b)=f(a)*f(b),f(x)>0
これではxが無理数の場所ではfはもとまらないことを証明してください
721132人目の素数さん:2007/05/02(水) 14:08:05
>>720
>>716
>f(x)はxが有理数だけに定義されているとします
有理数っていてるじゃないか
722132人目の素数さん:2007/05/02(水) 14:08:55
ワロスw
723132人目の素数さん:2007/05/02(水) 14:16:42
>>721
だからそれが本当に有理数だけに定義されてるかってことを証明しろつってんだが。
まぁもういいや。別のスレで聞いて見ます。バイバイノシ
724132人目の素数さん:2007/05/02(水) 14:27:28
>723
だめだああああああ
よそにいってはイカン
ここで遊んでてくれ
725132人目の素数さん:2007/05/02(水) 14:36:20
本当に有理数だけに定義されてるか

本当に有理数だけに成り立つか

だな、定義の意味しらんのかい・
726132人目の素数さん:2007/05/02(水) 14:36:42
>>723
関数の定義と定義域というものを誤解していないかい?
ま、それはよいとして、例えば次の集合 X の濃度をもとめてみな。
X={f: f は 実数全体で定義された実数値関数で、次をみたす f(1)=1、任意の有理数a、bに対してf(a+b)=f(a)*f(b) }
727132人目の素数さん:2007/05/02(水) 15:58:45
>>711
10年くらい前にも似たような問題出してたよなぁ
728132人目の素数さん:2007/05/02(水) 16:01:27
>>716
条件がなければ
定まらないというか
無理数の所は自由に決めて良いよ。
f(√2)=1でもf(√2)=√2でもいい。

普通はfに連続などの条件をつけてf(a+b)=f(a)*f(b)みたいな定義をする。
729132人目の素数さん:2007/05/02(水) 16:25:38
>>713
>全部√の中です。
√dφ ってなに?
730132人目の素数さん:2007/05/02(水) 16:42:16
>>707
x^2+y^2=1-2(x+y)^2≦1
5(x^2+y^2)=1+2(x-y)^2≧1
より、1/5≦x^2+y^2≦1。
731709です:2007/05/02(水) 17:55:39
まだ時間があまりたっていないのに諦めるのは申し訳ないのですが、3本の場合の
体積がどうしてもわかりません…どなたか丁寧に教えていただけませんかorz
732132人目の素数さん:2007/05/02(水) 18:46:45
>>717
{ AB の各行ベクトル }ってのは
ABの各行ベクトルa[1],a[2],…,a[n]で張られるベクトル空間[a[1],…,a[n]]って意味で
別にa[1],…,a[n]が線形独立じゃなくっても意味を持つ。
733132人目の素数さん:2007/05/02(水) 18:48:10
>>731
どこまでできた?
734132人目の素数さん:2007/05/02(水) 18:53:27
先輩から変な問題出されてしまいました。
僕にはどこから手を付けていいかわかりません。
なにとぞ、お願いいたします。


ゴルフボールの飛距離は以下の式によるという。

y=1/G*(F/M)^2*sin(2θ)

  但し、 y: 飛距離 (m) 
       G: 重力加速度  9.8 (m/s^2)
       F: 打上時の力   ??? (N)
       M: ボールの質量 0.05 (kg)
       θ: 打上時の仰角 ??? (rad)

仰角θは0度から45度の間で、Fは0から20Nの間で設定できる。

ティーからグリーン上のカップまでの距離: y= 500mであるとき、
ホールインワンさせるために最適なF,θを決定せよ。




     


735132人目の素数さん:2007/05/02(水) 18:55:36
{(δ^2)z}/(δxδy)って
z_(xy)のことk?
736132人目の素数さん:2007/05/02(水) 18:59:13
>>734
とりあえず、その変な問題とやらを、なにがどういう意味で変なのかを添えて書き込んでくれ。
737132人目の素数さん:2007/05/02(水) 19:02:19
問:
平面上に、円O1: x^2+y^2=r^2、円O2: (x-r)^2+y^2=r^2をとる。
円O1,O2の交点をA,Bとし 円O1上の定点C=(-r,0)をとる。
円C2の上でかつ円C1の内部にある点Mをとる。
線分MAの長さをα,MBの長さをβとする。

ベクトルの線形結合でMC=a・MA+b・MB となる実数a,bは一意に定まるが、このとき
a,bをα,βを用いて(rは用いないで)表せ。

----
自分で考えてみたんですが、よくわかりません。
ABCはO1に内接する正三角形で、R(r,0)とおくとx軸とRMのなす角をθとしたとき
M(r-rcosθ, rsinθ) ですが、φ=θ/2 とおくとx軸とOMのなす角はφであり、
かつ M(rcos^2(φ)、rcosφsinφ)でもある、α=2sin(φ-π/3), β=-2sin(φ-2π/3)である、
など手がかりっぽいことは分かっています。
角BAOは2π/3だから余弦定理から、α^2+β^2+αβ=r^2になるので、
a,bをα,β,rで表せればrを消すことはできるでしょう。
ただ、MC=aMA+bMBに直接Mを入れてa,bの連立方程式を作ってもθやφが入ってしまいますし、
それをαやβに帰着させるのがなかなか難しいです…。
738132人目の素数さん:2007/05/02(水) 19:12:44
>>734

どもです。
先輩が変な人なんです。
(この間は飲み会でさんざん酔っ払って公園のホームレスと意気投合してビートルズのレットイットビーを歌ってました。)

だから、問題もたぶん変なんだろうと思いました。

739sage:2007/05/02(水) 19:13:35
>>701
一番大きい角が一番長い辺X+3に対応する。
第2余弦定理より
 cos(長辺に対する角) = {(X-1)^2 + (X+1)^2 -(X+3)^2}/{2(X-1)(X+1)} <0,
また、X-1>0,
これから 1<X<7.

>>707,730
 x^2 + y^2 = (3x^2 +4xy +3y^2) -2(x+y)^2 ≦ 3x^2 +4xy +3y^2,
 x^2 + y^2 = {(3x^2 +4xy +3y^2) +2(x-y)^2}/5 ≧ (3x^2 +4xy +3y^2)/5.

>>708
 左辺 = 3x^2 +4xy +3y^2 = 5u^2 + v^2, u=(x+y)/√2, v=(x-y)/√2.
740731です:2007/05/02(水) 19:17:36
>>733東大のものに関してはなんとなくですが理解できたのですが、3軸のほうは
もう根本的にわからなくて…数学は苦手なのでお手上げ状態になってます(ーー;)
741132人目の素数さん:2007/05/02(水) 19:25:58
まずはじめにお詫び致します。
スレ違いな質問内容ですみません。
でも専用スレがないので一番人が集まるところで聞いてみるのが一番だと思いここにいたりました。
すみません。

暗算能力を向上させたいです。
まずは6桁同士の四則演算が目標です。
とりあえず筆算式暗算を少し改良して4桁同士の加減と4の2の乗算まではある程度できるようになってきました。
しかし除算やこれ以上の桁数増に今の方法では対応できないと感じています。
そこでそろばん式の暗算を知りました。フラッシュ暗算もそろばん式だったようで。
一応そろばんやったことあります。頭にイメージすることはできます。
加減くらいしかできませんが。それも久しぶりなので練習が必要です。
乗除算についてはこれから勉強しようかと。
それで実際にそろばん式暗算を得意とする方に質問です。

6桁同士の乗除算(除算は小数点第6位くらいで四捨五入)をそろばん式暗算で出来る方はいますか?

23535 * 95722 / 74 / 1928 + 18222 * 24

このレベルをできる方は実際にいますでしょうか?
僕自信が出来るようになるかは別として、実際に存在しているのか、可能なものなのか知りたいです。
スレ違いですみません。どうぞ宜しくお願いします。
742734:2007/05/02(水) 19:30:38
>>736 様

734です。
>>738は >>736様向けのレスでした。

方針としては、未知数2つだから連立方程式をたてるんだと思いますが、
与えられた式が1つしかありません。

F,θの誤差:ΔF,Δθを式の中に追加するときの変化量をΔyとして、
Δyが最小になるようなF,θを求めればいいのかな?














743132人目の素数さん:2007/05/02(水) 19:57:10
>>741
死ね
744132人目の素数さん:2007/05/02(水) 20:31:33
>>740
710の方針で3円柱の共通部分の体積Vは求められるから
三軸相貫円柱の体積は
(円柱の体積)×3
から、重複して足し合わせたぶん、つまりV×?を引く。
745132人目の素数さん:2007/05/02(水) 21:03:59
>>737
MA↑=r((-1/2)+cosθ, ((√3)/2)-sinθ)
α^2=r^2*(2-cosθ-(√3)sinθ)=r^2*(2-2cos(θ-π/3))=4*r^2*sin(θ/2-π/6)
よりα=-2rsin(θ/2-π/6)
同様に計算してβ=2rsin(θ/2+π/6)
この二式と二乗の和=1からcos(θ/2), sin(θ/2)をα,βで表す
a,bの連立方程式からa,bをθで表してθ→θ/2→α,β
で出来そう
746132人目の素数さん:2007/05/02(水) 21:43:01
>>719, >>732
答えていただき、ありがとうございます。

二人のレスを読んで、もう一度よく考えてみて、ある行列の行ベクトル (列ベクトルでもいい(はず))
のうち互いに線形独立となるものでその個数が最大となるものは基底ベクトルであり、
この基底ベクトルの個数が rank だということを思い出し、ようやく解答の意味が分かりました。

AB の各行ベクトル a[1],a[2],...,a[m] はそれぞれ、B の各行ベクトル b[1],b[2],...,b[n]
の<線形結合>であるため、AB の基底ベクトルの数が B の基底ベクトルの数を超えるわけがない、という
ことだと。 むしろ、A によっては基底ベクトルの数が減る可能性があるため、rank AB <= rank B
となる、ということですね。

しかし、{ AB の各行ベクトル } ってわざわざ書く意味はよく分かりませんが。
>>732 さんの言う「ベクトル空間である」ということを強調したかったのですかね。

とにかく、ありがとうございました。
747132人目の素数さん:2007/05/02(水) 22:00:14
>>745
ありがとうございます。
どうもこちらに計算ミスがあるようですね。考え直してみます。
748132人目の素数さん:2007/05/02(水) 22:11:19
>>746
確かに。正確にはdim{a[1],…,a[n]}とでもするべきでしょうね。
749740です:2007/05/02(水) 23:13:50
>>744共通部分の面積が…(T_T)
750132人目の素数さん:2007/05/02(水) 23:45:04
>>749
断面さえイメージできれば全体の形は気にしなくていい。
z=cを通るxy軸と平行な平面と、3円柱がどう交わるか考える。
ttp://www.imgup.org/iup375079.jpg
751132人目の素数さん:2007/05/03(木) 00:04:25
3^πとπ^3ではどちらが大きいか。
理由も答えよ。

という問題がレポート課題として出てしまいました。

いったいどうやればよいのでしょうか。

大小比較だからきっと3^π−π^3の正負を調べればよいのでしょうが。。
どなたかわかる方がいらっしゃれば教えてください。
お願いします。
752132人目の素数さん:2007/05/03(木) 00:09:20
πlog3 と3logπの比較、これをさらに
(log3)/3と(logπ)/πの比較と考えれば
一つの関数を考えればいい事になるな
753132人目の素数さん:2007/05/03(木) 00:36:00
>>749
東大のが分かったんなら同じようにやるだけだが
それができないということは東大の方も分かっていない
754132人目の素数さん:2007/05/03(木) 03:36:45
>>745
その方法でできました。
連立方程式はけっこう複雑でしたけど、解いてみると最終的に
a=-β/α, b=-α/βというシンプルな形になったので、
もっとエレガントな解法もあるかもしれませんね。
755132人目の素数さん:2007/05/03(木) 06:07:39
AMとBCの交点をD, BMとACの交点をEとする。
∠AMB=120°であることから、∠CAD=∠ABMで
CD:DB=△ACD:△ABD=sin∠CAD:sin∠BAD
=sin∠ABM:sin∠BAM=α:β=α^2:αβ
同様にCE:AE=β:α=β^2:αβ
よってβ^2MA↑+α^2MB↑+αβMC↑=0↑

エレガントでもないか
756132人目の素数さん:2007/05/03(木) 08:13:01
AAABCDのEつを一列に並べる
二つ以上が隣り合う並べ方は何通り?
757132人目の素数さん:2007/05/03(木) 09:37:44
6面体のサイコロを転がした時、1の数字が出る確率が常に1/6だとします。
そしてその1が出る事を当たりといいます。
この時20回転がして3回当たる確率を求めなさい。

こんな感じの問題だったのですが、
1/6^3*5/6^17
としたら×でした。
どこかのサイトの問題だったのですが行き方がわからなくなり答えもわかりません。

確かに自分のやり方では1/6が3連続で当たりその後5/6が17連続出る確率でしかないんですよね。
どうやればいいでしょうか?
758757:2007/05/03(木) 10:12:29
う〜んさすがに朝では人がいないのですね。
あれから考えてるんですけど、1/6とか5/6だと計算できないので
0.2と0.8でやってみることにしました。
又回数をとても減らしました。

0.2^2*0.8^3 = 0.2*0.3*0.3*0.2*0.3 = 0.00108
ですよね。
乗算は累乗でも結局順番の入れ替えが可能なものですよね。
そうなると先ほどの答えは

(1/6)^3*(5/6)^17
で正解だと思うのですがどうでしょうか?添削お願いします。
もしかしてさっきは括弧がなかったらダメだったのかな・・・
なんかフラッシュサイトで10択くらいあってそのうちの一つを選んだら
×が出てきてその次の瞬間IE7のエラーとかで画面が落ちちゃったんです。

どうでしょう。あってますよね?
759132人目の素数さん:2007/05/03(木) 10:14:42
(1/6)^3*(5/6)^17*C(20,3)
760757:2007/05/03(木) 10:18:32
>>759
ありがとうございます!
その
*C(20,3)というのはなんでしょうか?初めてみます。
761132人目の素数さん:2007/05/03(木) 10:21:32
普通は20C3みたいに書くけど
C(n,r)=n!/(r!(n-r!)) のこと
762757:2007/05/03(木) 10:41:20
>>761
全然わかんないですorz
階乗記号だけわかりましたが。
wikipediaで調べて見る場合には何の項目を見ればいいでしょうか?
又、中学生に理解できることなのでしょうか?

ちなみにその*C(20,3)があるのとないのとではどれくらい答えに違いがでるのでしょうか?
もうまるで違う答えになってしまうんでしょうか?
763132人目の素数さん:2007/05/03(木) 10:42:31
>>760
組み合わせでググレ
764132人目の素数さん:2007/05/03(木) 10:43:33
>>762
全く違う答えになる
具体的には1140倍になる
765132人目の素数さん:2007/05/03(木) 10:47:19
五分って何ミリ?
766132人目の素数さん:2007/05/03(木) 10:50:47
767132人目の素数さん:2007/05/03(木) 10:51:41
768132人目の素数さん:2007/05/03(木) 10:52:42
>>762
wikipediaなんか信用しちゃいけないよ
内容が滅茶苦茶なこと多いよ
769132人目の素数さん:2007/05/03(木) 10:58:45
>>762
wikipediaなら↓のページを見ればいいよ
http://en.wikipedia.org/wiki/Combination
770132人目の素数さん:2007/05/03(木) 13:57:58
質問です。
「5^n−1が61の倍数になる、最小の正整数nを求めよ(答え:30)」

この問題の解き方を教えてください。フェルマーの小定理を使うらしいんですが…
771KingOfUniverse ◆667la1PjK2 :2007/05/03(木) 14:00:31
talk:>>770 1,2,3,4,5,6,10,12,15,20,30,60 のいずれかになる。
772132人目の素数さん:2007/05/03(木) 14:16:00
多分基本なんですが・・・
1/{sin(x)+1} の不定積分のやり方を教えてくださいm(_ _)m
773132人目の素数さん:2007/05/03(木) 14:17:22
>>772
分母子に1-sinxをかける
774132人目の素数さん:2007/05/03(木) 14:18:03
半角公式でもいいな
775132人目の素数さん:2007/05/03(木) 14:20:36
>>773,774
ありがとうございます!ホントに基本でしたね・・・
776132人目の素数さん:2007/05/03(木) 14:26:25
>>771
解き方は?
777132人目の素数さん:2007/05/03(木) 14:37:40
微分方程式 dy/dx = xe^(x^2 + y^2)/y の一般解を求めよ。

この問題の y=0 のときの議論がよく分かりません・・・
どなたか解説していただけないでしょうか?
778132人目の素数さん:2007/05/03(木) 15:00:43
1/(2x^2+3) って、どうやって積分すればよいのでしょうか・・
779132人目の素数さん:2007/05/03(木) 15:06:04
>>776
60の約数の中からしらみつぶし。
780132人目の素数さん:2007/05/03(木) 15:10:09
>>778
(√2)x=(√3)(tan t) とでもおいてみ
781132人目の素数さん:2007/05/03(木) 15:11:51
>>780
ありがとうございます!
782132人目の素数さん:2007/05/03(木) 15:29:52
>>777
dy/dx=(x/y)・e^(x^2+y^2) ?
なら、
e^(x^2)+e^(-y^2)=C となるが、
は、x軸、y軸に対称だから、
dy/dxはy=0で無限大(図をかくと、接線がy軸に平行)
783132人目の素数さん:2007/05/03(木) 15:29:18
以前コレスキー分解について質問した者ですが、更に質問があります

http://ja.wikipedia.org/wiki/%E3%82%B3%E3%83%AC%E3%82%B9%E3%82%AD%E3%83%BC%E5%88%86%E8%A7%A3

これの中のコレスキー Banachiewicz 法で
L(i,j)を出すのにルートを使ってるんですが、ルートの中がマイナスになる事が
多々あるんすよ。これって虚数を使えって事なんすか?あるいは解ナシってやつっすか?
784132人目の素数さん:2007/05/03(木) 15:54:40
ほとんど内容読んでないけど、ありそうなのは元の行列が正定値行列じゃなかったとかいうオチだな。
785132人目の素数さん:2007/05/03(木) 16:05:26
>>779
なんで、60の約数に限定されるかがわかりません…
786132人目の素数さん:2007/05/03(木) 16:21:25
>>782

y≠0 のときは e^(x^2)+e^(-y^2)=C
y=0 は解にならない
ということでいいんでしょうか?
787132人目の素数さん:2007/05/03(木) 16:42:04
>>784
つまり、ルートの中が虚数になるなら、正定値行列とも言えるわけですか?
788132人目の素数さん:2007/05/03(木) 16:50:36
>>786
61は素数なので、{1,2,‥,60}はmod61での乗法に関して位数60の巡回群になるから。
789KingOfUniverse ◆667la1PjK2 :2007/05/03(木) 16:50:40
talk:>>785 それでは、60の約数でなかったらどうなるか?
790132人目の素数さん:2007/05/03(木) 16:57:01
>>788-789
わかりました。ありがとうございます!
791132人目の素数さん:2007/05/03(木) 17:42:53
>>786
えーと勘違いしていたようです。

e^(x^2)+e^(-y^2)=C

のグラフをCの値で場合分けしてみると、
C=1    x=0,y=±∞
1<C≦2 y軸上の2点を頂点とする対称図形、( 0, ±√(1/log(C-1)) )、x=±√(logC)が漸近線
       (C=2のとき、頂点が(0,0))
2<C   x軸上の2点を頂点とする対称図形、(±√(log(C-1),0)が頂点、x=±√(logC)が漸近線

2≦Cのときは、y=0も含まれるはずです。
792KingOfUniverse ◆667la1PjK2 :2007/05/03(木) 17:47:34
人の脳を読む能力を悪用する奴を潰せ。
793132人目の素数さん:2007/05/03(木) 17:58:21
>>787
バーカバーカ。つかウィキペディアみたいな嘘満載のウェブサイトじゃなくて
もっとまともな本を買って読め、ゴミカス。
794132人目の素数さん:2007/05/03(木) 18:11:08
wikipediaで数学を勉強するとか(笑)
795757:2007/05/03(木) 18:47:52
http://ja.wikipedia.org/wiki/%E7%B5%84%E5%90%88%E3%81%9B
先ほど教えていただきました。組み合わせの件ですが、
一つだけ回答お願いします。
上記のページで

nCm = ??? = n!/m!(n-m)!
の真ん中の???のとこですが、・・・というテンテンテンという記号が出てるんですけど、
あれはなんでしょうか?

分母で言えば
m(m-1)・・・1
というのは
0.00000000000001から1の全てを分母に取るみたいなことですかね?
そんなの初めてなんですけども。
つまり
1/1・・・5
という表記もありえ、日本語的にあらわすと
1から5分の1というとてもあいまいな数字になるわけですか?
796132人目の素数さん:2007/05/03(木) 18:51:04
>>795
あなたの都合に合わせて変化する素晴らしい発明です
797132人目の素数さん:2007/05/03(木) 18:51:22
>>795
m(m-1)・・・1はm以下の整数を全部かけるってこと
たとえば
5!=5*4*・・・*1=120
798132人目の素数さん:2007/05/03(木) 18:54:02
>>795
なんで日本語版より判りやすく纏まってる英語版のほうのURLを教えたのに
わかりにくい日本語版なんか読んでるんだ?
数式に英語も日本語も無いことぐらいわかるよな?
799132人目の素数さん:2007/05/03(木) 18:54:21
いろんな疑問があるもんだなあ
800132人目の素数さん:2007/05/03(木) 18:57:35
中学生には英語版読むのはつらいんじゃね
801132人目の素数さん:2007/05/03(木) 19:02:10
>>800
> {n \choose k} = \frac { ( n - 0 ) }{ (k - 0) } \times \frac { ( n - 1 ) }{ (k - 1) } \times \frac { ( n - 2 ) }{ (k - 2) } \times \frac { ( n - 3 ) }{ (k - 3) } \times \cdots \times \frac { ( n - (k - 1) ) }{ (k - (k - 1)) }.

という数式が画像で書いてあるのは日本語だろうと英語だろうと変わらないぞ
802132人目の素数さん:2007/05/03(木) 19:05:24
事実を総括的に概説するだけの百科事典ウィキペディアで
勉強ができるとか思ってるアホ中学生は死ね。
ウィキペディアの質は百科事典の中でも最低レベルだし、
そもそも勉強したいならそれようの教科書として書かれた本を買え。
803132人目の素数さん:2007/05/03(木) 19:10:29
wikipediaって何のためにあるのか良く分からないんだよなあ
ちょっと調べるために見るにしても本当に表面的な事しか書いてない。その上信頼性が高いわけでもない
何なのあれ?
804132人目の素数さん:2007/05/03(木) 19:12:42
ちょっと調べるために見るなら表面的なことだけで十分だろ
信頼性が高いとも限らないからちょっと調べるためだけに見るんだろ
大概のものが載ってるからちょっと調べたいときに便利なんだろ
805132人目の素数さん:2007/05/03(木) 19:16:05
「表面的でその上信頼性が高くもない事柄をちょっと調べて大量に身にまとう」
ことに対する需要と評価は、社会生活を送る上では非常に高い。
いわゆるウンチくって奴か。俺は嫌いだがね。
806132人目の素数さん:2007/05/03(木) 19:16:36
>>803
調べ物の足がかり、ほんとにほんとの始めの一歩を踏み出すために
同様の書式で情報が集まってるという点だけに意味がある。
ウィキペディアで調べ物が全部済むとおもって裏を取ることをしないやつは
妖怪人間だけ。
807132人目の素数さん:2007/05/03(木) 19:17:20
>>803は想像力が足りない馬鹿
808132人目の素数さん:2007/05/03(木) 19:17:21
>>805
おれ、ウンチ喰っておまえになる。
809132人目の素数さん:2007/05/03(木) 19:22:19
2chってそんなに信頼性高い所だったのか
810132人目の素数さん:2007/05/03(木) 19:25:46
は?
811132人目の素数さん:2007/05/03(木) 20:08:50
>>809
どこにそんなこと書いてあんの?
812132人目の素数さん:2007/05/03(木) 20:13:05
ちょっと調べるときに信頼性の低い2chで調べるのよりまし
813132人目の素数さん:2007/05/03(木) 20:22:37
人のUNOを読む能力を悪用する奴を潰せ。
814132人目の素数さん:2007/05/03(木) 20:24:11
手札読まれたらゲームにならんよな
815132人目の素数さん:2007/05/03(木) 20:26:03
>>806
他の数学関連のサイト検索した方が早い上に
信頼性も高いと思うんだが。
何でwikipediaみたいなしょうもないところ足がかりにしようとすんの?
一歩目で踏み外して落とし穴に落ちにいくようなものでは。
816132人目の素数さん:2007/05/03(木) 20:27:19
というか、ここって結構レベル高いと言うか、
東大、京大の入試問題ぐらいだと、5分ぐらいで回答返してくるよな。
入った大学で判断しようとするのは安直だが、参考程度に聞く。
皆大体どこの大学出てんの?
817132人目の素数さん:2007/05/03(木) 20:29:36
>>815
お前にとって世の中の事象は数学だけなのかよ。
818132人目の素数さん:2007/05/03(木) 20:30:56
東大・京大の入試問題と言っても、しょせんは高校数学で弄れる範囲なので、
たとえ三流大でも真面目に勉強すれば卒業することにはふつうに解ける。
819132人目の素数さん:2007/05/03(木) 20:31:36
>>817
一応数学板だから数学の話に絞るのは当然だが
他の分野はまともなにかい?
ウィキペディアでは数学を含むいくつかの分野だけが
著しく低品質なのかい?
820132人目の素数さん:2007/05/03(木) 20:32:26
2chには教授とかもいたりするからな…
821132人目の素数さん:2007/05/03(木) 20:34:46
教授が書き込んでると思うと増しておもしろい
822132人目の素数さん:2007/05/03(木) 20:36:17
>>819が必死すぎて痛いNE
823132人目の素数さん:2007/05/03(木) 20:37:11
黄金週間くらいマターリやれや
824132人目の素数さん:2007/05/03(木) 20:39:03
>>819
絞るのか絞らないのかハッキリしろ屑
825132人目の素数さん:2007/05/03(木) 20:40:14
>>824
絞っても絞らなくても
ウィキペディアには落とし穴しかないんだろう?
826132人目の素数さん:2007/05/03(木) 20:40:25
>>818
それは、高校生ぐらいなったら、大体脳は成長しきってて、
今更目を見張るような認知能力の向上はないと、普通思うのが、案外そうでもなく、
高校から大学までの間なんかでも脳は結構成長したりするという意味?
それとも、単純に経験積めば、高校数学程度の問題は解けるようになるということか?
827132人目の素数さん:2007/05/03(木) 20:41:24
>>819
空気嫁
828132人目の素数さん:2007/05/03(木) 20:42:04
>>825
そう思うなら証明してみせよ
829132人目の素数さん:2007/05/03(木) 20:42:52
>>826
> それとも、単純に経験積めば、高校数学程度の問題は解けるようになるということか?

そう
高校数学は正しい方向性で選択された問題群の演習経験が全て
830132人目の素数さん:2007/05/03(木) 20:44:00
>>828
まともな分野があるならしめしてごらんよ。
それが答えだろう。
831132人目の素数さん:2007/05/03(木) 20:45:51
832132人目の素数さん:2007/05/03(木) 20:47:09
それは、分野…なのか?w
卵かけご飯が秀逸なのは認める
833132人目の素数さん:2007/05/03(木) 20:48:41
>>826
2chに東大や京大の人がいるのも確かだし
院生室で話題になることもあるが(俺もいずれかの人w)
だけど、もう一ついうと大学以上になると
問題の裏側というのが見えてくるのも一因。
大学入試で使っていいかどうかよく話題になるロピタルの定理なんかもそうだ。
ロピタルの定理を使えばすぐの問題は
ロピタルの定理を使って解いた後で
ロピタルの定理を表面上使わないように処理すればよい。

大学で習う知識を使えばすぐなのに
というのは結構あるよ。
834132人目の素数さん:2007/05/03(木) 20:50:24
>>831
それが反例になると思ってるあたりが
馬鹿なんだよな…
削除されかけたり
秀逸な記事に推薦されても
ジョークはジョークとしてしか見られなくて落とされた記事を何故持ち出す?
835132人目の素数さん:2007/05/03(木) 20:51:26
悪ふざけとナンセンス候補であってもおかしくないw
836132人目の素数さん:2007/05/03(木) 20:51:42
馬鹿が馬鹿って言うな馬鹿
837132人目の素数さん:2007/05/03(木) 20:52:42
もっとなんとかならないか、って記事は多いけど、なんだかんだいって活用してるし、ないと困るからなぁ。
そもそも自分たち自身にも編集できる権利があるのに、一方的に文句を言うのはどうかと。
せっかくのシステムを役立たせるのも無駄にするのも自分たちなのだから。
838132人目の素数さん:2007/05/03(木) 20:52:47
酷い内容の記事だということで
秀逸とは呼ばれなかったのよ。
まずそこを考えようや。
どうしようもない屑記事。
839132人目の素数さん:2007/05/03(木) 20:53:17
wikiイラネってのを数学板で馬鹿が語り合ってる
滑稽ですね
840132人目の素数さん:2007/05/03(木) 20:54:05
>>837
そんな権利いらないな。
単に使えない信用してはならないサイトってだけ。
こればかりはハッキリ言っておかないと
被害者が増えるしよくない。
841132人目の素数さん:2007/05/03(木) 20:55:14
被害者が屑なだけ
842132人目の素数さん:2007/05/03(木) 20:56:01
結局wikipediaはワンクリック詐欺みたいなもんか。
843132人目の素数さん:2007/05/03(木) 20:57:03
高品質の2ちゃんの回答↓

http://science6.2ch.net/test/read.cgi/math/1176176012/114
844132人目の素数さん:2007/05/03(木) 21:00:35
結局、こいつがwiki大嫌いなのがよく分かっただけであった
845132人目の素数さん:2007/05/03(木) 21:08:48
あれ、クズの掛け合いもう終わったの?ツマンネ
規制でもかかったか
846132人目の素数さん:2007/05/03(木) 21:09:21
いろんな分野のことが一つの書式に則って集められてる
ってことだけがウィキペディアの唯一評価できる価値だ、
といってるのに、他の分野はまともかとか>>819はあほだな
847132人目の素数さん:2007/05/03(木) 21:12:37
少なくとも数学の分野については集まってないようだなw
848132人目の素数さん:2007/05/03(木) 21:13:29
内容が無いか
あってもメチャクチャなのに
書式が揃ってるところが
唯一の価値なんだな
849132人目の素数さん:2007/05/03(木) 21:14:39
ウィキペディアの内容が駄目すぎることについては
誰も否定しようともしないわけねw
850132人目の素数さん:2007/05/03(木) 21:15:46
結局、屑しか集まらなかったから
屑しか無いし
書式を合わせる以外の事ができなかっt
それだけのこと
851132人目の素数さん:2007/05/03(木) 21:16:13
>>847
いろんな分野って言ってるのに数学分野ってあほの子?
852132人目の素数さん:2007/05/03(木) 21:16:16
そんなにいじめるなよかわいそうだろ
853132人目の素数さん:2007/05/03(木) 21:17:11
>>848-849
事実だからしょうがないね。
たまに気が向いて手直ししたら数学知らんような履歴引っさげたガキに荒らし扱いされるし。
854132人目の素数さん:2007/05/03(木) 21:19:28
>>851
ん?内容のある分野でもあんの?
855132人目の素数さん:2007/05/03(木) 21:19:36
Wikiに限らず、百科事典の記述なんてそんなもんだろ。
平凡社の世界大百科でもめくってみ。
856132人目の素数さん:2007/05/03(木) 21:19:37
857132人目の素数さん:2007/05/03(木) 21:22:23
>>851
さっきまともな記事といって
アンサイクロペディアでも全く違和感の無い記事を持ち出した奴がいたよな?
少なくとも彼にとってウィキペディアにあるまともな記事が
どうにもならないアホな屑文だったわけだが

858132人目の素数さん:2007/05/03(木) 21:25:11
大体、なんのために数学板で
ウィキペディアとかいう屑サイトの
いろんな分野について話さねばならないのかー
859132人目の素数さん:2007/05/03(木) 21:25:50
>>854
内容も無いし局所的な分野では記事がほとんど無いが
横断的に数集まってるってとこだけに意味があるって話だ
と何度言えばわかるんだ、この脳無しが
860132人目の素数さん:2007/05/03(木) 21:25:59
書式にしか価値が無いのなら
テンプレートだけ作ってればいいだろうにねw
861132人目の素数さん:2007/05/03(木) 21:26:15
>>856
そりゃオマエらが記事書けって事じゃないの?
862132人目の素数さん:2007/05/03(木) 21:26:46
>>859
数と書式だけにしか価値が無いサイトなんか参考にしたらやばいね
863132人目の素数さん:2007/05/03(木) 21:27:03
>>858
おまえの調べ物は数学に関するもの以外ないのか
という極単純なツッコミがあっただけのことだが。
864132人目の素数さん:2007/05/03(木) 21:27:23
も り  あ   が   っ     て  ま い り ま した
865132人目の素数さん:2007/05/03(木) 21:27:31
2ちゃんって信用していいサイトなのか?
866132人目の素数さん:2007/05/03(木) 21:28:34
>>865
そんなこともわかんねーのか…
867132人目の素数さん:2007/05/03(木) 21:28:37
>>862
そういうこと。どうしても使いたいなら調べ物のとっかかりにしか使えん。
つーはなしだろ。
868132人目の素数さん:2007/05/03(木) 21:29:39
ウィキペディアで調べものして勉強できるとか思い込んでる
厨房がこのスレに沸いたtってだけでもウィキペディアの有害性を語るには十分。
869132人目の素数さん:2007/05/03(木) 21:31:06
>>865
信用しろとは言わないけど
少なくとも画面の向こうには
かなりリアルタイムに回答してくれる人が沢山いるし
嘘言ったら他の回答者が叩きまくるし
かなりマシなサイトだと思うぞ。
分からなければ聞き返せばよい。
ウィキペディアと比べものにはならないくらいまとも。
870132人目の素数さん:2007/05/03(木) 21:31:25
2chで勉強できると思っている奴のいるスレもあるが
871132人目の素数さん:2007/05/03(木) 21:33:13
2chは勉強の足がかりにはなる。
ウィキペディアは勉強の落とし穴にはなる。

こんな感じでどっちも使い道があるような気がする。
足を引っ張りたいライバルがいたらウィキペディアをよろすく
872132人目の素数さん:2007/05/03(木) 21:34:09
>>843は最後まで誰も訂正しなかったけどな
873132人目の素数さん:2007/05/03(木) 21:34:14
数と書式しか意味のないサイトだったのか・・・
874132人目の素数さん:2007/05/03(木) 21:34:53
>>868
その理屈でいけば、世の中のものがどんだけ有害になるんだよ
875132人目の素数さん:2007/05/03(木) 21:35:01
>>872
そもそも
どこのスレの話?
876132人目の素数さん:2007/05/03(木) 21:36:12
>>873
あとはいかに寒いジョークがいえるかの参考くらいには役に立つかも。
877132人目の素数さん:2007/05/03(木) 21:36:48
>>872
訂正しないと言えば、俺の大学の先生が書いた本の一節
ウィキペディアにそのまま写されているんだけど
二年間くらい誰も何もしてないぞw
878132人目の素数さん:2007/05/03(木) 21:37:14
世の中に有害なものはたくさんある
だから、使いようなんジャネーノ…
879132人目の素数さん:2007/05/03(木) 21:37:41
>>839
wikiは便利だから必要。有害で不要なのはwikipedia。
880132人目の素数さん:2007/05/03(木) 21:38:24
>>877
きっとその記事はwikipediaの中では
まともな内容の記事なんだろうねw
881132人目の素数さん:2007/05/03(木) 21:39:09
晒しage
882132人目の素数さん:2007/05/03(木) 21:39:23
>>877
出典があれば何を書いてもいいし、
出典が無くても{{要出典}}さえ貼れば、これも何書いても許される。
そんなサイトだから問題ないでしょ。
883132人目の素数さん:2007/05/03(木) 21:41:56
>>875
【sin】高校生のための数学の質問スレPART122【cos】
http://science6.2ch.net/test/read.cgi/math/1177282598/
884132人目の素数さん:2007/05/03(木) 21:45:42
>>882
> 出典があれば何を書いてもいいし、

んなアホなw
885132人目の素数さん:2007/05/03(木) 21:48:30
有害有害いってる奴らは何がどう具体的に有害だと思っているのか?

自分にとって有用でないというだけの意味であれば
それは自分自身が有効活用できていないだけ。
上手に役立てている賢い人間は山ほどいる。それはGoogleでのHit数が物語っている。

記事が社会的に有害だと思うなら書き直せばいい。
それが不可能だと言うのは、
日本人が低脳で、日本語圏にはまともな記事を書く人間がいないということを
暗に示しているようなもの。

記事の有害性はシステムに根本的な原因があり、改善の余地がないというのであれば
まあしょうがない。
886132人目の素数さん:2007/05/03(木) 21:49:55
>>883
04/28で、ここ2,3日それで騒いでる奴がいるけど
これがウィキペディアだったとしたら
何年放置されていてもおかしくない
887132人目の素数さん:2007/05/03(木) 21:50:01
>>884
実際にそんなサイトだから有害なんだよ
888132人目の素数さん:2007/05/03(木) 21:50:43
>>885
だからなんでウィキペディアのような屑サイトに寄稿してやらなきゃいかんのよ?
889132人目の素数さん:2007/05/03(木) 21:51:24
>>885のようなことを言いながら平気で嘘満載の中身スカスカ記事を
量産するバカが築いた廃墟の山がウィキペディアというサイト。
890132人目の素数さん:2007/05/03(木) 21:52:36
>>887
いまだに引用がどうとかでもめてたりしてるようだが
出典が書いてあっても記事の本文がそのまま写しだったら著作権侵害のような気が。
著作権侵害が許されているおおらかなサイトがウィキペディアってことでいいのかい?
891132人目の素数さん:2007/05/03(木) 21:53:16
スレ違い
消えろ屑ども
892132人目の素数さん:2007/05/03(木) 21:54:09
引用というには要件を満たす必要があるしな。
しかし九割以上が写してもどうせばれることはないだろうし
どんどんやっちゃっていいぞ
893132人目の素数さん:2007/05/03(木) 21:55:19
>>886
>>869が嘘であることに何の関係もないな
894132人目の素数さん:2007/05/03(木) 21:55:18
間違いまみれなだけならまだしも、間違ってるってしてきしただけで、
専門バカだの協調性が無いだの散々に返ってくるようなサイトが、
有害でないとは俺には思えないな。
間違いを書くことが一般人向けに工夫したことなんだそうだ。
895132人目の素数さん:2007/05/03(木) 21:55:56
誰でも編集できるから
気に入らなければあなたも参加すべき
って、どこの悪徳商法だよwwwww
896691:2007/05/03(木) 21:56:10
>693
ありがとうございます.

(ii)については,A∈Bということは,Aはすべてのεに含まれる
ということだから,A^cも必然的にD⊂εなるすべてのσ集合体
εに含まれるので,A∈⇒A^c∈Bということですか?
(iii)についても,A[1],…A[i],…∈Bより,A[1],…A[i],…
はすべてのεに含まれるため∪A[i]もすべてのεに含まれる
ということだと思うのですが,書き方がどうも分かりません

2.については,半開区間は「a∈Rについて
[a,∞)の形式のすべての区間を含むことになる.」との記述で,
ほぼ示されていると思うのですが.
閉区間についても,開区間{(-∞,a),(b,∞)}(この書き方でよいのでしょうか
・・・?)の補集合[a,b]をふくむから,すべての閉区間
を含むっていうイメージですがあってますでしょうか?
しかし,これも書き方がいまいち分かりません.

何度も申し訳ございませんが,よろしくお願いいたします.





897132人目の素数さん:2007/05/03(木) 21:56:39
>>890
YassieとかModehaとかそんな名前の奴らが
どんどんコピペしていい、著作権侵害とか考えなくていい
と宣伝していた。
898691:2007/05/03(木) 21:58:42
すみません,かなり問題が上のほうですので,
読みにくいかと存じますので,スレを消費して申し訳ありませんが,
再度書かせてください.

1.
DをCの部分集合の空でない集まりと仮定する(C:標本空間).
事象の全体
B=∩{ε:D⊂ε かつ εはσ集合体}
を考える.Φ∈Bであることに注意せよ.なぜなら,Φはそれぞれのσ集合体
に含まれ,したがって,とくにこれは各々のσ集合体ε⊃Dに含まれるからである.
同様に続けてBがσ集合体であることを示せ.

2.
C=Rとする.ここにRはすべての実数の集合である.また,IをRのすべての開区間とする.
いま,実数直線状のボレルσ集合体を考える.つまり
B0=∩{ε:I⊂ε かつ εはσ集合体}
によって与えられるσ集合体B0である.定義より,B0は開区間を含む.
[a,∞)=(-∞,a)^c であり,B0は補集合について閉じているので,a∈Rについて
[a,∞)の形式のすべての区間を含むことになる.このようにして,B0が実数上の
すべての閉区間と半開区間を含むことを示せ.
899132人目の素数さん:2007/05/03(木) 21:59:13
>>896
書き方が分からないというのは
どの部分が?
900132人目の素数さん:2007/05/03(木) 22:00:57
ω∈B⇔ω∈∀ε
なんだから1.は自明
901132人目の素数さん:2007/05/03(木) 22:01:47
>>893
(ii)OK(iii)もそれで証明は終わってる。
902132人目の素数さん:2007/05/03(木) 22:02:48
>>896
たまには、有限半開区間のことも、少しでもいいから、思いだしてあげてください
903132人目の素数さん:2007/05/03(木) 22:10:23
>>898
性質(ii)(iii)から、加算無限個の共通部分∩A[i]もBに含まれることが示せるので
A[n]=(-1-(1/n),1+(1/n))なんかをヒントに好きなだけ閉区間作っちゃってください。
904691:2007/05/03(木) 22:51:35
早速のご返信ありがとうございます.
>>900
やはり自明ですか・・・.
>>899
たとえば,{(-∞,a),(b,∞)}という書き方はいいのでしょうか?
また,考え方の流れとしては,私が書いたとおりでよいのでしょうか?
いまいち自信がありません.

よろしくお願いいたします.
905132人目の素数さん:2007/05/03(木) 23:06:07
{(-∞,a),(b,∞)}∈B0×B0だな。
906691:2007/05/03(木) 23:57:19
>> 901-903,905
ありがとうございます.
もう少しがんばってみます.
907132人目の素数さん:2007/05/04(金) 09:54:56
あるサイコロを30回振ったら数字の1が5回出た

問題:
1/6を30回したとき1が5回出る確率と1/5を30回したとき1が5回出る確率を求める式を答えよ。
さらに5面体と6面体のどちらのサイコロが使われていたか和が1になるように答えよ。

1/6が30回したとき1が5回出る確率を求める式
=
1/5が30回したとき1が5回出る確率を求める式
=
それぞれの割合(5面体率+6面体率=1)
=

こういう問題ですが、このスレでいいですか?
確率論だとは思うんですが、ここは問題全般でしょうか?
大丈夫でしたら回答お願いします。
908132人目の素数さん:2007/05/04(金) 10:53:23
>>907
1/6が30回したとき1が5回出る確率を求める式
=(30C5) (1/6)^5 (5/6)^(30-5)≒0.192108
1/5が30回したとき1が5回出る確率を求める式
=(30C5) (1/5)^5 (4/5)^(30-5)≒0.1722791828
それぞれの割合(5面体率+6面体率=1)
=問題の意味が不明
909132人目の素数さん:2007/05/04(金) 11:40:59
解析学の問題なのですが、集合列Aiに対して(i=1,2・・)

@limInfAi⊂limSupAi(i→∞)


AAi⊂Ai+1ならばlimAiが存在し、
limAi(i→∞)=∪Ai(∪はi=1からi=∞までの和集合)

BAi⊃Ai+1ならばlimAiが存在し、
limAi(i→∞)=∩Ai(∩はi=1からi=∞までの積集合)

@ABを証明してほしいのですが、よろしくお願いします。

910132人目の素数さん:2007/05/04(金) 13:08:14
「合成写像gfが単射ならばfは単射」
であることを示してください。できれば10分以内に。
911132人目の素数さん:2007/05/04(金) 13:13:38
f(x)=f(y)とするとgf(x)=gf(y) gfが単車だからx=y
912132人目の素数さん:2007/05/04(金) 13:14:35
「合成写像gfが単射ならばfは単射」
を示してください。
913132人目の素数さん:2007/05/04(金) 13:15:23
あ、ありがとうございます。なんか間違えて二回かきこんですみません。
914907:2007/05/04(金) 14:30:21
>>908
ありがとうございます。

>それぞれの割合(5面体率+6面体率=1)
>=問題の意味が不明

多分なんですけど、1/6のサイコロであったのか1/5のサイコロであったのかってことだと思うんです。
だから1/6のサイコロであった可能性が80%で1/5のサイコロであった可能性が20%みたいな意味だと思うんです。

0.192108 + 0.1722791828 ≠ 1
ですよね?つまりこれを比率にしてから0.6と0.4みたいな感じかと。
0.192108:0.1722791828 → 
そんな計算方法あるんですかね。
両方を同じだけ乗算していって和が1になるところを見つけるのかな。
もしかしたら意味が違うかもしれません。
915132人目の素数さん:2007/05/04(金) 14:53:28
直線y=mxが円x^2+y^2-6x+5=0を切る弦の中点の軌跡を求めよ。

直線の式を円の式に代入して判別式でmの範囲を求めて、
次に交点を求めたのですがmが残ります。

mの消し方とか上手な計算方法等お願いします。
916132人目の素数さん:2007/05/04(金) 15:00:26
>>915
交点は求めない

x^2+(mx)^2-6x+5=0 の2解を a, b とすると
題意の点は ((a+b)/2, m(a+b)/2)
917132人目の素数さん:2007/05/04(金) 15:05:35
その中点をP(p,q)
p,qをmで表したら、
・p,qの範囲を出す
・mを消去してp,qの関係式を導く
918132人目の素数さん:2007/05/04(金) 15:10:14
妹の宿題なのですが、何とも言えないので、救ってくださると助かります。

「0を正の数でわっても、負の数でわっても、商は0になる。
また、どのような数も0でわることはできない。」

何故0でわる事は出来ないのか、理由を教えて頂きたいです。お願いいたします。
919132人目の素数さん:2007/05/04(金) 15:11:05
妹は何才だ?
920132人目の素数さん:2007/05/04(金) 15:14:12
12才中1です。
921132人目の素数さん:2007/05/04(金) 15:17:35
きくまえにぐぐったらどうなのさ
これとかわかりやすいんじゃないの
http://www.kwansei.ac.jp/hs/z90010/sugakua/base/base.htm
922132人目の素数さん:2007/05/04(金) 15:19:05
>>920
0で割ってしまったら、その夜には数字の悪魔に食われてしまうんだ
もう既に多くの人が食われてしまった
923132人目の素数さん:2007/05/04(金) 15:20:40
決められているから。
924132人目の素数さん:2007/05/04(金) 15:35:38
割り算を掛け算の逆演算と定義してるから0で割る事を定義しない方が便利だから
925132人目の素数さん:2007/05/04(金) 15:40:13
>>918
割る話の前の話だが、
どんな数でも0を掛けたら0になるというのは、説明できてるんかな?
926918:2007/05/04(金) 15:58:05
>>925それは大丈夫です。サイト様を見て、0を使うかけ算結果を使って説明すれば良いことに、遅くなりましたが気づきました。

教えて頂いたサイト様の文をまとめて妹に教えたら、納得してくれました!
助かりました、ありがとうございました!
927132人目の素数さん:2007/05/04(金) 16:02:50
サイト様
928915:2007/05/04(金) 16:45:49
>>916,917
p=3/(1+m^2) , q=3m/(1+m^2)
がでてきたのですが、mが消去できません。
また、p , qの範囲はどうすればいいのですか?
mの範囲は-2<m<2とわかったんですが、そこからどうすればいいか分かりません。
再度お願いします。
929132人目の素数さん:2007/05/04(金) 17:03:49
>>928
1+m^2 = 3/p を他方に代入した後 m について解け
930132人目の素数さん:2007/05/04(金) 17:04:18
そこまでできたのなら、p=3/(1+m^2) より 1+m^2 = 3/p
(これを q = … の分母に代入)、m = ±√(3/p - 1)
(これを分子に代入)すればよいでしょう。
931132人目の素数さん:2007/05/04(金) 17:05:10
>>929-930
q/p=mのほうが楽じゃね?
932132人目の素数さん:2007/05/04(金) 17:07:15
q=m*p
q^2=m^2*p^2
p=3/(1+m^2)よりm^2消去

図描けばわかるがpの範囲だけでおk
1<=1+m^2<5, 3/5<p<=3

とゆー感じ
933907:2007/05/04(金) 17:07:57
>>914の割合のところだけ誰かわかりませんか?
1対1 = 50% : 50% → 0.5+0.5=1
のように
1対2対3対4を足したら1になるようにそれぞれを等しい倍率で小さくして
総和が1にするための計算方法が知りたいです。
934132人目の素数さん:2007/05/04(金) 17:09:58
>>933
和で割れ
935132人目の素数さん:2007/05/04(金) 17:12:30
>>929-930
よく他人に教えられるね
936915:2007/05/04(金) 17:12:48
>>929-932
ありがとうございました!
937907:2007/05/04(金) 17:14:59
一応自分としては

1+2+3+4=10なので
1/10 : 2/10 : 3/10 : 4/10
だと思います。
3対4対5対6
なら総和の分母でそれぞれを割れば少数表記になりますよね?
その少数を足せばおよそ1になるはずですけど。
合ってるんでしょうか。何か関数みたいなのあるんですかね。
938907:2007/05/04(金) 17:16:15
>>934
すいません。ちょうど書き込みチュでした。
それも937分母とか意味不明な事言ってました。
とりあえず和で割れば正解ということでスッキリ全部解決です。
ありがとうございました。
939907:2007/05/04(金) 17:21:16
すいません最後に一つお願いします。
「和で割れ」
という今回の場合のことを
「総和で除算しろ」
って言ってもいいんですか?
それとも総和ってこういう時に使う言葉じゃありませんか?

ちょっとかっこよく言ってみたくなりまして^^;
940132人目の素数さん:2007/05/04(金) 17:29:45
>>937
好きにしろ
ただ難しく言うよりわかりやすく言うほうがいいと思うのだが
941132人目の素数さん:2007/05/04(金) 17:33:12
>>939
ま、かっこ付けがかっこ悪いってのはよく知られた事実だからね。
942907:2007/05/04(金) 17:47:08
>>940-941
一応意味としてはいいのですね。ありがとうございました。GW中なのにすいませんでしたm(__)m
943132人目の素数さん:2007/05/04(金) 17:58:30
>>907
問題の趣意は以前不明なのだが、これをベイズ統計(ベイズの定理)を
使用するべきものと解釈する。サイコロを30回振ったら1の目が5回
でたという観測結果から、6種の目の出るサイコロが使われたか(確率
p)、5種のサイコロが使われたか(確率q)推定せよ、というものだ。

事前に情報はないので、どちらを使う確率も同じ、p=q=1/2としておく。

pのもとに1の目が 5回出るのは >>908 の計算を採用すれば0.19
だ。q ならば 0.17だ。

実際に目が 5回出たという結果から、その原因を推定すると、
pとなっていたのは p' = (1/2)×0.19 / ((1/2)×0.19 + (1/2)×0.17)
とすればよい、と計算するのかな?


1/6が30回したとき1が5回出る確率を求める式
=(30C5) (1/6)^5 (5/6)^(30-5)≒0.192108
1/5が30回したとき1が5回出る確率を求める式
=(30C5) (1/5)^5 (4/5)^(30-5)≒0.1722791828
それぞれの割合(5面体率+6面体率=1)
=問題の意味が不明
944132人目の素数さん:2007/05/04(金) 18:56:52
初項から第n項までの和がSn=2n^2+3で表せるとき、
この数列の一般項を求めよ。

Sn-S(n-1) で求めとおかしくなりました。お願いします。
945KingOfUniverse ◆667la1PjK2 :2007/05/04(金) 19:00:21
talk:>>944 おかしい?
946132人目の素数さん:2007/05/04(金) 19:02:59
>>944
自分の書いた文章よく見てみろよ
これでお願いしますって頭おかしいぜ
947944:2007/05/04(金) 19:10:39
求めと⇒求めると の所ですか?すいません。

S(n-1)=2(n-1)^2+3=2n^2-4n+5 で、Sn-S(n-1)=4n-2なんです。
Sn-S(n-1)で求める解法ではないんですか?
948132人目の素数さん:2007/05/04(金) 19:13:48
お前がおかしいと思ったところを理由を添えて具体的に書いてもらわないと
お前がおかしいと思ったところが全然分からないってことだよ
949132人目の素数さん:2007/05/04(金) 19:25:38
>>947
> S(n-1)=2(n-1)^2+3=2n^2-4n+5 で、Sn-S(n-1)=4n-2なんです。

> Sn-S(n-1)で求める解法ではないんですか?
のあいだに一体お前の脳内ではどんな珍説が補完されてるんだ?
950944:2007/05/04(金) 19:26:49
了解。
a1=S1=5で、4n-2だと、n=1のとき2で、初項が合わないんです。
951132人目の素数さん:2007/05/04(金) 19:28:05
a[n]=S[n]-S[n-1], n>=2
            ~~~~~
952132人目の素数さん:2007/05/04(金) 19:28:42
>>950
合わないからどうした?
953132人目の素数さん:2007/05/04(金) 19:30:02
赤玉と白玉が半分ずつ入ってる箱から
5個を取り出した場合、全部同じ色の確率はどれくらいなのでしょうか?
また、一つだけ赤の場合の確率はどれくらいでしょうか・・・?

どうしてもわかりません・・・宜しくお願いします
954944:2007/05/04(金) 19:32:34

n=1のときと、n>=2のときで分ければいいんですね?
どんな場合でもn=1のときまで保証できると思ってました。
955132人目の素数さん:2007/05/04(金) 19:32:55
>赤玉と白玉が半分ずつ入ってる箱

半球ってことかな?
956132人目の素数さん:2007/05/04(金) 19:34:35
>>954
おk
a[1]=S[1]-S[0]だと言ったなら、S[0]って何?ということになる
分けて調べる
957132人目の素数さん:2007/05/04(金) 19:43:55
和から一般項を求める問題と、階差数列を使って和を求める問題は
セットだと思った方がいい(そもそも同じことだし)。
958132人目の素数さん:2007/05/04(金) 21:11:32
四十六日。
959944:2007/05/04(金) 21:18:08
Sn=1+2x+3x^2+4x^2+…+nx^(n-1) を求めよ。
Snにxをかけて、Snと引いたんですけど、その後どうすればいいか分かりません。
nx^nをどう処理すればいいんですか?
960959:2007/05/04(金) 21:19:26
944ではないです。
念のため。
961132人目の素数さん:2007/05/04(金) 21:24:15
>>959
nx^nはそのまま置いとく
x+x^2+...+x^(n-1)を公式に当てはめ
962959:2007/05/04(金) 21:32:07
わかりました。
等比数列の和ですね。
求めよって書いてあるから何か数字がでるのかと思いました。
Snの式みたいなのを求めておわりなんですね。
963907:2007/05/04(金) 22:12:50
>>943
p' = (1/2)×0.19 / ((1/2)×0.19 + (1/2)×0.17)
1/2が3箇所にありますよね。
もしも8割方、5面サイコロを使う傾向があるデータがあれば

p' = (1/5)×0.19 / ((1/5)×0.19 + (4/5)×0.17)
とすればいいのですか?

それとこういうのは尤度設定とかいうものですか?
964943:2007/05/04(金) 23:10:03
>>963
うん。事前確率が推定できるなら、上記のように計算するらしい。
オレは実はベイズ統計は苦手で…。どうも感覚的に受け付けない。
965907:2007/05/04(金) 23:13:58
>>964
尤度というか事前確率っていうんですね。尤度でもおかしくはないのかな。よくわかんないですけど。
でもありがとうございました。事前確率というか傾向というか。
それがあるとなしじゃ全然違ってきますよね。
もちろん事前確率の推測は結局あくまで推測にすぎないわけですが。
不確定要素を少しでも減らす努力は確率関係の計算においては重要ですね。

ありがとうございました。
966907:2007/05/04(金) 23:22:54
p' = (1/5)×0.19 / ((1/5)×0.19 + (4/5)×0.17)
というか計算してみたんですが。
6面体の事前確率が0.2だと
(30C5) (1/6)^5 (5/6)^(30-5)
これ自体は1.8%程度しか影響を及ぼさないんですね。
要は30回くらいの結果から1/30の抽選確率?の差は浮き彫りになってこないってことですね。

この1/30の差がうきぼりになるであろう抽選回数って計算できるんですか?
967132人目の素数さん:2007/05/04(金) 23:34:46
馬鹿ですいません
7:3X=4:(X+5)

(X−3):2=5X:12
Xを求めろ 

すいません解説して下さい
968132人目の素数さん:2007/05/05(土) 00:05:05
>>967
比の式の内側と外側をそれぞれかけてできた方程式を解けばよい。
969132人目の素数さん:2007/05/05(土) 00:26:20
>>967
マルチ死ね
970132人目の素数さん:2007/05/05(土) 04:34:26
おやすみking
971132人目の素数さん:2007/05/05(土) 07:15:03
9冊の異なる絵本を5冊、2冊、2冊の三組に分ける方法は何通りあるか?
という問題で答えが、(9C5*4C2*2C2)/2!と書かれていました。
これはつまり図で書くと、5冊をabcde、2冊をfg、2冊をhiとした場合。

図 

組みに区別がある場合            組みに区別なし
(abcde)(fg)(hi)               (abcde)(fg)(hi) 
(abcde)(hi)(fg)               (fg)(hi)(abcde)
(fg)(abcde)(hi)               (fg)(abcde)(hi)
    ・                      の3通り。
    ・ 
    ・
等、3!通り。

となりますか?

今回は2冊、2冊に区別はないが、5冊とは区別しなきゃいけないので
上のような場合の数になり、組みに区別がある場合はない場合の2!倍になっているので
組みに区別がある場合の総数(9C5*4C2*2C2)を2!で割ったものが答えとなるわけですか?
答えは本に載っているのですが、図が載っていなかったので
別の問題の図を参考に自分で考えたんですが
図が合ってるか合っていないか教えてください。
972132人目の素数さん:2007/05/05(土) 08:08:45
>>971
わざわざ、ワケのわからん図を描いて
理解を阻害することもないと思うが。

まあ
>組みに区別がある場合の総数(9C5*4C2*2C2)を2!で割ったものが答えとなるわけですか?
が間違ってるわけじゃないからいいんだけど
この程度の問題は、図なんかに頼らず
式だけでイメージできるようにしとけ。な。
973132人目の素数さん:2007/05/05(土) 08:17:20
>>972
図があった方が理解しやすいんですよね。
2!で割ると言う事は、組みに区別がある場合の数は区別がない場合の数の2!倍になっていると言う事。
区別がある場合の数は一つの並べ方に付き3!通り。 と言う事は区別が無い方は3通りになるはず。
で、3通り書いてみたわけなんですが・・・・。
974132人目の素数さん:2007/05/05(土) 08:22:45
>>973
区別をつけるつけない以前に、5-2-2なら
5だけは自動的に区別されるだろ。
975132人目の素数さん:2007/05/05(土) 08:28:10
>>974
まぁ・・・そうなんですが・・・図で示すとすると、こんな感じなのかな?
と自分で考えてみたんですが、それが合ってるのかどうなのか解らなくて・・・・。
976943:2007/05/05(土) 10:36:44
>>966
まず苦言を呈しておくが、「5面体のサイコロ」などと軽々しく言わない
でほしい。どんな形か見当もつかない。1から5までの目が均等に出ること
を納得させるサイコロの形状は、たとえば正20面体を使って、同じ数字を
4回ずつ振ったようなものだろう。
977943:2007/05/05(土) 10:46:16
>>966
6目のサイコロで各目の出現率は A=1/6だ。5目なら B=1/5 だ。当然
B>A。振ってみての観測結果からどちらが使われたかを推定するのは、
ベイズ統計ではなく、仮説検定のテーマであろう。その検定を可能に
する回数は、大づかみに

 √n > α×(√(A(1-A) + √(B(1-B))/(B-A)

となる。上の A, B の数値を入れ、α=1とすれば、この回数 n はだいたい
537回となり、どちらのサイコロが使われたかを確信を持って推定(検定)
するためには、このくらいの試行を必要とする。αは、よく「nσの検定」
という用語が使われるが、ほぼそれに相当するパラメータと思ってくれ。

特定の目の出現回数の統計しかない場合は上のような計算だが、もし
複数の出目について統計が知られているなら、これはカイ2乗検定を適用
すべき問題となる。

ある統計的推定のために、標本のサイズをどうすべきかは、標本論という
古典的テーマで論じられている。
978132人目の素数さん:2007/05/05(土) 10:50:24
>>976
シツモーン
このとき4回ずつ書く書き込みパターンは5つの数字どれをとっても同じ形、同じ配置にできますか?
確率的にはどの面も1/20で出現するから、どのように書き込んでも、
どの数字についてもそれが出る確率が1/5になるのはわかるけど、
心理的には、形状的に対称になっていて欲しいから。
979907:2007/05/05(土) 11:11:05
>>976-977
5面体すいませんです。

なるほどです。
1/30の差であるかではなくて、1/5と1/6という事が重要なんですね。
49999/50000と14997/15000も1/30の差ですが1/5と1/6の時とは又違った
標本数が必要になってくるわけですね。

公式ありがたく頂きますm(__)m
一杯ありがとうございます!
980132人目の素数さん:2007/05/05(土) 11:16:42
18*3^x+7>3^-x
やりかた教えてください
981132人目の素数さん:2007/05/05(土) 11:18:14
>980
やり方とは?
まず こくご の べんきょう しよう
982132人目の素数さん:2007/05/05(土) 11:20:35
>978
少しは検索して考えろ。正20面体の図ぐらいあるだろ。
983132人目の素数さん:2007/05/05(土) 11:29:23
>981
解き方です
984132人目の素数さん:2007/05/05(土) 11:35:59
>>980
y = 3^xとおく。y>0
18y+7>1/y
18y^2 +7y-1>0
(2y+1)(9y-1)>0
y>0だから
9y-1>0
y>1/9 = 3^(-2)

x>-2
985132人目の素数さん:2007/05/05(土) 11:36:40
分からない問題はここに書いてね275
http://science6.2ch.net/test/read.cgi/math/1178332358/
986犬笠銀次郎:2007/05/05(土) 12:11:38
>>978
無理があるような希ガス。

http://ginjiro.blogspot.com
987132人目の素数さん:2007/05/05(土) 13:34:07
微分係数の定義の形にしたいのですが、変形の方法が分かりません。

問、f(x)が微分可能な時、次の極限を求めよ


      f(a+h)+f(a-h)-2f(a)
lim ――――――――――――――――――
h→0       h^2


宜しくお願いします。
988132人目の素数さん:2007/05/05(土) 13:49:51
これは力作
989132人目の素数さん:2007/05/05(土) 13:54:17
>>987
h^2の項まで展開して分子を書き下せ。
990132人目の素数さん:2007/05/05(土) 13:57:06
>>987
すっげえ見やすいね。
ちょっと 1/8 + 2/3 + 1/4 をそれでやってみてよ。
991132人目の素数さん:2007/05/05(土) 13:59:38
次からはこう変形してNE

lim[h→0](f(a+h)+f(a-h)-2f(a))/h^2
992132人目の素数さん:2007/05/05(土) 14:14:00
>>989
ありがとうございます。低能ですが、アドバイス通りに頑張らせて頂きます。
>>990
今は携帯からでして、間隔が解りません。申し訳ありません。
>>991
了解しました。次回からは一行にまとめさせて頂きます。ご指摘ありがとうございます。
993132人目の素数さん:2007/05/05(土) 14:39:37
次のように 2段階に lim をとるようにすれば、
lim(f(a+h)+f(a-h)-2f(a))/h^2
 = lim(lim(f(a+h)-f(a))/h - lim(f(a)-f(a-h))/h)/h
 = lim(f'(a)-f'(a-h))/h = f''(a)。
ただ、1段階だった lim を 2段にしてよいかは、議論のあるところ。
994132人目の素数さん:2007/05/05(土) 14:45:29
テーラー展開すべきところだろうが、条件が少し弱いな。
995132人目の素数さん:2007/05/05(土) 14:46:30
0/0の不定形になるから、ろぴたる使え。
996132人目の素数さん:2007/05/05(土) 14:47:45
>>987が条件書き忘れてるんじゃね
997132人目の素数さん:2007/05/05(土) 14:49:01
f(x)のテーラー展開可能性を仮定し(これは必ずしも微分可能性だけ
では保障されないと思う)
f(a+h) = f(a) + f'(a)h + (1/2)f''(a)h^2 + o(h^3)
f(a-h) = f(a) - f'(a)h + (1/2)f''(a)h^2 + o(h^3)
とおけば、自動的に解ける。
998132人目の素数さん:2007/05/05(土) 14:53:38
>>993のやりかたは f'(x)がx=aの近傍で微分可能でないと意味のない式だしね。
999132人目の素数さん:2007/05/05(土) 14:53:48
>>993-995
ありがとうございます。
工校卒の私には、wiki読んでも手も足も出ませんでした。本当に助かります。
1000132人目の素数さん:2007/05/05(土) 14:58:56
分からない問題はここに書いてね275
http://science6.2ch.net/test/read.cgi/math/1178332358/
↑次スレ

>>997-998
ありがとうございます。メモらせて頂きました。
皆様、とても聡明な方々なんですね。
10011001
このスレッドは1000を超えました。
もう書けないので、新しいスレッドを立ててくださいです。。。